100
29 A Modern Introduction to Probability and Statistics Full Solutions February 24, 2006 F.M. Dekking, C. Kraaikamp, H.P. Lopuha¨a, L.E. Meester

Probability and Statistics Book Solutions

Embed Size (px)

Citation preview

Page 1: Probability and Statistics Book Solutions

29

A Modern Introduction to Probability andStatistics

Full Solutions

February 24, 2006

©F.M. Dekking, C. Kraaikamp, H.P. Lopuhaa, L.E. Meester

Page 2: Probability and Statistics Book Solutions

458 Full solutions from MIPS: DO NOT DISTRIBUTE

29.1 Full solutions

2.1 Using the relation P(A ∪B) = P(A)+P(B)−P(A ∩B), we obtain P(A ∪B) =2/3 + 1/6− 1/9 = 13/18.

2.2 The event “at least one of E and F occurs” is the event E ∪ F . Using thesecond DeMorgan’s law we obtain: P(Ec ∩ F c) = P((E ∪ F )c) = 1 − P(E ∪ F ) =1− 3/4 = 1/4.

2.3 By additivity we have P(D) = P(Cc ∩D)+P(C ∩D). Hence 0.4 = P(Cc ∩D)+0.2. We see that P(Cc ∩D) = 0.2. (We did not need the knowledge P(C) = 0.3!)

2.4 The event “only A occurs and not B or C” is the event A ∩ Bc ∩ Cc. Wethen have using DeMorgan’s law and additivity

P(A ∩Bc ∩ Cc) = P(A ∩ (B ∪ C)c) = P(A ∪B ∪ C)− P(B ∪ C) .

The answer is yes , because of P(B ∪ C) = P(B) + P(C)− P(B ∩ C)

2.5 The crux is that B ⊂ A implies P(A ∩B) = P(B). Using additivity we obtainP(A) = P(A ∩B)+P(A ∩Bc) = P(B)+P(A \B). Hence P(A \B) = P(A)−P(B).

2.6 a Using the relation P(A ∪B) = P(A) + P(B) − P(A ∩B), we obtain 3/4 =1/3 + 1/2− P(A ∩B), yielding P(A ∩B) = 4/12 + 6/12− 9/12 = 1/12.

2.6 b Using DeMorgan’s laws we get P(Ac ∪Bc) = P((A ∩B)c) = 1−P(A ∩B) =11/12.

2.7 P((A ∪B) ∩ (A ∩B)c) = 0.7.

2.8 From the rule for the probability of a union we obtain P(D1 ∪D2) ≤ P(D1) +P(D2) = 2 · 10−6. Since D1 ∩ D2 is contained in both D1 and D2, we obtainP(D1 ∩D2) ≤ minP(D1) , P(D2) = 10−6. Equality may hold in both cases: forthe union, take D1 and D2 disjoint, for the intersection, take D1 and D2 equal toeach other.

2.9 a Simply by inspection we find thatA = TTH, THT, HTT, B = TTH, THT, HTT, TTT,C = HHH, HHT, HTH, HTT, D = TTT, TTH, THT, THH.2.9 b Here we find that Ac = TTT, THH, HTH, HHT, HHH,A ∪ (C ∩D) = A ∪ ∅ = A, A ∩Dc = HTT.2.10 Cf. Exercise 2.7: the event “A or B occurs, but not both” equals C = (A∪B)∩(A ∩ B)c Rewriting this using DeMorgan’s laws (or paraphrasing “A or B occurs,but not both” as “A occurs but not B or B occurs but not A”), we can also writeC = (A ∩Bc) ∪ (B ∩Ac).

2.11 Let the two outcomes be called 1 and 2. Then Ω = 1, 2, and P(1) = p, P(2) =p2. We must have P(1) + P(2) = P(Ω) = 1, so p + p2 = 1. This has two solutions:p = (−1 +

√5 )/2 and p = (−1−√5 )/2. Since we must have 0 ≤ p ≤ 1 only one is

allowed: p = (−1 +√

5 )/2.

2.12 a This is the same situation as with the three envelopes on the doormat, butnow with ten possibilities. Hence an outcome has probability 1/10! to occur.

2.12 b For the five envelopes labeled 1, 2, 3, 4, 5 there are 5! possible orders, andfor each of these there are 5! possible orders for the envelopes labeled 6, 7, 8, 9, 10.Hence in total there are 5! · 5! outcomes.

Page 3: Probability and Statistics Book Solutions

29.1 Full solutions 459

2.12 c There are 32·5!·5! outcomes in the event “dream draw.” Hence the probabilityis 32 · 5!5!/10! = 32 · 1 · 2 · 3 · 4 · 5/(6 · 7 · 8 · 9 · 10) = 8/63 =12.7 percent.

2.13 a The outcomes are pairs (x, y).

The outcome (a, a) has probability 0 tooccur. The outcome (a, b) has probability1/4× 1/3 = 1/12 to occur.The table becomes:

a b c d

a 0 112

112

112

b 112

0 112

112

c 112

112

0 112

d 112

112

112

0

2.13 b Let C be the event “c is one of the chosen possibilities”. Then C =(c, a), (c, b), (a, c), (b, c). Hence P(C) = 4/12 = 1/3.

2.14 a Since door a is never opened, P((a, a)) = P((b, a)) = P((c, a)) = 0. If the can-didate chooses a (which happens with probability 1/3), then the quizmaster chooseswithout preference from doors b and c. This yields that P((a, b)) = P((a, c)) = 1/6.If the candidate chooses b (which happens with probability 1/3), then the quizmas-ter can only open door c. Hence P((b, c)) = 1/3. Similarly, P((c, b)) = 1/3. Clearly,P((b, b)) = P((c, c)) = 0.

2.14 b If the candidate chooses a then she or he wins; hence the correspondingevent is (a, a), (a, b), (a, c), and its probability is 1/3.

2.14 c To end with a the candidate should have chosen b or c. So the event is(b, c), (c, b) and P((b, c), (c, b)) = 2/3.

2.15 The rule is:

P(A ∪B ∪ C) = P(A)+P(B)+P(C)−P(A ∩B)−P(A ∩ C)−P(B ∩ C)+P(A ∩B ∩ C) .

That this is true can be shown by applying the sum rule twice (and using the setproperty (A ∪B) ∩ C = (A ∩ C) ∪ (B ∩ C)):

P(A ∪B ∪ C) = P((A ∪B) ∪ C) = P(A ∪B) + P(C)− P((A ∪B) ∩ C)

= P(A) + P(B)− P(A ∩B) + P(C)− P((A ∩ C) ∪ (B ∩ C))

= s− P(A ∩B)− P((A ∩ C))− P((B ∩ C)) + P((A ∩ C) ∩ (B ∩ C))

= s− P(A ∩B)− P(A ∩ C)− P(B ∩ C) + P(A ∩B ∩ C) .

Here we did put s := P(A) + P(B) + P(C) for typographical convenience.

2.16 Since E ∩ F ∩ G = ∅, the three sets E ∩ F , F ∩ G, and E ∩ G are disjoint.Since each has probability 1/3, they have probability 1 together. From these twofacts one deduces P(E) = P(E ∩ F )+P(E ∩G) = 2/3 (make a diagram or use thatE = E ∩ (E ∩ F ) ∪ E ∩ (F ∩G) ∪ E ∩ (E ∩G)).

2.17 Since there are two queues we use pairs (i, j) of natural numbers to indicatethe number of customers i in the first queue, and the number j in the second queue.Since we have no reasonable bound on the number of people that will queue, wetake Ω = (i, j) : i = 0, 1, 2, . . . , j = 0, 1, 2, . . . .2.18 The probability r of no success at a certain day is equal to the probabilitythat both experiments fail, hence r = (1 − p)2. The probability of success for thefirst time on day n therefore equals rn−1(1− r). (Cf. Section2.5.)

Page 4: Probability and Statistics Book Solutions

460 Full solutions from MIPS: DO NOT DISTRIBUTE

2.19 a We need at least two days to see two heads, hence Ω = 2, 3, 4, . . . .2.19 b It takes 5 tosses if and only if the fifth toss is heads (which has probabilityp), and exactly one of the first 4 tosses is heads (which has probability 4p(1− p)3).Hence the probability asked for equals 4p2(1− p)3.

3.1 Define the following events: B is the event “point B is reached on the secondstep,” C is the event “the path to C is chosen on the first step,” and similarly wedefine D and E. Note that the events C, D, and E are mutually exclusive and thatone of them must occur. Furthermore, that we can only reach B by first going to Cor D. For the computation we use the law of total probability, by conditioning onthe result of the first step:

P(B) = P(B ∩ C) + P(B ∩D) + P(B ∩ E)

= P(B |C) P(C) + P(B |D) P(D) + P(B |E) P(E)

=1

3· 1

3+

1

4· 1

3+

1

3· 0 =

7

36.

3.2 a Event A has three outcomes, event B has 11 outcomes, and A ∩ B =(1, 3), (3, 1). Hence we find P(B) = 11/36 and P(A ∩B) = 2/36 so that

P(A |B) =P(A ∩B)

P(B)=

2/36

11/36=

2

11.

3.2 b Because P(A) = 3/36 = 1/12 and this is not equal to 2/11 = P(A |B) theevents A and B are dependent.

3.3 a There are 13 spades in the deck and each has probability 1/52 of being chosen,hence P(S1) = 13/52 = 1/4. Given that the first card is a spade there are 13−1 = 12spades left in the deck with 52 − 1 = 51 remaining cards, so P(S2 |S1) = 12/51. Ifthe first card is not a spade there are 13 spades left in the deck of 51, so P(S2 |Sc

1) =13/51.

3.3 b We use the law of total probability (based on Ω = S1 ∪ Sc1):

P(S2) = P(S2 ∩ S1) + P(S2 ∩ Sc1) = P(S2 |S1) P(S1) + P(S2 |Sc

1) P(Sc1)

=12

51· 1

4+

13

51· 3

4=

12 + 39

51 · 4 =1

4.

3.4 We repeat the calculations from Section 3.3 based on P(B) = 1.3 · 10−5:

P(T ∩B) = P(T |B) · P(B) = 0.7 · 0.000 013 = 0.000 0091

P(T ∩Bc) = P(T |Bc) · P(Bc) = 0.1 · 0.999 987 = 0.099 9987

so P(T ) = P(T ∩B) + P(T ∩Bc) = 0.000 0091 + 0.099 9987 = 0.100 0078 and

P(B |T ) =P(T ∩B)

P(T )=

0.000 0091

0.100 0078= 0.000 0910 = 9.1 · 10−5.

Further, we find

P(T c ∩B) = P(T c |B) · P(B) = 0.3 · 0.000 013 = 0.000 0039

and combining this with P(T c) = 1− P(T ) = 0.899 9922:

P(B |T c) =P(T c ∩B)

P(T c)=

0.000 0039

0.899 9922= 0.000 0043 = 4.3 · 10−6.

Page 5: Probability and Statistics Book Solutions

29.1 Full solutions 461

3.5 Define the events R1 and R2 meaning: a red ball is drawn on the first andsecond draw, respectively. We are asked to compute P(R1 ∩R2). By conditioningon R1 we find:

P(R1 ∩R2) = P(R2 |R1) · P(R1) =3

4· 1

2=

3

8,

where the conditional probability P(R2 |R1) follows from the contents of the urnafter R1 has occurred: one white and three red balls.

3.6 a Let E denote the event “outcome is an even numbered month” and H theevent “outcome is in the first half of the year.” Then P(E) = 1/2 and, becausein the first half of the year there are three even and three odd numbered months,P(E |H) = 1/2 as well; the events are independent.

3.6 b Let S denote the event “outcome is a summer month”. Of the three summermonths, June and August are even numbered, so P(E |S) = 2/3 6= 1/2. Therefore,E and S are dependent.

3.7 a The best approach to a problem like this one is to write out the conditionalprobability and then see if we can somehow combine this with P(A) = 1/3 tosolve the puzzle. Note that P(B ∩Ac) = P(B |Ac) P(Ac) and that P(A ∪B) =P(A) + P(B ∩Ac). So

P(A ∪B) =1

3+

1

1− 1

3

=

1

3+

1

6=

1

2.

3.7 b From the conditional probability we find P(Ac ∩Bc) = P(Ac |Bc) P(Bc) =12

(1− P(B)). Recalling DeMorgan’s law we know P(Ac ∩Bc) = P((A ∪B)c) =1−P(A ∪B) = 1/3. Combined this yields an equation for P(B): 1

2(1− P(B)) = 1/3

from which we find P(B) = 1/3.

3.8 a This asks for P(W ). We use the law of total probability, decomposing Ω =F ∪ F c. Note that P(W |F ) = 0.99.

P(W ) = P(W ∩ F ) + P(W ∩ F c) = P(W |F ) P(F ) + P(W |F c) P(F c)

= 0.99 · 0.1 + 0.02 · 0.9 = 0.099 + 0.018 = 0.117.

3.8 b We need to determine P(F |W ), and this can be done using Bayes’ rule. Someof the necessary computations have already been done in a, we can copy P(W ∩ F )and P(W ) and get:

P(F |W ) =P(F ∩W )

P(W )=

0.099

0.117= 0.846.

3.9 Deciphering the symbols we conclude that P(B |A) is to be determined. Fromthe probabilities listed we find P(A ∩B) = P(A) + P(B)−P(A ∪B) = 3/4 + 2/5−4/5 = 7/20, so that P(B |A) = P(A ∩B) /P(A) = (7/20)/(3/4) = 28/60 = 7/15.

3.10 Let K denote the event “the student knows the answer” and C the event“the answer that is given is the correct one.” We are to determine P(K |C). Fromthe information given, we know that P(C |K) = 1 and P(C |Kc) = 1/4, and thatP(K) = 0.6. Therefore:

P(C) = P(C |K) · P(K) + P(C |Kc) · P(Kc) = 1 · 0.6 +1

4· 0.4 = 0.6 + 0.1 = 0.7

and P(K |C) = 0.6/0.7 = 6/7.

Page 6: Probability and Statistics Book Solutions

462 Full solutions from MIPS: DO NOT DISTRIBUTE

3.11 a The probability that a driver that is classified as exceeding the legal limit infact does not exceed the limit.

3.11 b It is given that P(B) = 0.05. We determine the answer via P(Bc |A) =P(Bc ∩A) /P(A). We find P(Bc ∩A) = P(A |Bc) ·P(Bc) = 0.95 (1−p), P(B ∩A) =P(A |B) · P(B) = 0.05 p, and by adding them P(A) = 0.95− 0.9 p. So

P(Bc |A) =0.95 (1− p)

0.95− 0.9 p=

95 (1− p)

95− 90 p= 0.5 when p = 0.95.

3.11 c From b we find

P(B |A) = 1− P(Bc |A) =95− 90 p− 95 (1− p)

95− 90 p=

5p

95− 90 p.

Setting this equal to 0.9 and solving for p yields p = 171/172 ≈ 0.9942.

3.12 We start by deriving some unconditional probabilities from what is given:P(B ∩ C) = P(B |C) · P(C) = 1/6 and P(A ∩B ∩ C) = P(B ∩ C) · P(A |B ∩ C) =1/24. Next, we should realize that B∩C is the union of the disjoint events A∩B∩Cand Ac ∩B ∩ C, so that

P(Ac ∩B ∩ C) = P(B ∩ C)− P(A ∩B ∩ C) =1

6− 1

24=

1

8.

3.13 a There are several ways to see that P(D1) = 5/9. Method 1: the first teamwe draw is irrelevant, for the second team there are always 5 “good” choices out of 9remaining teams. Method 2: there are

102

= 45 possible outcomes when two teams

are drawn; among these, there are 5 · 5 = 25 favorable outcomes (all weak-strongpairings), resulting in P(D1) = 25/45 = 5/9.

3.13 b Given D1, there are 4 strong and 4 weak teams left. Using one of the methodsfrom a on this reduced number of teams, we find P(D2 |D1) = 4/7 and P(D1 ∩D2) =P(D2 |D1) · P(D1) = (4/7) · (5/9) = 20/63.

3.13 c Proceeding in the same fashion, we find P(D3 |D1 ∩ D2) = 3/5 andP(D1 ∩D2 ∩D3) = P(D3 |D1 ∩D2) · P(D1 ∩D2) = (3/5) · (20/63) = 12/63.

3.13 d Subsequently, we find P(D4 |D1∩· · ·∩D3) = 2/3 and P(D5 |D1∩· · ·∩D4) =1. The final result can be written as

P(D1 ∩ · · · ∩D5) =5

9· 4

7· 3

5· 2

3· 1 =

8

63.

The probability of a “dream draw” with n strong and n weak teams is P(D1 ∩ · · · ∩Dn) =2n/2nn

.

3.14 a If you chose the right door, switching will make you lose, so P(W |R) = 0.If you chose the wrong door, switching will make you win for sure: P(W |Rc) = 1.

3.14 b Using P(R) = 1/3, we find:

P(W ) = P(W ∩R) + P(W ∩Rc) = P(W |R) P(R) + P(W |Rc) P(Rc)

= 0 · 1

3+ 1 · 2

3=

2

3.

Page 7: Probability and Statistics Book Solutions

29.1 Full solutions 463

3.15 This is a puzzle and there are many ways to solve it. First note that P(A) =1/2. We condition on A ∪B:

P(B) = P(B |A ∪B) · P(A ∪B)

=2

3P(A) + P(B)− P(A) P(B)

=2

31

2+ P(B)− 1

2P(B)

=1

3+

1

3P(B) .

Solving the resulting equation yields P(B) =13

1− 13

= 12.

3.16 a Using Bayes’ rule we find:

P(D |T ) =P(D ∩ T )

P(T )=

0.98 · 0.01

0.98 · 0.01 + 0.05 · 0.99≈ 0.1653.

3.16 b Denote the event “the second test indicates you have the disease” with theletter S.Method 1 (“unconscious”): from the first test we know that the probability we havethe disease is not 0.01 but 0.1653 and we reason that we shoud redo the calculationfrom a with P(D) = 0.1653:

P(D |S) =P(D ∩ S)

P(S)=

0.98 · 0.1653

0.98 · 0.1653 + 0.05 · 0.8347≈ 0.7951.

This is the correct answer, but a more thorough consideration is warrented.Method 2 (“conscientious”): we are in fact to determine

P(D |S ∩ T ) =P(D ∩ S ∩ T )

P(S ∩ T )

and we should wonder what “independent repetition of the test” exactly means.Clearly, both tests are dependent on whether you have the disease or not (and as aresult, S and T are dependent), but given that you have the disease the outcomesare independent, and the same when you do not have the disease. Formally put:given D, the events S and T are independent; given Dc, the events S and T areindependent. In formulae:

P(S ∩ T |D) = P(S |D) · P(T |D),

P(S ∩ T |Dc) = P(S |Dc) · P(T |Dc).

So P(D ∩ S ∩ T ) = P(S |D) · P(T |D)P(D) = (0.98)2 · 0.01 = 0.009604 andP(Dc ∩ S ∩ T ) = (0.05)2 ·0.99 = 0.002475. Adding them yields P(S ∩ T ) = 0.012079and so P(D |S ∩ T ) = 0.009604/0.012079 ≈ 0.7951. Note that P(S |T ) ≈ 0.2037which is much larger than P(S) ≈ 0.0593 (and for a good test, it should be).

3.17 a I win the game after the next two rallies if I win both: P(W ∩G) = p2.Similarly for losing the game if you win both rallies: P(W c ∩G) = (1 − p)2. SoP(W |G) = p2/(p2 + (1− p)2).

Page 8: Probability and Statistics Book Solutions

464 Full solutions from MIPS: DO NOT DISTRIBUTE

3.17 b Note that D = Gc and so P(D) = 1− p2 − (1− p)2 = 2p(1− p). Using thelaw of total probability we find:

P(W ) = P(W ∩G) + P(W ∩D) = p2 + P(W |D)P(D) = p2 + 2p(1− p)P(W |D).

Substituting P(W |D) = P(W ) (when it become deuce again, the initial situationrepeats itself) and solving yields P(W ) = p2/(p2 + (1− p)2).

3.17 c If the game does not end after the next two rallies, each of us has againthe same probability to win the game as we had initially. This implies, that theprobability of winning the game is split between us in the same way as the probabilityof an “immediate win,” i.e., in the next two rallies. So P(W ) = P(W |G).

3.18 a No: A ∩B = ∅ so P(A |B) = 0 6= P(A) which is positive.

3.18 b Since, by independence, P(A ∩B) = P(A)·P(B) > 0 it must be that A∩B 6=∅.3.18 c If A ⊂ B then P(B |A) = 1. However, P(B) < 1, so P(B) 6= P(B |A): A enB are dependent.

3.18 d Since A ⊂ A ∩ B, P(A ∪ B |A) = 1, so for the required independenceP(A ∪B) = 1 should hold. On the other hand, P(A ∩B) = P(A)+P(B)−P(A)·P(B)and so P(A ∪B) = 1 can be rewritten as (1−P(A)) · (1−P(B)) = 0, which clearlycontradicts the assumptions.

4.1 a In two independent throws of a die there are 36 possible outcomes, eachoccurring with probability 1/36. Since there are 25 ways to have no 6’s, 10 ways tohave one 6, and one way to have two 6’s, we find that pZ(0) = 25/36, pZ(1) = 10/36,and pZ(2) = 1/36. So the probability mass function pZ of Z is given by the followingtable:

z 0 1 2

pZ(z) 2536

1036

136

The distribution function FZ is given by

FZ(a) =

8>>><>>>:0 for a < 02536

for 0 ≤ a < 12536

+ 1036

= 3536

for 1 ≤ a < 22536

+ 1036

+ 136

= 1 for a ≥ 2.

Z is the sum of two independent Ber(1/6) distributed random variables, so Z hasa Bin(2, 1/6) distribution.

4.1 b If we denote the outcome of the two throws by (i, j), where i is the out-come of the first throw and j the outcome of the second, then M = 2, Z = 0 = (2, 1), (1, 2), (2, 2) , S = 5, Z = 1 = ∅, S = 8, Z = 1 = (6, 2), (2, 6) . Fur-thermore, P(M = 2, Z = 0) = 3/36, P(S = 5, Z = 1) = 0, and P(S = 8, Z = 1) =2/36.

4.1 c The events are dependent, because, e.g., P(M = 2, Z = 0) = 336

differs fromP(M = 2) · P(Z = 0) = 3

36· 25

36.

4.2 a Since P(Y = 0) = P(X = 0), P(Y = 1) = P(X = −1)+P(X = 1), P(Y = 4) =

P(X = 2), the following table gives the probability mass function of Y :a 0 1 4

pY (a) 18

38

12

Page 9: Probability and Statistics Book Solutions

29.1 Full solutions 465

4.2 b FX(1) = P(X ≤ 1) = 1/2; FY (1) = 1/2; FX( 34) = 3

8; FY ( 3

4) = 1

8;

FX(π − 3) = FX(0.1415...) = 3/8; FY (π − 3) = 1/8.

4.3 For every random variable X and every real number a one has that X = a =X ≤ a \ X < a, so

P(X = a) = P(X ≤ a)− P(X < a) .

Since F is right-continuous (see p. 49) we have that F (a) = limε↓0 F (a + ε) =limε↓0 P(X ≤ a + ε). Moreover, P(X < a) = limε↓0 P(X ≤ a− ε) = limε↓0 F (a− ε),so we see that P(X = a) > 0 precisely for those a for which F (a) “makes a jump.” Inthis exercise this is for a = 0, 1

2, and a = 3

4, and p(0) = P(X ≤ 0)−P(X < 0) = 1/3,

etc. We find a 0 1/2 3/4

p(a) 1/3 1/6 1/2.

4.4 a By conditioning one finds that each coin has probability 2p−p2 to give heads.The outcomes of the coins are independent, so the total number of heads has abinomial distribution with parameters n and 2p− p2.

4.4 b Since the total number of heads has a binomial distribution with parametersn and 2p− p2, we find for k = 0, 1, . . . , n,

pX(k) =

n

k

!2p− p2kp2 − 2p + 1

n−k,

and pX(k) = 0 otherwise.

4.5 In one throw of a die one cannot exceed 6, so F (1) = P(X ≤ 1) = 0. SinceP(X = 1) = 0, we find that F (2) = P(X = 2), and from Table 4.1 it follows thatF (2) = 21/36. No matter the outcomes, after seven throws we have that the sumcertainly exceeds 6, so F (7) = 1.

4.6 a Let (ω1, ω2, ω3) denote the outcome of the three draws, where ω1 is the out-come of the first draw, ω2 the outcome of the second draw, and ω3 the outcome ofthe third one. Then the sample space Ω is given by

Ω = (ω1, ω2, ω3) : ω1, ω2, ω3 ∈ 1, 2, 3 = (1, 1, 1), (1, 1, 2), . . . , (1, 1, 6), (1, 2, 1), . . . , (3, 3, 3),

and

X(ω1, ω2, ω3) =ω1 + ω2 + ω3

3for (ω1, ω2, ω3) ∈ Ω.

The possible outcomes of X are 1, 43, 5

3, 2, 7

3, 8

3, 3. Furthermore, X = 1 = (1, 1, 1),

so PX = 1

= 1

27, and

X =4

3 = (1, 1, 2), (1, 2, 1), (2, 1, 1), so P

X =

4

3

=

3

27.

Since

X =5

3 = (1, 1, 3), (1, 3, 1), (3, 1, 1), (1, 2, 2), (2, 1, 2), (2, 2, 1),

we find that PX = 5

3

= 6

27, etc. Continuing in this way we find that the probability

mass function pX of X is given by

Page 10: Probability and Statistics Book Solutions

466 Full solutions from MIPS: DO NOT DISTRIBUTE

a 1 43

53

2 73

83

3pX(a) 1

27327

627

727

627

327

127

4.6 b Setting, for i = 1, 2, 3,

Yi =

(1 if Xi < 2

0 if Xi ≥ 2,

and Y = Y1 + Y2 + Y3, then Y is Bin(3, 13). It follows that the probability that

exactly two draws are smaller than 2 is equal to

P(Y = 2) =

3

2

!1

3

2 1− 1

3

=

6

27.

Another way to find this answer is to realize that the event that exactly two drawsare equal to 1 is equal to

(1, 1, 2), (1, 1, 3), (1, 2, 1), (1, 3, 1), (2, 1, 1), (3, 1, 1).4.7 a Setting Xi = 1 if the ith lamp is defective, and Xi = 0 if not, for i =1, 2, . . . , 1000, we see that Xi is a Ber(0.001) distributed random variable. Assumingthat these Xi are independent, we find that X (as the sum of 1000 independentBer(0.001) random variables) has a Bin(1000, 0.001) distribution.

4.7 b Since X has a Bin(1000, 0.001) distribution, these probabilities are P(X = 0) =

( 9991000

)1000 = 0.367695, P(X = 1) = 999999

10001000 = 0.36806, and P(X > 2) = 1 −P(X ≤ 2) = 0.08021.

4.8 a Assuming that O-rings fail independently of one-another, X can be seen asthe sum of six independent random variables with a Ber(0.8178) distribution. Con-sequently, X has a Bin(6, 0.8178) distribution.

4.8 b Since X has a Bin(6, 0.8178) distribution, we find that P(X ≥ 1) = 1 −P(X = 0) = 1− (1− 0.8178)6 = 0.9999634.

4.9 a With these assumptions, X has a Bin(6, 0.0137) distribution, so we findthat the probability that during a launch at least one O-ring fails is equal toP(X ≥ 1) = 1 − P(X = 0) = 1 − (1 − 0.0137)6 = 0.079436. But then the proba-bility, that at least one O-ring fails for the first time during the 24th launch is equaltoP(X = 0)

23P(X ≥ 1) = 0.01184.

4.9 b The probability that no O-ring fails during a launch is P(X = 0) = (1 −0.0137)6 = 0.92056, so no O-ring fails during 24 launches is equal to

P(X = 0)

24=

0.13718.

4.10 a Each Ri has a Bernoulli distribution, because it can only attain the values 0and 1. The parameter is p = P(Ri = 1). It is not easy to determine P(Ri = 1), butit is fairly easy to determine P(Ri = 0). The event Ri = 0 occurs when none ofthe m people has chosen the ith floor. Since they make their choices independentlyof each other, and each floor is selected by each of these m people with probability1/21, it follows that

P(Ri = 0) =

20

21

m

.

Now use that p = P(Ri = 1) = 1− P(Ri = 0) to find the desired answer.

Page 11: Probability and Statistics Book Solutions

29.1 Full solutions 467

4.10 b If R1 = 0, . . . , R20 = 0, we must have that R21 = 1, so we cannotconclude that the events R1 = a1, . . . , R21 = a21, where ai is 0 or 1, are indepen-dent. Consequently, we cannot use the argument from Section 4.3 to conclude thatSm is Bin(21, p). In fact, Sm is not Bin(21, p) distributed, as the following shows.The elevator will stop at least once, so P(Sm = 0) = 0. However, if Sm would havea Bin(21, p) distribution, then P(Sm = 0) = (1− p)21 > 0, which is a contradiction.

4.10 c This exercise is a variation on finding the probability of no coincident birth-days from Section 3.2. For m = 2, S2 = 1 occurs precisely if the two persons enteringthe elevator select the same floor. The first person selects any of the 21 floors, thesecond selects the same floor with probability 1/21, so P(S2 = 1) = 1/21. For m = 3,S3 = 1 occurs if the second and third persons entering the elevator both select thesame floor as was selected by the first person, so P(S3 = 1) = (1/21)2 = 1/441.Furthermore, S3 = 3 occurs precisely when all three persons choose a different floor.Since there are 21 · 20 · 19 ways to do this out of a total of 213 possible ways, wefind that P(S3 = 3) = 380/441. Since S3 can only attain the values 1, 2, 3, it followsthat P(S3 = 2) = 1− P(S3 = 1)− P(S3 = 3) = 60/441.

4.11 Since the lotteries are different, the event to win something (or not) in onelottery is independent of the other. So the probability to win a prize the first timeyou play is p1p2 + p1(1− p2) + (1− p1)p2. Clearly M has a geometric distribution,with parameter p = p1p2 + p1(1− p2) + (1− p1)p2.

4.12 The “probability that your friend wins” is equal to

p + p(1− p)2 + p(1− p)4 + · · · = p · 1

1− (1− p)2=

1

2− p.

The procedure is favorable to your friend if 1/(2−p) > 1/2, and this is true if p > 0.

4.13 a Since we wait for the first time we draw the marked bolt in independentdraws, each with a Ber(p) distribution, where p is the probability to draw the bolt(so p = 1/N), we find, using a reasoning as in Section 4.4, that X has a Geo(1/N)distribution.

4.13 b Clearly, P(Y = 1) = 1/N . Let Di be the event that the marked bolt wasdrawn (for the first time) in the ith draw. For k = 2, . . . , N we have that

P(Y = k) = P(Dc1 ∩ · · · ∩Dc

k−1 ∩Dk)

= P(Dk |Dc1 ∩ · · · ∩Dc

k−1) · P(Dc1 ∩ · · · ∩Dc

k−1) .

Now P(Dk |Dc1 ∩ · · · ∩Dc

k−1) = 1N−k+1

,

P(Dc1 ∩ · · · ∩Dc

k−1) = P(Dck−1 |Dc

1 ∩ · · · ∩Dck−2) · P(Dc

1 ∩ · · · ∩Dck−2) ,

and

P(Dck−1 |Dc

1 ∩ · · · ∩Dck−1) = 1− P(Dk−1 |Dc

1 ∩ · · · ∩Dck−1) = 1− 1

N − k + 2.

Continuing in this way, we find after k steps that

P(Y = k) =1

N − k + 1· N − k + 1

N − k + 2· N − k + 2

N − k + 3· · · N − 2

N − 1· N − 1

N=

1

N.

See also Section 9.3, where the distribution of Y is derived in a different way.

Page 12: Probability and Statistics Book Solutions

468 Full solutions from MIPS: DO NOT DISTRIBUTE

4.13 c For k = 0, 1, . . . , r, the probability P(Z = k) is equal to the number of waysthe event Z = k can occur, divided by the number of ways

Nr

we can select r

objects from N objects, see also Section 4.3. Since one can select k marked boltsfrom m marked ones in

mk

ways, and r−k nonmarked bolts from N−m nonmarked

ones in

N−mr−k

ways, it follows that

P(Z = k) =

mk

N−mr−k

Nr

, for k = 0, 1, 2, . . . , r.

4.14 a Denoting ‘heads’ by H and ‘tails’ by T , the event X = 2 occurs if we havethrown HH, i.e, if the outcome of both the first and the second throw was ‘heads’.Since the probability of ‘heads’ is p, we find that P(X = 2) = p·p = p2. Furthermore,X = 3 occurs if we either have thrown HTH, or THH, and since the probabilityof throwing ‘tails’ is 1− p, we find that P(X = 3) = p · (1− p) · p + (1− p) · p · p =2p2(1−p). Similarly, X = 4 can only occur if we throw TTHH, THTH, or HTTH,so P(X = 4) = 3p2(1− p)2.

4.14 b If X = n, then the nth throw was heads (denoted by H), and all but one ofthe previous n− 1 throws were tails (denoted by T ). So the possible outcomes are

HTT · · ·T| z n−2 times

H, THTT · · ·T| z n−3 times

H, TTHTT · · ·T| z n−4 times

H, . . . , TT · · ·T| z n−2 times

HH

Notice there are exactly

n−11

= n− 1 of such possible outcomes, each with proba-

bility p2(1− p)n−2, so P(X = n) = (n− 1)p2(1− p)n−2, for n ≥ 2.

5.1 a

Sketch of probability density f : ...................................................................................................................................................................................................................0 1 2 3

.....................................................................................................................................................................................................

....................................................................................................

5.1 b Since f(x) = 0 for x < 0, F (b) = 0 for b < 0. For 0 ≤ b ≤ 1 we compute

F (b) =R b

−∞ f(x) dx =R b

03/4 dx = 3b/4. Since f(x) = 0 for 1 ≤ x ≤ 2, F (b) =

F (1) = 3/4 for 1 ≤ b ≤ 2. For 2 ≤ b ≤ 3 we compute F (b) =R b

−∞ f(x) dx =R 1

0f(x) dx +

R b

2f(x) dx = F (1) +

R b

21/4 dx = 3/4 + (1/4)(b− 2) = b/4 + 1/4. Since

f(x) = 0 for x > 3, F (b) = F (3) = 1 for b > 3.

Sketch of distribution function F : ...................................................................................................................................................................................................................0 1 2 3

....................................................................................................................................................

.............................................................................

5.2 The event 1/2 < X ≤ 3/4 can be written as

1/2 < X ≤ 3/4 = X ≤ 3/4 ∩ X ≤ 1/2c = X ≤ 3/4 \ X ≤ 1/2.

Noting that X ≤ 1/2 ⊂ X ≤ 3/4 we find

P

1

2< X ≤ 3

4

= P

X ≤ 3

4

− P

X ≤ 1

2

=3

4

2

−1

2

2

=5

16.

(cf. Exercise 2.5.)

Page 13: Probability and Statistics Book Solutions

29.1 Full solutions 469

5.3 a From P(X ≤ 3/4) = P(X ≤ 1/4) + P(1/4 < X ≤ 3/4) we obtain

P

1

4< X ≤ 3

4

= P

X ≤ 3

4

− P

X ≤ 1

4

= F

3

4

− F

1

4

=

11

16.

5.3 b The probability density function f(x) of X is obtained by differentiating thedistribution function F (x): f(x) = d/dx

2x2 − x4

= 4x − 4x3 for 0 ≤ x ≤ 1 (and

f(x) = 0 elsewhere).

5.4 a Let T be the the time until the next arrival of a bus. Then T has U(4, 6)distribution. Hence P(X ≤ 4.5) = P(T ≤ 4.5) =

R 4.5

41/2 dx = 1/4.

5.4 b Since Jensen leaves when the next bus arrives after more than 5 minutes,P(X = 5) = P(T > 5) =

R 6

512

dx = 1/2.

5.4 c Since P(X = 5) = 0.5 > 0, X cannot be continuous. Since X can take any ofthe uncountable values in [4, 5], it can also not be discrete.

5.5 a A probability density f has to satisfy (I) f(x) ≥ 0 for all x, and (II)R∞−∞ f(x) dx = 1. Start with property (II):Z ∞

−∞f(x)dx =

Z −2

−3

(cx+3) dx+

Z 3

2

(3−cx) dx =h c

2x2 + 3x

i−2

−3+h3x− c

2x2i32= 6−5c.

So (II) leads to the conclusion that c = 1. Substituting this yields f(x) = x + 3 for−3 ≤ x ≤ −2, and f(x) = 3 − x for 2 ≤ x ≤ 3 (and f(x) = 0 elsewhere), so f alsosatisfies (I).

5.5 b Since f(x) = 0 for x < −3, F (b) = 0 for b ≤ −3. For −3 ≤ b ≤ −2 we

compute F (b) =R b

−∞ f(x) dx =R b

−3(x + 3) dx = (b + 3)2/2. Similarly one finds

F (b) = 1 − (3 − b)2/2 for 2 ≤ b ≤ 3. For −2 ≤ x ≤ 2 f(x) = 0, and henceF (b) = F (−2) = 1/2 for −2 ≤ b ≤ 2. Finally F (b) = 1 for b ≥ 3.

5.6 If X has an Exp(0.2) distribution, then its distribution function is given byF (b) = 1− e−λb. Hence P(X > 5) = 1−P(X ≤ 5) = 1−F (5) = e−5λ. Here λ = 0.2,so P(X > 5) = e−1 = 0.367879 . . . .

5.7 a P(failure) = P(S < 0.55) =R 0.55

−∞ f(x) dx =R 0.55

04x dx +

R 0.55

0.5(4 − 4x) dx =

0.595.

5.7 b The 50-th percentile or median q0.5 is obtained by solving F (q0.5) = 0.5. For

b ≤ 0.5 we obtain F (b) =R b

04x dx = 2b2. Here F (q0.5) = q0.5 yields 2q2

0.5 = 0.5, orq0.5 = 0.5.

5.8 a The probability density g(y) = 1/(2√

ry) has an asymptote in 0 and decreasesto 1/2r in the point r. Outside [0, r] the function is 0.

5.8 b The second darter is better: for each 0 < b < r one has (b/r)2 <p

b/r so thesecond darter always has a larger probability to get closer to the center.

5.8 c Any function F that is 0 left from 0, increasing on [0, r], takes the value 0.9in r/10, and takes the value 1 in r and to the right of r is a correct answer to thisquestion.

5.9 a The area of a triangle is 1/2 times base times height. So the largest area thatcan occur is 1/2 (when the y-coordinate of a point is 1).The event A ≤ 1/4 occurs if and only if the y-coordinate of the random point isat most 1/2, so the answer is (x, y) :2≤x≤3, 1≤y≤3/2.

Page 14: Probability and Statistics Book Solutions

470 Full solutions from MIPS: DO NOT DISTRIBUTE

5.9 b Generalising a we see that A ≤ a if and only if the y-coordinate of the randompoint is at most a. Hence f(a) = P(A ≤ a) = a for 0 ≤ a ≤ 1. Furthermore F (a) = 0for a < 0, and F (a) = 1 for a ≥ 1.

5.9 c Differentiating F yields f(x) = 2 for 0 ≤ x ≤ 1/2; f(x) = 0 elsewhere.

5.10 The residence time T has an Exp(0.5) distribution, so its distribution functionis F (x) = 1− e−x/2. We are looking for P(T ≤ 2) = F (2) = 1− e−1 = 63.2%.

5.11 We have to solve 1 − e−λx = 1/2, or e−λx = 1/2, which means that −λx =ln(1/2) or x = ln(2)/λ.

5.12 We have to solve 1− x−α = 1/2, or x−α = 1/2, which means that xα = 2 orx = 21/α.

5.13 a This follows with a change of variable transformation x 7→ −x in the integral:Φ(−a) =

R −a

−∞ φ(x) dx =R∞

aφ(−x) dx =

R∞a

φ(x) dx = 1− Φ(a).

5.13 b This is straightforward: P(Z ≤ −2) = Φ(−2) = 1− Φ(2) = 0.0228.

5.14 The 10-th percentile q0.1 is given by Φ(q0.1) = 0.1. To solve this we have touse Table ??. Since the table only contains tail probabilities larger than 1/2, wehave to use the symmetry of the normal distribution (φ(−x) = φ(x)): Φ(a) = 0.1 ifand only if Φ(−a) = 0.9 if and only if 1−Φ(a) = 0.9. This gives −q0.1 = 1.28, henceq0.1 = −1.28.

6.1 a If 0 ≤ U ≤ 16, put X = 1, if 1

6< U ≤ 2

6, put X = 2, etc., i.e., if (i − 1)/6 ≤

U < i/6, then set X = i.

6.1 b The number 6U + 1 is in the interval [1, 7], so Y is one of the numbers 1, 2,3, . . . , 7. For k = 1, 2, . . . , 6 we see that P(Y = k) = P(i ≤ 6U + 1 < i + 1) = 1

6;

P(Y = 7) = P(6U + 1 = 7) = P(U = 1) = 0.

6.2 a Substitute u = 0.3782739 in x = 1 + 2√

u: x = 1 + 2√

0.3782739 = 2.2300795.

6.2 b The map u 7→ 1 + 2√

u is increasing, so u = 0.3 will result in a smaller value,since 0.3 < 0.3782739.

6.2 c Any number u smaller than 0.3782739 will result in a smaller realization thanthe one found in a. This happens with probability P(U < 0.3782739) = 0.3782739.

6.3 The random variable Z attains values in the interval [0, 1], so to show that itsdistribution is the U(0, 1), it suffices to show that FZ(a) = a, for 0 ≤ a ≤ 1. Wefind:

FZ(a) = P(Z ≤ a) = P(1− U ≤ a) = P(U ≥ 1− a) = 1−FU (1−a) = 1−(1−a) = a.

6.4 Since 0 ≤ U ≤ 1, 1 ≤ X ≤ 3 follows, so FX(a) = 0 for a < 1 and FX(a) = 1 fora > 3. If we show that FX(a) = F (a) for 1 ≤ a ≤ 3, then we have shown FX = F :

FX(a) = P(X ≤ a) = P1 + 2

√U ≤ a

= P

√U ≤ (a− 1)/2

= P

U ≤ ((a− 1)/2)2

= FU

1

4(a− 1)2

=

1

4(a− 1)2 = F (a).

Page 15: Probability and Statistics Book Solutions

29.1 Full solutions 471

6.5 We see that

X ≤ a ⇔ − ln U ≤ a ⇔ ln U ≥ −a ⇔ U ≥ e−a,

and so P(X ≤ a) = PU ≥ e−a

= 1 − P

U ≤ e−a

= 1 − e−a, where we use

P(U ≤ p) = p for 0 ≤ p ≤ 1 applied to p = e−a (remember that a ≥ 0).

6.6 We know that X = − 12

ln U has an Exp(2) distribution when U is U(0, 1).Inverting this relationship, we find U = e−2X , which should be the way to get Ufrom X. We check this, for 0 < a ≤ 1:

P(U ≤ a) = Pe−2X ≤ a

= P(−2X ≤ ln a)

= P

X ≥ −1

2ln a

= e−2(− 1

2 ln a)

= a = FU (a).

6.7 We need to obtain F inv, and do this by solving F (x) = u, for 0 ≤ u ≤ 1:

1− e−5x2= u ⇔ e−5x2

= 1− u ⇔ −5x2 = ln(1− u)

⇔ x2 = −0.2 ln(1− u) ⇔ x =p−0.2 ln(1− u).

The solution is Z =√−0.2 ln U (replacing 1−U by U , see Exercise 6.3). Note that

Z2 has an Exp(5) distribution.

6.8 We need to solve F (x) = u for 0 ≤ u ≤ 1 to obtain F inv(x):

1− x−3 = u ⇔ 1− u = x−3 ⇔ x = (1− u)−13 .

So, X = (1− U)−13 has a Par(3) distribution, and the same holds for X = U−

13 =

1/ 3√

U .

6.9 a For six of the eight possible outcomes our algorithm terminates succesfully,so the probability of success is 6/8 = 3/4.

6.9 b If the first toss (of the three coins) is unsuccessful, we try again, and repeatuntil the first success. This way, we generate a sequence of Ber(3/4) trials and stopafter the first success. From Section 4.4 we know that the number N of trials neededhas a Geo(3/4) distribution.

6.10 a Define random variables Bi = 1 if Ui ≤ p and Bi = 0 if Ui > p. ThenP(Bi = 1) = p and P(Bi = 0) = 1 − p: each Bi has a Ber(p) distribution. If B1 =B2 = · · · = Bk−1 = 0 and Bk = 1, then N = k, i.e., N is the position in thesequence of Bernoulli random variables, where the first 1 occurs. This is a Geo(p)distribution. This can be verified by computing the probability mass function: fork ≥ 1,

P(N = k) = P(B1 = B2 = · · · = Bk−1 = 0, Bk = 1)

= P(B1 = 0) P(B2 = 0) · · · P(Bk−1 = 0) P(Bk = 1)

= (1− p)k−1 p.

Page 16: Probability and Statistics Book Solutions

472 Full solutions from MIPS: DO NOT DISTRIBUTE

6.10 b If Y is (a real number!) greater than n, then rounding upwards means weobtain n + 1 or higher, so Y > n = Z ≥ n + 1 = Z > n. Therefore,P(Z > n) = P(Y > n) = e−λn =

e−λ

n. From λ = − ln(1− p) we see: e−λ = 1− p,

so the last probability is (1 − p)n. From P(Z > n− 1) = P(Z = n) + P(Z > n) wefind: P(Z = n) = P(Z > n− 1)−P(Z > n) = (1− p)n−1 − (1− p)n = (1− p)n−1 p.Z has a Geo(p) distribution.

6.11 a There are many possibilities, one is: Y1 = g + 0.5 + Z1, where g and Z1 areas defined earlier. In this case, the bribed jury member adds a bonus of 0.5 points.Of course, this is assuming that she was bribed to support the candidate. If she wasbribed against her, Y1 = g − 0.5 + Z1 would be more appropriate.

6.11 b We call the resulting deviation R and it is the average of Z1, Z2, . . . , Z7 afterwe have removed two of them, chosen randomly. This can be done as follows. Let Iand J be independent random variables, such that P(I = n) = 1/7 for n = 1, 2, . . . , 7and P(J = m) = 1/6 for m = 1, 2, . . . , 6. Put K = J if J < I; otherwise, putK = J + 1. Now, the pair I, K is a random pair chosen from the set of indices1, 2, . . . , 7. This can be verified, for example, the probability that 1 and 2 areselected equals P(I = 1, J = 1) + P(I = 2, J = 1) = 1/21, which is correct, becausethere are

72

= 21 pairs to choose from, each we the same probability.

Removing ZI and ZK from the jury list, we can compute R as the average ofthe remaining ones. We expect this rule to be more sensitive to bribery, becausethe bribed jury member is more likely to have assigned one of the extreme scores(highest or lowest). With either of the two other rules, this score has no influenceat all, because it is not taken into account.

6.12 We need to generate stock prices for the next five years, or 60 months. So weneed sixty U(0, 1) random variables U1, . . ., U60. Let Si denote the stock price inmonth i, and set S0 = 100, the initial stock price. From the Ui we obtain the stockmovement, as follows, for i = 1, 2, . . .:

Si =

8><>:0.95 Si−1 if Ui < 0.25,

Si−1 if 0.25 ≤ Ui ≤ 0.75,

1.05 Si−1 if Ui > 0.75.

We have carried this out, using the realizations below:

1–10: 0.72 0.03 0.01 0.81 0.97 0.31 0.76 0.70 0.71 0.2511–20: 0.88 0.25 0.89 0.95 0.82 0.52 0.37 0.40 0.82 0.0421–30: 0.38 0.88 0.81 0.09 0.36 0.93 0.00 0.14 0.74 0.4831–40: 0.34 0.34 0.37 0.30 0.74 0.03 0.16 0.92 0.25 0.2041–50: 0.37 0.24 0.09 0.69 0.91 0.04 0.81 0.95 0.29 0.4751–60: 0.19 0.76 0.98 0.31 0.70 0.36 0.56 0.22 0.78 0.41

We do not list all the stock prices, just the ones that matter for our investmentstrategy (you can verify this). We first wait until the price drops below ¿ 95, whichhappens at S4 = 94.76. Our money has been in the bank for four months, so we own¿ 1000 · 1.0054 = ¿ 1020.15, for which we can buy 1020.15/94.76 = 10.77 shares.Next we wait until the price hits ¿ 110, this happens at S15 = 114.61. We sell theour shares for ¿ 10.77 · 114.61 = ¿ 1233.85, and put the money in the bank. AtS42 = 92.19 we buy stock again, for the ¿ 1233.85 · 1.00527 = ¿ 1411.71 that hasaccrued in the bank. We can buy 15.31 shares. For the rest of the five year period

Page 17: Probability and Statistics Book Solutions

29.1 Full solutions 473

nothing happens, the final price is S60 = 100.63, which puts the value of our portfolioat ¿ 1540.65.For a real simulation the above should be repeated, say, one thousand times. Theone thousand net results then give us an impression of the probability distributionthat corresponds to this model and strategy.

6.13 Set p = P(H). Toss the coin twice. If HT occurs, I win; if TH occurs, youwin; if TT or HH occurs, we repeat. Since P(HT ) = P(TH) = p (1 − p), we havethe same probability of winning. The probability that the game ends is 2 p (1 − p)for each double toss. So, if 2N is the total number of tosses needed, where N has aGeo(2 p (1− p)) distribution.

7.1 a Outcomes: 1, 2, 3, 4, 5, and 6. Each has probability 1/6.

7.1 b E[T ] = 16· 1 + 1

6· 2 + · · · 1

6· 6 = 1

6(1 + 2 = · · ·+ 6) = 7

2.

For the variance, first compute ET 2:

ET 2 =

1

6· 1 +

1

6· 4 + · · · 1

6· 36 =

91

6.

Then: Var(T ) = ET 2− (E[T ])2 = 91

6− 49

4= 35

12.

7.2 a Here E[X] = (−1)P(X = −1)+0 ·P(X = 0)+1 ·P(X = 1) =− 15+0+ 2

5= 1

5.

7.2 b The discrete random variable Y can take the values 0 and 1. The probabil-ities are P(Y = 0) = P

X2 = 0

= P(X = 0) = 2

5, and P(Y = 1) = P

X2 = 1

=

P(X = −1) + P(X = 1) = 15

+ 25

= 35. Moreover, E[Y ] = 0 · 2

5+ 1 · 3

5= 3

5

7.2 c According to the change of variable formula: EX2

= (−1)2 · 15+02 · 2

5+12 · 2

5=

35.

7.2 d With the alternative expression for the variance: Var(X) = EX2−(E[X])2 =

35− ( 1

5)2 = 14

25.

7.3 Since Var(X) = EX2− (E[X])2,we find E

X2

= Var(X) + (E[X])2 = 7.

7.4 By the change of units rule: E[3− 2X] = 3− 2E[X] = −1, and Var(3− 2X) =4Var(X) = 16.

7.5 If X has a Ber(p) distribution, then E[X] = 0 · (1− p) + 1 · p = p and E[X]2 =0 · (1− p) + 1.p = p. So Var(X) = E[X]2 − (E[X])2 = p− p2 = p(1− p).

7.6 Since f is increasing on the interval [2, 3] we know from the interpretation ofexpectation as center of gravity that the expectation should lie closer to 3 than to 2.The computation: E[Z] =

R 3

2319

z3 dz =

376

z432

= 2 4376

.

7.7 We use the rule for expectation and variance under change of units. First,E[X] =

R 1

0x · (4x− 4x3) dx =

R 1

04x2 − 4x4 dx =

(4/3x3 − 4/5x5)

10

= 4/3− 4/5 =8/15. Then, changing units, E[2X + 3] = 2 ·8/15+3 = 61/15. For the variance, firstcompute E

X2

=R 1

0x2 · (4x − 4x3) dx = 1/3. Hence Var(X) = 1/3 − (8/15)2 =

11/225, and changing units, Var(2X + 3) = 44/225.

7.8 Let f be the probability density of X Then f(x) = ddx

F (x) = 2 − 2x for

0 ≤ x ≤ 1, and f(x) = 0 elsewhere. So E[X] =R +∞−∞ xf(x) dx =

R 1

0(2x− 2x2) dx =

[x2 − 23x3]10 = 1

3.

7.9 a If X has U(α, β) distribution, then X has probability density function f(x) =1/(β − α) for α ≤ x ≤ β, and 0 elsewhere. So

E[X] =

Z β

α

x/(β − α) dx = 1/(β − α) ·1

2x2

β

α

=1

2

(β2 − α2)

β − α=

1

2(α + β).

Page 18: Probability and Statistics Book Solutions

474 Full solutions from MIPS: DO NOT DISTRIBUTE

7.9 b First compute EX2: E[X] =

R β

αx2

(β−α)dx = 1

3(β3−α3)

β−α= 1

3(β2 + αβ + α2).

Then Var(X) = EX2 − (E[X])2 = 1

3(β2 + αβ + α2) − 1

4(α + β)2 = 1

12(β − α)2.

A quicker way to do this is to note that X has the same variance as a U(0, β − α)distribution U . Hence Var(X) = Var(U) = 1

12(β − α)2.

7.10 a If X has an Exp(X) distribution, then X has probability density f(X) =λe−λx for x ≥ 0, and f(x) = 0 elsewhere. So, using the partial integration formula,E[X] =

R∞0

xλe−λx dx =−xe−λx

∞0− R∞

0−eλx dx =

− 1λe−λx

∞0

= 1λ.

7.10 b First compute EX2, using partial integration, and using the result from

part a:

EX2 =

Z ∞

0

x2λe−λx dx =h−x2e−λx

i∞0−Z ∞

0

−2xe−λx dx =2

λ

Z ∞

0

xλe−λx dx =2

λ2.

Then Var(X) = E[X]2 − (E[X])2 = 2λ2 − ( 1

λ)2 = 1

λ2 .

7.11 a If X has a Par(α) distribution, then X has p.d f(x) = αx−α−1 for x ≥ 1, andf(x) = 0 elsewhere. So the Par(2) distribution has probability density f(x) = 2x−3,and then E[X] =

R∞1

x · 2x−3 dx = 2R∞1

x−2 dx =−2x−1

∞1

= 2.

7.11 b Now X has probability density function f(x) = 12x−3/2. So now E[X] =R∞

112x−1/2 dx =

√x∞1

= ∞.

7.11 c In the general case E[X] =R∞1

x αxα+1 dx = α

R∞1

x−α dx = αh

x−α+1

−α+1

i∞1

=

αα−1

, provided x−α+1 → 0 as x →∞, which happens for all α > 1.

7.12 From 7.11c we already know that E[X] = α/(α − 1) if and only of α > 1.

Now we compute EX2

: EX2

= αR∞1

x−α+1 dx = [α · x−α+2

−α+2]∞1 = α

α−2provided

α > 2 (otherwise EX2

= ∞). It follow that Var(X) exists only if α > 2 , and then

Var(X) = E[X]2 − (E[X])2 = α(α−2)

− α2

(α−1)2= α[α2−2α+1−α2+2α]

(α−1)2(α−2)= α

(α−1)2(α−2).

7.13 Since Var(X) = Eh

X − E[X]2i ≥ 0, but also Var(X) = E

X2− (E[X]2)

we must have that EX2 ≥ (E[X])2.

7.14 The area of a triangle is half the length of the base time the height. HenceA = 1

2Y , where Y is U(0, 1) distributed. It follows that E[A] = 1

2E[Y ] = 1

4.

7.15 a We use the change-of-units rule for the expectation twice:

Var(rX) = E(rX − E[rX])2

= E

(rX − rE[X])2

= E

r2(X − E[X])2

= r2E

(X − E[X])2

= r2Var(X) .

7.15 b Now we use the change-of-units rule for the expectation once:

Var(X + s) = E((X + s)− E[X + s])2

= E

((X + s)− E[X] + s)2

= E

(X − E[X])2

= Var(X) .

7.15 c With first b, and then a: Var(rX + s) = Var(rX) = r2Var(X) .

7.16 We have to use partial integration: E[X] =R 1

0−4x2 ln x dx =

− 43x3 ln x

10+R 1

043x3 · 1

xdx = 4

3

R 1

0x2 dx = 4

9.

Page 19: Probability and Statistics Book Solutions

29.1 Full solutions 475

7.17 a Since ai ≥ 0 and pi ≥ 0 it must follow that a1p1 + · · · + arpr ≥ 0. So0 = E[U ] = a1p1 + · · ·+ arpr ≥ 0. As we may assume that all pi > 0, it follows thata1 = a2 = · · · = ar = 0.

7.17 b Let m = E[V ] = p1b1+· · ·+prbr. Then the random variable U = (V −E[V ])2

takes the values a1 = (b1−m)2, . . . , ar = (br −m)2. Since E[U ] = Var(V ) = 0, parta tells us that 0 = a1 = (b1 − m)2, . . . , 0 = ar = (br − m)2. But this is onlypossible if b1 = m, . . . , br = m. Since m = E[V ], this is the same as saying thatP(V = E[V ]) = 1.

8.1 The random variable Y can take the values |80 − 100| = 20, |90 − 100| =10, |100−100| = 0, |110−100| = 10 and |120−100| = 20. We see that the values are0, 10 and 20, and the latter two occur in two ways: P(Y = 0) = P(X = 100) = 0.2,but P(Y = 10) = P(X = 110) + P(X = 90) = 0.4; P(Y = 20) = P(X = 120) +P(X = 80) = 0.4.

8.2 a First we determine the possible values that Y can take. Here these are −1, 0,and 1. Then we investigate which x-values lead to these y-values and sum the prob-abilities of the x-values to obtain the probability of the y-value. For instance,

P(Y = 0) = P(X = 2) + P(X = 4) + P(X = 6) =1

6+

1

6+

1

6=

1

2.

Similarly, we obtain for the two other values

P(Y = −1) = P(X = 3) =1

6, P(Y = 1) = P(X = 1) + P(X = 5) =

1

3.

8.2 b The values taken by Z are −1, 0, and 1. Furthermore

P(Z = 0) = P(X = 1) + P(X = 3) + P(X = 5) =1

6+

1

6+

1

6=

1

2,

and similarly P(Z = −1) = 1/3 and P(Z = 1) = 1/6.

8.2 c Since for any α one has sin2(α) + cos2(α) = 1, W can only take the value 1,so P(W = 1) = 1.

8.3 a Let F be the distribution function of U , and G the distribution function ofV . Then we know that F (x) = 0 for x < 1, F (x) = x for 0 ≤ x ≤ 1, and F (x) = 1for x > 1. Thus

G(y) = P(V ≤ y) = P(2U + 7 ≤ y) = P

U ≤ y − 7

2

=

y − 7

2,

provided 0 ≤ (y − 7)/2 ≤ 1 which happens if and only if 0 ≤ y − 7 ≤ 2 if and only7 ≤ y ≤ 9. Furthermore, G(y) = 0 if y < 7 and G(y) = 1 if y > 9. We recognize Gas the distribution function of a U(7, 9) random variable.

8.3 b Let F and G be as before. Now

G(y) = P(V ≤ y) = P(rU + s ≤ y) = PU ≤ y − s

r

=

y − s

r,

provided 0 ≤ (y − s)/r ≤ 1 , which happens if and only if 0 ≤ y − s ≤ r (note thatwe use that r > 0), if and only if s ≤ y ≤ s + r. We see that G has a U(s, s + r)distribution.

Page 20: Probability and Statistics Book Solutions

476 Full solutions from MIPS: DO NOT DISTRIBUTE

8.4 a Let F be the distribution function of X, and G that of Y . Then we know thatF (x) = 1 − e−x/2 for x ≥ 0, and we find that G(y) = P(Y ≤ y) = P

12X ≤ y

=

P(X ≤ 2y) = 1 − e−y. We recognize G as the distribution function of an Exp(1)distribution.

8.4 b Let F be the distribution function of X, and G that of Y . Then we knowthat F (x) = 1− e−λx for x ≥ 0, and we find that G(y) = P(Y ≤ y) = P(λX ≤ y) =P(X ≤ y/λ) = 1 − e−y. We recognize G as the distribution function of an Exp(1)distribution.

8.5 a For 0 ≤ b ≤ 2 we have that FX(b) =R b

034x(2 − x) dx =

34x2 − 1

4x3b0

=34b2 − 1

4b3. Furthermore FX(b) = 0 for b < 0, and FX(b) = 1 for b > 2.

8.5 b For 0 ≤ y ≤ √2 we have FY (y) = P(Y ≤ y) = P

√X ≤ y

= P

X ≤ y2

=

34y4 − 1

4y6.

8.5 c We simply differentiate FY : fY (y) = ddy

FY (y) = 3y3− 3y5/2 for 0 ≤ y ≤ √2,

and fY (y) = 0 elsewhere.

8.6 a Compute the distribution function of Y : FY (y) = P(Y ≤ y) = P

1X≤ y

=

PX ≥ 1

y

= 1 − P

X < 1

y

= 1 − FX( 1

y), where you use that P

X < 1

y

=

PX ≤ 1

y

since X has a continuous distribution. Differentiating we obtain: fY (y) =

ddy

FY (y) = ddy

(1− FX( 1y)) = 1

y2 fX( 1y) for y > 0 (fY (y) = 0 for y ≤ 0).

8.6 b Applying part a with Z = 1/Y we obtain fZ(z) = 1z2 fY ( 1

z). Then applying

a again: fZ(z) = 1z2 fY

1z

= 1

z21

(1/z)2fX

1

(1/z)

) = fX(z).

This is of course what should happen: Z = 1/Y = 1/(1/X) = X, so Z and X havethe same probability density function.

8.7 Let X be any random variable that only takes positive values, and let Y =ln(X). Then for y > 0 :

FY (y) = P(Y ≤ y) = P(ln(X) ≤ y) = P(X ≤ ey) = FX(ey).

If X has a Par(α) distribution, then FX(x) = 1 − x−α for x ≥ 1. Hence FY (y) =1 − e−αy for y ≥ 0 is the distribution function of Y . We recognize this as thedistribution function of an Exp(α) distribution.

8.8 Let X be any random variable that only takes positive values, and let W =X1/α/λ. Then for w > 0 :

FW (w) = P(W ≤ w) = PX1/α/λ ≤ w

= P(X ≤ (λw)α) = FX((λw)α).

If X has an Exp(1) distribution, then FX(x) = 1− e−x for x ≥ 0. Hence FW (w) =1− e−(λw)α

for w ≥ 0.

8.9 If Y = −X, then FY (y) = P(Y ≤ y) = P(−X ≤ y) = P(X ≥ −y) =1− FX(−y) for all Y (where you use that X has a continuous distribution). Differ-entiating we obtain fY (y) = fX(−y) for all y.

8.10 Because of symmetry: P(X ≥ 3) = 0.500. Furthermore: σ2 = 4, so σ = 2.Then Z = (X − 3)/2 is an N(0, 1) distributed random variable, so that P(X ≤ 1) =P((X − 3)/2) ≤ (1− 3)/2 = P(Z ≤ −1) = P(Z ≥ 1) = 0.1587.

Page 21: Probability and Statistics Book Solutions

29.1 Full solutions 477

8.11 Since −g is a convex function, Jensen’s inequality yields that −g(E[X]) ≤E[−g(X)]. Since E[−g(X)] = −E[g(X)], the inequality follows by multiplying bothsides by −1.

8.12 a The possible values Y can take are√

0 = 0,√

1 = 1,√

100 = 10, and√10 000 = 100. Hence the probability mass function is given by

y 0 1 10 100

P(Y = y) 14

14

14

14

8.12 b Compute the second derivative: d2

dx2

√x = − 1

4x−3/2 < 0. Hence g(x) = −√x

is a convex function. Jensen’s inequality yields thatp

E[X] ≥ Eh√

Xi.

8.12 c We obtainp

E[X] =p

(0 + 1 + 100 + 10 000)/4 = 50.25, but

Eh√

Xi

= E[Y ] = (0 + 1 + 10 + 100)/4 = 27.75.

8.13 On the interval [π, 2π] the function sin w is a convex function, so by Jensen’sinequality sin(E[W ]) ≤ E[sin(W )]]. Verified by computations : sin(E[W ]) = sin( 3

2π) =

−1 ≤ E[sin(W )] =R 2π

πsin(w)/π dw = [− cos(w)/π]2π

π = −2/π.

8.14 a An example is X with P(X = −1) = P(X = 1) = 12. Then E[X] = 1

2·(−1)+

12· 1 = 0 and also E

X3

= 12· (−1) + 1

2· (1) = 0.

8.14 b The function g(x) = x3 is strictly convex on the interval (0,∞). Hence ifX > 0 (and X is not constant) the inequality will hold.

8.15 a We know that P(Z ≤ z) = [P(X1 ≤ z)]2. So FZ(z) = z2 for 0 ≤ z ≤ 1,and fZ(z) = 2z on this interval. Therefore E[Z] =

R 1

02z2 dz = 2

3. For V we have

FV (v) = 1− (1− v)2 = 2v − v2. Hence fV (v) = 2 − 2v. Therefore E[V ] =R 1

0(2v −

2v2) dv = 1− 2/3 = 1/3.

8.15 b For general n we have FZ(z) = zn for 0 ≤ z ≤ 1, and fZ(z) = nzn−1.Therefore E[Z] =

R 1

0nzn dz = n

n+1. For V we have FV (v) = 1 − (1 − v)n, and

fV (v) = n(1− v)n−1. Therefore E[V ] =R 1

0nv(1− v)n−1 dv =

R 1

0n(1−u)un−1 du =

nR 1

0un−1 du− n

R 1

0un du = n. 1

n− n. 1

n+1= 1

n+1.

8.15 c Since Yi = 1 − Xi are also uniform we get E[Z] = E[maxY1, . . . , Yn] =E[max1−X1, . . . , 1−Xn] = 1− E[minX1, . . . , Xn] = 1− E[V ].

8.16 a Suppose first that a ≤ b Then mina, b = a , and also |a − b| = b − a. Soa+b−|a−b| = a+b−b+a = 2a, and the formula holds. If a > b then mina, b = b,and a + b− |a− b| = a + b− a + b = 2b, and the formula holds also for this case.

8.16 b From part a and linearity of expectations we have

E[minX, Y ] = E[(X + Y − |X − Y |)/2] =1

2E[X] +

1

2E[Y ]− 1

2E[X − Y |]

= E[X]− 1

2E[|X − Y |] ,

since E[X] = E[Y ].

8.16 c From b we obtain, since E[|] X − Y | ≥ 0 that E[minX, Y ] ≤ E[X] . In-terchanging X and Y we also have E[minX, Y ] ≤ E[Y ]. Combining these two wehave inequalities we obtain E[minX, Y ] ≤ minE[X] , E[Y ].

Page 22: Probability and Statistics Book Solutions

478 Full solutions from MIPS: DO NOT DISTRIBUTE

8.17 a For n = 2 : if x1 ≤ x2 then minx1, x2 = x1 and max−x1,−x2 = −x1

since −x1 ≥ −x2; similarly for the case x1 > x2. In general, if xi is the smallest ofx1, . . . , xn then −xi will be the largest of −x1, . . . ,−xn.

8.17 b Using part a we have

FV (a) = P(minX1, . . . , Xn ≤ a) = P(−max−X1, . . . ,−Xn ≤ a)

= P(max−X1, . . . ,−Xn ≥ −a) = 1− P(max−X1, . . . ,−Xn ≤ −a)

= 1− (P(−X1 ≤ −a))n = 1− (1− P(X1 ≤ a))n ,

using Exercise 8.9 in the last step.

8.18 The distribution function of each of the Xi is F (x) = 1 − e−λx. Then thedistribution function of V is 1 − (1 − F (x))n = 1 − e−λnx. This is the distributionfunction of an Exp(nλ) random variable.

8.19 a This happens for all ϕ in the interval [π/4, π/2], which corresponds to theupper right quarter of the circle.

8.19 b Since Z ≤ t = X ≤ arctan(t), we obtain

FZ(t) = P(Z ≤ t) = P(X ≤ arctan(t)) =1

2+

1

πarctan(t).

8.19 c Differentiating FZ we obtain that the probability density function of Z is

fZ(z) =d

dzFZ(z) =

d

dz

1

2+

1

πarctan(z)

=

1

π(1 + z2)for −∞ < z < ∞.

9.1 For a and b from 1 to 4 we have

P(X = a) = P(X = a, Y = 1) + · · ·+ P(X = a, Y = 4) =1

4,

and

P(Y = b) = P(X = 1, Y = b) + · · ·+ P(X = 4, Y = b) =1

4.

9.2 a From P(X = 1, Y = 1) = 1/2, P(X = 1) = 2/3, and the fact that P(X = 1) =P(X = 1, Y = 1) + P(X = 1, Y = −1), it follows that P(X = 1, Y = −1) = 1/6.Since P(Y = 1) = 1/2 and P(X = 1, Y = 1) = 1/2, we must have: P(X = 0, Y = 1)and P(X = 2, Y = 1) are both zero. From this and the fact that P(X = 0) = 1/6 =P(X = 2) one finds that P(X = 0, Y = −1) = 1/6 = P(X = 2, Y = −1).

9.2 b Since, e.g., P(X = 2, Y = 1) = 0 is different from P(X = 2)P(Y = 1) = 16· 1

2,

one finds that X and Y are dependent.

9.3 a P(X = Y ) = P(X = 1, Y = 1) + · · ·+ P(X = 4, Y = 4) = 14.

9.3 b P(X + Y = 5) = P(X = 1, Y = 4) + P(X = 2, Y = 3) + P(X = 3, Y = 2) +P(X = 4, Y = 1) = 1

4.

9.3 c P(1 < X ≤ 3, 1 < Y ≤ 3) = P(X = 2, Y = 2)+P(X = 2, Y = 3)+P(X = 3, Y = 2)+P(X = 3, Y = 3) = 1

4.

9.3 d P((X, Y ) ∈ 1, 4 × 1, 4) = P(X = 1, Y = 1)+P(X = 1, Y = 4)+P(X = 4, Y = 1)+P(X = 4, Y = 4) = 1

4.

Page 23: Probability and Statistics Book Solutions

29.1 Full solutions 479

9.4 Since P(X = i, Y = j) is either 0, or is equal to 1/14 for each i and j from 1to 5, we know that all the entries of the first row and the second column of thetable are equal to 1/14. Since P(X = 1) = 1/14, this then determines the rest ofthe first column (apart of the first entry it contains only zeroes). Similarly, sinceP(Y = 5) = 1/14 one must have that—apart from the second entry—all the entriesin the fifth row are zero. Continuing in this way we find

a

b 1 2 3 4 5 P(Y = b)

1 1/14 1/14 1/14 1/14 1/14 5/142 0 1/14 1/14 1/14 1/14 4/143 0 1/14 1/14 0 0 2/144 0 1/14 1/14 0 0 2/145 0 1/14 0 0 0 1/14

P(X = a) 1/14 5/14 4/14 2/14 2/14 1

9.5 a From the first row it follows that 116≤ η ≤ 1

4. The second and third row do

not add extra information, so we find that 116≤ η ≤ 1

4.

9.5 b For any (allowed) value of η we have that P(X = 1, Y = 4) = 0. Since116

≤ P(X = 1) ≤ 18, and P(Y = 4) = 3

16, we find that P(X = 1, Y = 4) 6=

P(X = 1)P(Y = 4). Hence, there does not exist a value for η for which X andY are independent.

9.6 a U attains the values 0, 1, and 2, while V attains the values 0 and 1. ThenP(U = 0, V = 0) = P(X = 0, Y = 0) = P(X = 0)P(Y = 0) = 1

4, due to the inde-

pendence of X and Y . In a similar way the other joint probabilities of U and V areobtained, yielding the following table:

u

v 0 1 2

0 1/4 0 1/4 1/21 0 1/2 0 1/2

1/4 1/2 1/4 1

9.6 b Since P(U = 0, V = 0) = 146= 1

8= P(U = 0)P(V = 0), we find that U and V

are dependent.

9.7 a The joint probability distribution of X and Y is given by

a

b 1 2 3 P(Y = b)

1 0.22 0.15 0.06 0.432 0.11 0.24 0.22 0.57

P(X = a) 0.33 0.39 0.28 1

Page 24: Probability and Statistics Book Solutions

480 Full solutions from MIPS: DO NOT DISTRIBUTE

9.7 b Since P(X = 1, Y = 1) = 11685383

6= 17415383

22985383

= P(X = 1)P(Y = 1), we find thatX and Y are dependent.

9.8 a Since X can attain the values 0 and 1 and Y the values 0 and 2, Z can attainthe values 0, 1, 2, and 3 with probabilities: P(Z = 0) = P(X = 0, Y = 0) = 1/4,P(Z = 1) = P(X = 1, Y = 0) = 1/4, P(Z = 2) = P(X = 0, Y = 2) = 1/4, andP(Z = 3) = P(X = 1, Y = 2) = 1/4.

9.8 b Since X = Z − Y , X can attain the values −2, −1, 0, 1, 2, and 3 withprobabilities

PX = −2

= P

Z = 0, Y = 2

= 1/8,

PX = −1

= P

Z = 1, Y = 2

= 1/8,

PX = 0

= P

Z = 0, Y = 0

+ P

Z = 2, Y = 2

= 1/4,

PX = 1

= P

Z = 1, Y = 0

+ P

Z = 3, Y = 2

= 1/4,

PX = 2

= P

Z = 2, Y = 0

= 1/8,

PX = 3

= P

Z = 3, Y = 0

= 1/8.

We have the following table:

z −2 −1 0 1 2 3

pX(z) 1/8 1/8 1/4 1/4 1/8 1/8

9.9 a One has that FX(x) = limy→∞ F (x, y). So for x ≤ 0: FX(x) = 0, and forx > 0: FX(x) = F (x,∞) = 1− e−2x. Similarly, FY (y) = 0 for y ≤ 0, and for y > 0:FY (y) = F (∞, y) = 1− e−y.

9.9 b For x > 0 and y > 0: f(x, y) = ∂2

∂x ∂yF (x, y) = ∂

∂x

e−y − e−(2x+y)

=

2e−(2x+y).

9.9 c There are two ways to determine fX(x):

fX(x) =

Z ∞

−∞f(x, y) dy =

Z ∞

0

e−(2x+y)dy = 2e−2x for x > 0

and

fX(x) =d

dxFX(x) = 2e−2x for x > 0.

Using either way one finds that fY (y) = e−y for y > 0.

9.9 d Since F (x, y) = FX(x)FY (y) for all x, y, we find that X and Y are indepen-dent.

9.10 a

P

1

4≤ X ≤ 1

2,1

3≤ Y ≤ 2

3

=

Z 12

14

Z 23

13

12

5xy(1 + y) dx dy

=12

5

Z 12

14

x Z 2

3

13

y(1 + y) dydx

=

Z 12

14

82

135x dx =

41

720.

Page 25: Probability and Statistics Book Solutions

29.1 Full solutions 481

9.10 b Since f(x, y) = 0 for x < 0 or y < 0,

F (a, b) = P(X ≤ a, Y ≤ b) =

Z a

−∞

Z b

−∞f(x, y) dy

dx

=

Z a

0

Z b

0

12

5xy(1 + y) dy

dx =

3

5a2b2 +

2

5a2b3.

9.10 c Since f(x, y) = 0 for y > 1, we find for a between 0 and 1, and b ≥ 1,

F (a, b) = P(X ≤ a, Y ≤ b) = P(X ≤ a, Y ≤ 1) = F (a, 1) = a2.

Hence, applying (9.1) one finds that FX(a) = a2, for a between 0 and 1.

9.10 d Another way to obtain fX is by differentiating FX .

9.10 e f(x, y) = fX(x)fY (y), so X and Y are independent.

9.11 To determine P(X < Y ) we must integrate f(x, y) over the region G of points(x, y) in R2 for which x is smaller than y:

P(X < Y ) =

ZZ(x,y)∈R2; x<y

f(x, y) dx dy

=

Z ∞

−∞

Z y

−∞f(x, y) dx

dy =

Z 1

0

Z y

0

12

5xy(1 + y) dx

dy

=12

5

Z 1

0

y(1 + y)

Z y

0

x dx

dy =

12

10

Z 1

0

y3(1 + y) dy =27

50.

Here we used that f(x, y) = 0 for (x, y) outside the unit square.

9.12 a Since the integral over R2 of the joint probability density function is equalto 1, we find from Z 1

0

Z 2

0

(3u2 + 8uv) du dv = 10

that K = 110

.

9.12 b

P(2X ≤ Y ) =

Z Z(x,y):2x<y

1

10(3x2 + 8xy) dx dy

=

Z 1

x=0

Z 2

y=2x

1

10(3x2 + 8xy) dx dy

= 0.45.

9.13 a For r ≥ 0, let Dr be the disc with origin (0, 0) and radius r. Since 1 =R∞−∞

R∞−∞ f(x, y) dx dy =

RRD1

c dx dy = c · area of D1, we find that c = 1/π.

9.13 b Clearly FR(r) = 0 for r < 0, and FR(r) = 1 for r > 1. For r between 0 and1,

FR(r) = P((X, Y ) ∈ Dr) =

ZZDr

1

πdx dy =

πr2

π= r2.

Page 26: Probability and Statistics Book Solutions

482 Full solutions from MIPS: DO NOT DISTRIBUTE

9.13 c For x between −1 and 1,

fX(x) =

Z √1−x2

−√

1−x2

1

πdy =

2

π

p1− x2.

For x outside the interval −1, 1] we have that fX(x) = 0.

9.14 a Let f be the joint probability density function of the pair (X, Y ), and F

their joint distribution function. Since f(x, y) = ∂2

∂x ∂yF (x, y), we first determine F .

Setting G = (−∞, a]× (−∞, b], for −1 ≤ a, b ≤ 1, we have that

F (a, b) = P((X, Y ) ∈ G) =(a + 1)(b + 1)

4.

Furthermore, if a ≤ −1 or b ≤ −1, we have that F (a, b) = 0, since G ∩ = ∅. Ina similar way, we find for a > 1 and −1 ≤ b ≤ 1 that F (a, b) = (b + 1)/2, whilefor −1 ≤ a ≤ 1 and b > 1 we have that F (a, b) = (a + 1)/2. Finally, if a, b > 1,then F (a, b) = 1. Taking derivatives, it then follows that f(x, y) = 1/4 for a and bbetween −1 and 1, and f(x, y) = 0 for all other values of x and y.

9.14 b Note that for x between −1 and 1, the marginal probability density functionfX of X is given by

fX(x) =

Z 1

−1

f(x, y) dy =1

2,

and that fX(x) = 0 for all other values of x. Similarly, fY (y) = 1/2 for y between−1 and 1, and fY (y) = 0 otherwise. But then we find that f(x, y) = fX(x)fY (y),for all possible xs and ys, and we find that X and Y are independent, U(−1, 1)distributed random variables.

9.15 a Setting (a, b) as the set of points (x, y), for which x ≤ a and y ≤ b, wehave that

F (a, b) =area (∆ ∩(a, b))

area of ∆.

If a < 0 or if b < 0 (or both), then area (∆ ∩(a, b)) = ∅, so F (a, b) = 0, If (a, b) ∈ ∆, then area (∆ ∩(a, b)) = a(b− 1

2a), so F (a, b) = a(2b− a),

If 0 ≤ b ≤ 1, and a > b, then area (∆ ∩(a, b)) = 12b2, so F (a, b) = b2,

If 0 ≤ a ≤ 1, and b > 1, then area (∆ ∩(a, b)) = a− 12a2, so F (a, b) = 2a− a2,

If both a > 1 and b > 1, then area (∆ ∩(a, b)) = 12, so F (a, b) = 1.

9.15 b Since f(x, y) = ∂2

∂x ∂yF (x, y), we find for (x, y) ∈ ∆ that f(x, y) = 2. Fur-

thermore, f(x, y) = 0 for (x, y) outside the triangle ∆.

9.15 c For x between 0 and 1,

fX(x) =

Z ∞

−∞f(x, y) dy =

Z 1

x

2 dy = 2(1− x).

For y between 0 and 1,

fY (y) =

Z ∞

−∞f(x, y) dy =

Z y

0

2 dx = 2y.

Page 27: Probability and Statistics Book Solutions

29.1 Full solutions 483

9.16 Following the solution of Exercise 9.14 one finds that U and V are independentU(0, 1) distributed random variables. Using the results from Section 8.4 we find thatFU (x) = 1 − (1 − x)2 for x between 0 and 1, and that FV (y) = y2 for y between 0and 1. Differentiating yields the desired result.

9.17 For 0 ≤ s ≤ t ≤ a, it follows from the fact that U1 and U2 are independentuniformly distributed random variables over [0, a], that

P(U1 ≤ t, U2 ≤ t) = t2/a2,

and thatP(s < U1 ≤ t, s < U2 ≤ t) = (t− s)2/a2.

But then the answer is an immediate consequence of the hint.The statement can also be obtained as follows. Note that

P(V ≤ s, Z ≤ t) = P(U2 ≤ s, s < U1 ≤ t) + P(U1 ≤ s, s < U2 ≤ t)

+P(U1 ≤ s, U2 ≤ s) .

Using independence and that fact that Ui has distribution function FU1(u) = u/a,we find for 0 ≤ s ≤ t ≤ a:

P(V ≤ s, Z ≤ t) =s(t− s)

a2+

s(t− s)

a2+

s2

a2=

t2 − (t− s)2

a2.

9.18 a By definition

E[Xi] =

NXk=1

kpXi(k) =1

N(1 + 2 + · · ·+ N) =

1

N· 1

2N(N + 1) =

N + 1

2.

9.18 b Using the identity, we find

EX2

i

=

NXk=1

k2P(Xi = k) =1

N

NXk=1

k2 =(N + 1)(2N + 1)

6.

But then we have that

Var(Xi) = EX2

i

− E[Xi]2

=N2 − 1

12.

9.19 a Clearly we must have that a =√

50 = 5√

2, but then it follows that 2ab = 80,implying that b = 4

√2. Since 32 + c = 50, we find that c = 18.

9.19 b Note thatZ ∞

−∞e−(5

√2y−4

√2x)2 dy =

Z ∞

−∞e−

12 ( y−µ

σ ) dy

= σ√

Z ∞

−∞

1

σ√

2πe−

12 ( y−µ

σ ) dy

= σ√

2π,

since1

σ√

2πe−

12

y−µ

σ

for −∞ < y < ∞

is the probability density function of an N(µ, σ2) distributed random variable (andtherefore integrates to 1).

Page 28: Probability and Statistics Book Solutions

484 Full solutions from MIPS: DO NOT DISTRIBUTE

9.19 c

fX(x) =

Z ∞

−∞f(x, y) dy =

Z ∞

−∞

30

πe−50x2−50y2+80xy dy

=30

π

Z ∞

−∞e−(5

√2y−4

√2x)2e−18x2

dy

=30

π

√2π

10e−18x2

=1

16

√2π

e− 1

2

x

1/6

2,

for −∞ < x < ∞. So we see that X has a N(0, 136

) distribution.

9.20 a Since the needle hits the sheet of paper at an random position, the midpoint(X, Y ) falls completely randomly between some lines. Consequently, the distanceZ between (X, Y ) and the line “under” (X, Y ) has a U(0, 1) distribution. Also theorientation of the needle is completely random, so the angle between the needle andthe positive x-axis can be anything between 0 and 180 degrees. But then H has aU(0, π) distribution.

9.20 b From Figure 29.1 we see that—in case Z ≤ 1/2—the needle hits the lineunder it when Z ≤ 1

2sin H. In case Z > 1/2, we have a similar picture, but then

the needle hits the line above it when 1− Z ≤ 12

sin H.

(X, Y )

12

sin H

Z

.................................................................. ................

.....................................................................................................

............................................................................................................................

...........................................................................................................

Fig. 29.1. Solution of Exercise 9.20, case Z ≤ 1/2

9.20 c Since Z and H are independent, uniformly distributed random variables, theprobability we are looking for is equal to the area of the two regions in [0, π)× [0, 1]for which either

Z ≤ 1

2sin H or 1− Z ≤ 1

2sin H,

divided (!) by the total area π. Note that these two regions both have the same are,and that this area is equal toZ π

0

1

2sin H dH =

1

2· 2 = 1.

Page 29: Probability and Statistics Book Solutions

29.1 Full solutions 485

So the probability we are after is equal to

1

π+

1

π=

2

π.

10.1 a The joint probability distribution of X and Y is given by

a

b 1 2 3 P(Y = b)

1 0.217 0.153 0.057 0.4272 0.106 0.244 0.223 0.573

P(X = a) 0.323 0.397 0.280 1

This means that

E[X] = 1 · 0.323 + 2 · 0.397 + 3 · 0.280 = 1.957

E[Y ] = 1 · 0.427 + 2 · 0.573 = 1.573

E[XY ] = 1 · 1 · 0.217 + · · ·+ 2 · 3 · 0.223 = 3.220.

It follows that Cov(X, Y ) = E[XY ]− E[X] E[Y ] = 3.220− 1.957 · 0.245 = 0.142.

10.1 b From the joint probability distribution of X and Y we compute

EX2 = 1 · 0.323 + 4 · 0.397 + 9 · 0.280 = 4.431

Var(X) = EX2− (E[X])2 = 0.601

EY 2 = 1 · 0.427 + 4 · 0.573 = 2.719

Var(Y ) = EY 2− (E[Y ])2 = 0.245.

Using the result from a we find

ρ(X, Y ) =Cov(X, Y )p

Var(X)Var(Y )=

0.142√0.601 · 0.245

= 0.369.

10.2 a From Exercise 9.2:

x

y 0 1 2 pY (y)

−1 1/6 1/6 1/6 1/21 0 1/2 0 1/2

pX(x) 1/6 2/3 1/6 1

so that E[XY ] = 16· 1 · (−1) + 1

6· 2 · (−1) + 1

2· 1 · 1 = 0.

10.2 b E[Y ] = 12· (−1) + 1

2· 1 = 0, so that Cov(X, Y ) = E[XY ] − E[X] · E[Y ] =

0− 0 = 0.

10.2 c From the marginal distributions we find

Page 30: Probability and Statistics Book Solutions

486 Full solutions from MIPS: DO NOT DISTRIBUTE

E[X] = 0 · 1

6+ 1 · 2

3+ 2 · 1

6= 1

EX2 = 0 · 1

6+ 1 · 2

3+ 4 · 1

6= 1

Var(X) = EX2− (E[X])2 =

4

3− 12 =

1

3

EY 2 = (−1)2 · 1

2+ 12 · 1

2= 1

Var(Y ) = EY 2− (E[Y ])2 = 1− 02 = 1.

Since Cov(X, Y ) = 0 we have Var(X + Y ) = Var(X) + Var(Y ) = 13

+ 1 = 43.

10.2 d Since Cov(X,−Y ) = −Cov(X, Y ) = 0 we have Var(X − Y ) = Var(X) +Var(−Y ) = Var(X) + (−1)2Var(Y ) = 1

3+ 1 = 4

3.

10.3 We have E[U ] = 1, E[V ] = 12, EU2

= 32, EV 2

= 12, Var(U) = 1

2, Var(V ) =

14, and E[UV ] = 1

2. Hence Cov(U, V ) = 0 and ρ(U, V ) = 0.

10.4 Both X and Y have marginal probabilities ( 14, 1

4, 1

4, 1

4), so that

E[X] = 1 · 1

4+ 2 · 1

4+ 3 · 1

4+ 4 · 1

4= 2.5

E[Y ] = 1 · 1

4+ 2 · 1

4+ 3 · 1

4+ 4 · 1

4= 2.5

E[XY ] = 1 · 1 · 16

136+ · · ·+ 4 · 4 · 1

136= 6.25.

Hence Cov(X, Y ) = E[XY ]− E[X] E[Y ] = 6.25− 2.5 · 2.5 = 0.

10.5 a First find P(X = 1, Y = 0) = 13− 8

72− 10

72= 6

72, and similarly P(X = 0, Y = 2) =

472

and P(Y = 2) = 16. Then P(X = 1) = 1

4and P(X = 2) = 5

12, and finally

P(X = 2, Y = 2) = 571

:

a

b 0 1 2

0 8/72 6/72 10/72 1/31 12/72 9/72 15/72 1/22 4/72 3/72 5/72 1/6

1/3 1/4 5/12 1

10.5 b With a we find:

E[X] = 0 · 1

3+ 1 · 1

4+ 2 · 5

12=

13

12

E[Y ] = 0 · 1

3+ 1 · 1

2+ 2 · 1

6=

5

6

E[XY ] = 1 · 1 · 9

72+ · · ·+ 2 · 2 · 5

72=

65

72.

Hence Cov(X, Y ) = E[XY ]− E[X] E[Y ] = 6572− 13

12· 5

6= 0.

10.5 c Yes, for all combinations (a, b) it holds that P(X = a, Y = b) = P(X = a) P(Y = b).

Page 31: Probability and Statistics Book Solutions

29.1 Full solutions 487

10.6 a When c = 0, the joint distribution becomes

a

b −1 0 1 P(Y = b)

−1 2/45 9/45 4/45 1/30 7/45 5/45 3/45 1/31 6/45 1/45 8/45 1/3

P(X = a) 1/3 1/3 1/3 1

We find E[X] = (−1) · 13

+ 0 · 13

+ 1 · 13

= 0, and similarly E[Y ] = 0. By leaving outterms where either X = 0 or Y = 0, we find

E[XY ] = (−1) · (−1) · 2

45+ (−1) · 1 · 4

45+ 1 · (−1) · 6

45+ 1 · 1 · 8

45= 0,

which implies that Cov(X, Y ) = E[XY ]− E[X] E[Y ] = 0.

10.6 b Note that the variables X and Y in part b are equal to the ones from part a,shifted by c. If we write U and V for the variables from a, then X = U + c andY = V + c. According to the rule on the covariance under change of units, we thenimmediately find Cov(X, Y ) = Cov(U + c, V + c) = Cov(U, V ) = 0.Alternatively, one could also compute the covariance from Cov(X, Y ) = E[XY ] −E[X] E[Y ]. We find E[X] = (c−1) · 1

3+c · 1

3+(c+1) · 1

3= c, and similarly E[Y ] = c.

Since

E[XY ] = (c− 1) · (c− 1) · 2

45+ (c− 1) · c · 9

45+ (c + 1) · (c + 1) · 4

45

+c · (c− 1) · 7

45+ c · c · 5

45+ c · (c + 1) · 3

45

+(c + 1) · (c− 1) · 6

45+ (c + 1) · c · 1

45+ (c + 1) · (c + 1) · 8

45= c2,

we find Cov(X, Y ) = E[XY ]− E[X] E[Y ] = c2 − c · c = 0.

10.6 c No, X and Y are not independent. For instance, P(X = c, Y = c + 1) = 1/45,which differs from P(X = c) P(Y = c + 1) = 1/9.

10.7 a E[XY ] = 18, E[X] = E[Y ] = 1

2, so that Cov(X, Y ) = E[XY ]−E[X] E[Y ] =

18− ( 1

2)2 = − 1

8.

10.7 b Since X has a Ber( 12) distribution, Var(X) = 1

4. Similarly, Var(Y ) = 1

4, so

that

ρ(X, Y ) =Cov(X, Y )p

Var(X)Var(Y )=−1/8

1/4= −1

2.

10.7 c For any ε between − 14

and 14, Cov(X, Y ) = E[XY ]− ( 1

2)2 = ( 1

4−ε)− ( 1

2)2 =

−ε, so that

ρ(X, Y ) =Cov(X, Y )p

Var(X)Var(Y )=−ε

1/4= −4ε.

Hence, ρ(X, Y ) is equal to −1, 0, or 1, for ε equal to 1/4, 0, or −1/4.

10.8 a EX2

= Var(X) + (E[X])2 = 8.

10.8 b E−2X2 + Y

= −2E

X2+ E[Y ] = −2 · 8 + 3 = −13.

Page 32: Probability and Statistics Book Solutions

488 Full solutions from MIPS: DO NOT DISTRIBUTE

10.9 a If the aggregated blood sample tests negative, we do not have to performadditional tests, so that Xi takes on the value 1. If the aggregated blood sampletests positive, we have to perform 40 additional tests for the blood sample of eachperson in the group, so that Xi takes on the value 41. We first find that P(Xi = 1) =P(no infections in group of 40) = (1− 0.001)40 = 0.96, and therefore P(Xi = 41) =1− P(Xi = 1) = 0.04.

10.9 b First compute E[Xi] = 1·0.96+41·0.04 = 2.6. The expected total number oftests is E[X1 + X2 + · · ·+ X25] = E[X1]+E[X2]+· · ·+E[X25] = 25·2.6 = 65. Withthe original procedure of blood testing, the total number of tests is 25·40 = 1000. Onaverage the alternative procedure would only require 65 tests. Only with very smallprobability one would end up with doing more than 1000 tests, so the alternativeprocedure is better.

10.10 a We find

E[X] =

Z ∞

−∞xfX(x) dx =

Z 3

0

2

225

9x3 + 7x2 dx =

2

225

9

4x4 +

7

3x3

30

=109

50,

E[Y ] =

Z ∞

−∞yfY (y) dy =

Z 2

1

1

25(3y3 + 12y2) dy =

1

25

3

4y4 + 4y3

21

=157

100,

so that E[X + Y ] = E[X] + E[Y ] = 15/4.

10.10 b We find

EX2 =

Z ∞

−∞x2fX(x) dx =

Z 3

0

2

225

9x4 + 7x3 dx =

2

225

9

5x5 +

7

4x4

30

=1287

250,

EY 2 =

Z ∞

−∞y2fY (y) dy =

Z 2

1

1

25(3y4 + 12y3) dy =

1

25

3

5y5 + 3y4

21

=318

125,

E[XY ] =

Z 3

0

Z 2

1

xyf(x, y) dy dx =

Z 3

0

Z 2

1

2

75

2x3y2 + x2y3dy dx

=4

75

Z 3

0

x3

Z 2

1

y2 dy

dx +

2

75

Z 3

0

x2

Z 2

1

y3 dy

dx

=4

75

7

3

Z 3

0

x3 dx +2

75

15

4

Z 3

0

x2 dx =171

50,

so that E(X + Y )2

= E

X2+ E

Y 2+ 2E[XY ] = 3633/250.

10.10 c We find

Var(X) = EX2− (E[X])2 =

1287

250−

109

50

2

=989

2500,

Var(Y ) = EY 2− (E[Y ])2 =

318

125−

157

100

2

=791

10 000,

Var(X + Y ) = E(X + Y )2

− (E[X + Y ])2 =3633

250−

15

4

2

=939

2000.

Hence, Var(X) + Var(Y ) = 0.4747, which differs from Var(X + Y ) = 0.4695.

10.11 Cov(T, S) = Cov

95X + 32, 9

5X + 32

= ( 9

5)2Cov(X, Y ) = 9.72 and ρ(T, S) =

ρ

95X + 32, 9

5X + 32

= ρ(X, Y ) = 0.8.

Page 33: Probability and Statistics Book Solutions

29.1 Full solutions 489

10.12 Since H has a U(25, 35) distribution, it follows immediately that E[H] = 30.Furthermore

ER2 =

Z 12.5

7.5

r2 1

5dr =

r3

15

17.5

2.5 = 102.0833.

Hence πE[H] ER2

= π · 30 · 102.0833 = 9621.128.

10.13 a You can apply Exercise 8.15 directly for the case n = 2 to conclude thatE[X] = 1

3and E[Y ] = 2

3. In order to compute the variances we need the probability

densities of X and Y . Using the rules about the distribution of the minimum andmaximum of independent random variables in Section 8.4, we find

fX(x) = 2(1− x) for 0 ≤ x ≤ 1fY (y) = 2y for 0 ≤ y ≤ 1.

This means that EX2

=R 1

02x2(1− x) dx = 1

6, so that Var(X) = 1

6− ( 1

3)2 = 1

18.

Similarly, EY 2

=R 1

02y3 dy = 1

2, so that Var(Y ) = 1

2− ( 2

3)2 = 1

18.

10.13 b Since U and V are independent, each with variance 112

, it follows thatVar(X + Y ) = Var(U + V ) = Var(U) + Var(V ) = 1

6.

10.13 c To compute Cov(X, Y ) we need E[XY ]. This can be solved from

1

6= Var(X + Y ) = E

(X + Y )2

− (E[X + Y ])2

= EX2+ 2E[XY ] + E

Y 2− (E[X] + E[Y ])2.

Substitute E[X] = 13

and EX2

= Var(X) + (E[X])2 = 118

+ 19

= 318

, and similarlyE[Y ] = 2

3and E

Y 2

= Var(Y ) + (E[Y ])2 = 118

+ 49

= 12. This leads to E[XY ] = 1

4.

10.14 a By using the alternative expression for the covariance and linearity of ex-pectations, we find

Cov(X + s, Y + u)

= E[(X + s)(Y + u)]− E[X + s] E[Y + u]

= E[XY + sY + uX + su]− (E[X] + s)(E[Y ] + u)

= (E[XY ] + sE[Y ] + uE[X] + su)− (E[X] E[Y ] + sE[Y ] + uE[X] + su)

= E[XY ]− E[X] E[Y ]

= Cov(X, Y ) .

10.14 b By using the alternative expression for the covariance and the rule onexpectations under change of units, we find

Cov(rX, tY ) = E[(rX)(tY )]− E[rX] E[tY ]

= E[rtXY ]− (rE[X])(tE[Y ])

= rtE[XY ]− rtE[X] E[Y ]

= rt (E[XY ]− E[X] E[Y ])

= rtCov(X, Y ) .

10.14 c First applying part a and then part b yields

Cov(rX + s, tY + u) = Cov(rX, tY ) = rtCov(X, Y ) .

Page 34: Probability and Statistics Book Solutions

490 Full solutions from MIPS: DO NOT DISTRIBUTE

10.15 a Left plot: looks like 500 realizations from a pair (X, Y ) whose 2-dimensionaldistribution has contourlines that are circles. This means X and Y are uncorrelated.Middle plot: looks like 500 realizations from a pair (X, Y ) whose 2-dimensionaldistribution has contourlines that are ellipsoids with the line y = x as main axis. Thismeans X and Y are positively correlated; Right plot: looks like 500 realizations froma pair (X, Y ) whose 2-dimensional distribution has contourlines that are ellipsoidswith the line y = x as main axis. This means X and Y are negatively correlated.

10.15 b In the right picture the points are concentrated more closely than in theother pictures. Hence |ρ(X, Y )| will be the largest for the right picture.

10.16 a Cov(X, X + Y ) = Cov(X, X) + Cov(X, Y ) = Var(X) + Cov(X, Y )

10.16 b Anything can happen, depending on the sign of Cov(X, Y ) and its magni-tude compared to Var(X).

10.16 c If X and Y are uncorrelated, Cov(X, Y ) = Var(X) ≥ 0, so apart from thespecial case where X is constant, X and X + Y are positively correlated.

10.17 a Since all expectations are zero, it is sufficient to show

E(X + Y + Z)2

= E

X2+ E

Y 2+ E

Y 2+ 2E[XY ] + 2E[XZ] + 2E[Y Z] .

This follows immediately from the hint with n = 3, and using linearity of expecta-tions.

10.17 b Write X = X − E[X], and similarly Y and Z. Then part a applies to X,Y , and Z:

VarX + Y + Z

= Var

X

+ VarY

+ VarZ

+ 2CovX, Y

+ 2Cov

X, Z

+ 2Cov

Y , Z

.

According to the rules on pages 104 and 151 about the variance and covariance under

a change of units, it follows that VarX

= Var(X), CovX, Y

= Cov(X, Y ), and

similarly for all other variances and covariances.

10.17 c Similar to part a first consider the case with all random variables havingexpectation zero. As in part a, the hint in a together with linearity of expectationsyields the desired equality. Then argue as in part b by introducing Xi = Xi−E[Xi].

10.17 d Use part c and note that there are n terms Var(Xi) and n(n − 1) termsCov(Xi, Xj) with i 6= j.

10.18 First note that X1 + X2 + · · · + XN is the sum of all numbers, which isa nonrandom constant. Therefore, Var(X1 + X2 + · · ·+ XN ) = 0. In Section 9.3we argued that, although we draw without replacement, each Xi has the samedistribution. By the same reasoning, we find that each pair (Xi, Xj), with i 6= j,has the same joint distribution, so that Cov(Xi, Xj) = Cov(X1, X2) for all pairswith i 6= j. Direct application of Exercise 10.17 with σ2 = (N − 1)(N + 1) andγ = Cov(X1, X2) gives

0 = Var(X1 + X2 + · · ·+ XN ) = N · (N − 1)(N + 1)

12+ N(N − 1)Cov(X1, X2) .

Solving this identity gives Cov(X1, X2) = −(N + 1)/12.

Page 35: Probability and Statistics Book Solutions

29.1 Full solutions 491

10.19 By definition and linearity of expectations:

Cov(X, Y ) = E[(X − E[X])(Y − E[Y ])]

= E[XY −XE[Y ]− E[X] Y + E[X] E[Y ]]

= E[XY ]− E[XE[Y ]]− E[E[X] Y ] + E[X] E[Y ]

= E[XY ]− E[X] E[Y ]− E[X] E[Y ] + E[X] E[Y ]

= E[XY ]− E[X] E[Y ] .

10.20 To compute the correlation, we need:

CovU, U2 = E

U3− E[U ] E

U2 ,

Var(U) = EU2− (E[U ])2 ,

VarU2 = E

U4− EU22 .

Hence we determine

EhUki

=

Z a

0

uk 1

adu =

ak

k + 1,

which yields

CovU, U2 =

a3

4− a

2

a2

3=

1

4− 1

2

1

3

a3 =

1

12a3,

Var(U) =a2

3−a

2

2

=

1

3− 1

4

a2 =

1

12a2,

VarU2 =

a4

5−

a2

3

2

=

1

5− 1

9

a4 =

4

45a4.

Hence

ρU, U2 =

1

12

√135 =

1

4

√15 = 0.968.

Note that the answer does not depend on a.

11.1 a In the addition rule, k is between 2 and 12. We must always have 1 ≤ k−` ≤ 6and 1 ≤ ` ≤ 6, or equivalently, k − 6 ≤ ` ≤ k − 1 and 1 ≤ ` ≤ 6. For k = 2, . . . , 6,this means

pZ(k) = P(X + Y = k) =

k−1X=1

pX(k − `)pY (`) =

k−1X=1

1

6· 1

6=

k − 1

36

and for k = 7, . . . , 12,

pZ(k) = P(X + Y = k) =

6X`=k−6

pX(k − `)pY (`) =

6X`=k−6

1

6· 1

6=

13− k

36.

11.1 b In the addition rule, k is between 2 and 2N . We must always have 1 ≤k − ` ≤ N and 1 ≤ ` ≤ N , or equivalently, k −N ≤ ` ≤ k − 1 and 1 ≤ ` ≤ N . Fork = 2, . . . , N , this means

pZ(k) = P(X + Y = k) =

k−1X=1

pX(k − `)pY (`) =

k−1X=1

1

N· 1

N=

k − 1

N2

Page 36: Probability and Statistics Book Solutions

492 Full solutions from MIPS: DO NOT DISTRIBUTE

and for k = N + 1, . . . , 2N ,

pZ(k) = P(X + Y = k) =

NX`=k−N

pX(k − `)pY (`) =

NX`=k−N

1

N· 1

N=

2N − k + 1

N2.

11.2 a By using the rule on addition of two independent discrete random variables,we have

P(X + Y = k) = pZ(k) =

∞X=0

pX(k − `)pY (`).

Because pX(a) = 0 for a ≤ −1, all terms with ` ≥ k + 1 vanish, so that

P(X + Y = k) =

kX=0

1k−`

(k − `)!e−1 · 1`

`!e−1 =

e−2

k!

kX=0

k

`

!=

2k

k!e−2,

also usingPk

`=0

k`

= 2k in the last equality.

11.2 b Similar to part a, by using the rule on addition of two independent discreterandom variables and leaving out terms for which pX(a) = 0, we have

P(X + Y = k) =

kX=0

λk−`

(k − `)!e−λ · µ`

`!e−µ =

(λ + µ)k

k!e−(λ+µ)

kX=0

k

`

!λk−`µ`

(λ + µ)k.

Next, write

λk−`µ`

(λ + µ)k=

µ

λ + µ

λ + µ

k−`

=

µ

λ + µ

` 1− µ

λ + µ

k−`

= p`(1− p)k−`

with p = µ/(λ + µ). This means that

P(X + Y = k) =(λ + µ)k

k!e−(λ+µ)

kX=0

k

`

!p`(1− p)k−` =

(λ + µ)k

k!e−(λ+µ),

using thatPk

`=0

k`

p`(1− p)k−` = 1.

11.3

pZ(k) =

kX=0

n

k − `

! m

`

!( 12)k−`( 1

4)`( 1

2)n−(k−`)( 3

4)m−`

=

kX=0

n

k − `

! m

`

!( 12)n( 1

4)`( 3

4)m−`.

This cannot be simplified and is not equal to a binomial probability of the typen+m

k

rk(1− r)n+m−k for some 0 < r < 1.

11.4 a From the fact that X has an N(2, 5) distribution, it follows that E[X] =2 and Var(X) = 5. Similarly, E[Y ] = 5 and Var(Y ) = 9. Hence by linearity ofexpectations,

E[Z] = E[3X − 2Y + 1] = 3E[X]− 2E[Y ] + 1 = 3 · 2− 2 · 5 + 1 = −3.

By the rules for the variance and covariance,

Var(Z) = 9Var(X) + 4Var(Y )− 12Cov(X, Y ) = 9 · 5 + 4 · 9− 12 · 0 = 81,

using that Cov(X, Y ) = 0, due to independence of X and Y .

Page 37: Probability and Statistics Book Solutions

29.1 Full solutions 493

11.4 b The random variables 3X and −2Y + 1 are independent and, according tothe rule for the normal distribution under a change of units (page 112), it followsthat they both have a normal distribution. Next, the sum rule for independentnormal random variables then yields that Z = (3X) + (−2Y + 1) also has a normaldistribution. Its parameters are the expectation and variance of Z. From a it followsthat Z has an N(−3, 81) distribution.

11.4 c From b we know that Z has an N(−3, 81) distribution, so that (Z + 3)/9has a standard normal distribution. Therefore

P(Z ≤ 6) = P

Z + 3

9≤ 6 + 3

9

= Φ(1),

where Φ is the standard normal distribution function. From Table ?? we find thatΦ(1) = 1− 0.1587 = 0.8413.

11.5 According to the addition rule, the probability density of Z = X + Y is givenby

fZ(z) =

Z ∞

−∞fX(z − y)fY (y) dy =

Z 1

0

fX(z − y)fY (y) dy,

where we use that fY (y) = 0 for y /∈ [0, 1]. When 0 ≤ y ≤ 1, the following holds.For z < 0, also z − y < 0 so that fX(z − y) = 0, and for z > 2, z − y > 1 so thatfX(z − y) = 0. For 0 ≤ z < 1:

fZ(z) =

Z 1

0

fX(z − y)fY (y) dy =

Z z

0

1 dy = z,

whereas for 1 ≤ z ≤ 2:

fZ(z) =

Z 1

0

fX(z − y)fY (y) dy =

Z 1

z−1

1 dy = 2− z.

11.6 According to the addition rule

fZ(z) =

Z z

0

fX(z − y)fY (y) dy =

Z z

0

1

4(z − y)e−(z−y)/2 1

4ye−y/2 dy.

=1

16e−z/2

Z z

0

(z − y)y dy =1

16e−z/2

1

2zy2 − 1

3y3

z

0

=z3

96e−z/2.

11.7 Each Xi has the same distribution as the sum of k independent Exp(λ) dis-tributed random variables. Since the Xi are independent, X1 + X2 + · · · + Xn hasthe same distribution as the sum of nk independent Exp(λ) distributed randomvariables, which is a Gam(nk, λ) distribution.

11.8 a Y = rX + s has probability density function

fY (y) =1

rfX

y − s

r

=

1

rπ1 +

y−s

r

2 =r

π (r2 + (y − s)2).

We see that Y = rX + s has a Cau(s, r) distribution.

11.8 b If Y = (X − β)/α then Y has a Cau(0, 1) distribution.

Page 38: Probability and Statistics Book Solutions

494 Full solutions from MIPS: DO NOT DISTRIBUTE

11.9 a According to the product rule on page 172,

fZ(z) =

Z z

1

fY

z

x

fX(x)

1

xdx =

Z z

1

1zx

2 3

x4

1

xdx

=3

z2

Z z

1

1

x3dx =

3

z2

−1

2x−2

z

1

=3

2

1

z2

1− 1

z2

=

3

2

1

z2− 1

z4

.

11.9 b According to the product rule,

fZ(z) =

Z z

1

fY

z

x

fX(x)

1

xdx =

Z z

1

βzx

β+1

α

xα+1

1

xdx

=αβ

zβ+1

Z z

1

xβ−α−1 dx =αβ

zβ+1

xβ−α

β − α

z

1

=αβ

α− β

1

zβ+1

1− zβ−α

=

αβ

β − α

1

zβ+1− 1

zα+1

.

11.10 a In the quotient rule for Z = X/Y for 0 < z < ∞ fixed, we must have1 ≤ zx < ∞ and 1 ≤ x < ∞. Hence for 0 < z < 1,

fZ(z) =

Z ∞

1/z

fX(zx)fY (y)x dx =

Z ∞

1/z

2

(zx)32

x3x dx

=4

z3

Z ∞

1/z

x−5 dx =4

z3

−x−4

4

∞1/z

=1

z3

0− (−z4)

= z.

For z ≥ 1, we find

fZ(z) =

Z ∞

1

fX(zx)fY (y)x dx =

Z ∞

1

2

(zx)32

x3x dx

=4

z3

Z ∞

1

x−5 dx =4

z3

−x−4

4

∞1

=1

z3.

11.10 b In the quotient rule for Z = X/Y for 0 < z < ∞ fixed, we must have1 ≤ zx < ∞ and 1 ≤ x < ∞. Hence for 0 < z < 1,

fZ(z) =

Z ∞

1/z

fX(zx)fY (y)x dx =

Z ∞

1/z

α

(zx)α+1

β

xβ+1x dx

=αβ

zα+1

Z ∞

1/z

x−α−β−1 dx =αβ

zα+1

−x−α−β

α + β

∞1/z

=αβ

α + β

1

zα+1

0− (−zα+β)

=

αβ

α + βzβ−1.

For z ≥ 1, we find

fZ(z) =

Z ∞

1/z

fX(zx)fY (y)x dx =

Z ∞

1

α

(zx)α+1

β

xβ+1dx

=αβ

zα+1

Z ∞

1

x−α−β−1 dx =αβ

zα+1

−x−α−β

α + β

∞1

=αβ

α + β

1

zα+1.

Page 39: Probability and Statistics Book Solutions

29.1 Full solutions 495

11.11 a Put T3 = X1 + X2 + X3. Then T3 can be interpreted as the time of thethird success in a series of independent experiments with probability p of success.Then T3 = k means that the first (k− 1) experiments contained exactly 2 successes,and the k-th experiment was also a success. Since the number of successes in (k−1)experiments has a Bin(k − 1, p) distribution, it follows that

P(T3 = k) =

k − 1

2

!p2(1− p)(k−1−2) · p =

1

2(k − 1)(k − 2)p3(1− p)k−3.

11.11 b This follows from a smart calculation:

∞Xk=3

pZ(k) = 1

⇔∞X

k=3

1

2(k − 1)(k − 2)p3(1− p)k−3 = 1

⇔ 1

2p2

∞Xk=3

(k − 1)(k − 2)p(1− p)k−3 = 1 ( now put k − 2 = m)

⇔ 1

2p2

∞Xm=1

m(m + 1)p(1− p)m−1 = 1

⇔ 1

2p2

∞Xm=1

(m2 + m)P(X1 = m) = 1

⇔ 1

2p2 (E

X2

1

+ E[X1]) = 1

⇔ p2 (EX2

1

+ E[X1]) = 2.

11.11 c The first part follows directly from b:

EX2

1

=

2

p2− E[X1] =

2

p2− 1

p=

2− p

p2.

For the second part:

Var(X1) = EX2

1

− (E[X1])2 =

2− p

p2−

1

p

2

=1− p

p2.

11.12 Γ(1) =R∞0

e−t dt = 1 and

Γ(x + 1) =

Z ∞

0

txe−t dt =−txe−tt=∞

t=0+

Z ∞

0

xtx−1e−t dt.

Since x > 0, the first term on the right hand side is zero and the second term isequal to xΓ(x).

11.13 a

Page 40: Probability and Statistics Book Solutions

496 Full solutions from MIPS: DO NOT DISTRIBUTE

P(Zn ≤ a) =

Z a

0

λnzn−1e−λz

(n− 1)!dz

=

−λn−1zn−1e−λz

(n− 1)!

z=a

z=0

+

Z a

0

λn−1zn−2e−λz

(n− 2)!dz

= − (λa)n−1

(n− 1)!e−λa +

Z a

0

λn−1zn−2e−λz

(n− 2)!dz

= − (λa)n−1

(n− 1)!e−λa + P(Zn−1 ≤ a) .

11.13 b Use part a recursively:

P(Zn ≤ a) = − (λa)n−1

(n− 1)!e−λa + P(Zn−1 ≤ a)

= − (λa)n−1

(n− 1)!e−λa − (λa)n−2

(n− 2)!e−λa + P(Zn−2 ≤ a)

...

= −n−1Xi=1

(λa)i

i!e−λa + P(Z1 ≤ a) .

11.13 c Since P(Z1 ≤ a) = 1− e−λa, we find

P(Zn ≤ a) = P(Z1 ≤ a)−n−1Xi=1

(λa)i

i!e−λa

= 1− e−λa −n−1Xi=1

(λa)i

i!e−λa = 1−

n−1Xi=0

(λa)i

i!e−λa.

12.1 e This is certainly open to discussion. Bankruptcies: no (they come in clusters,don’t they?). Eggs: no (I suppose after one egg it takes the chicken some time toproduce another). Examples 3 and 4 are the best candidates. Example 5 could bemodeled by the Poisson process if the crossing is not a dangerous one; otherwiseauthorities might take measures and destroy the homogeneity.

12.2 Let X be the number of customers on a day. Given is that P(X = 0) = 10−5.Since X is Poisson distributed, P(X = 0) = e−λ. So e−λ = 10−5, which implies−λ = −5 ln(10), and hence λ = 11.5. Then also E[X] = λ = 11.5.

12.3 When N has a Pois(4) distribution, P(N = 4) = 44e−4/4! = 0.195.

12.4 When X has a Pois(2) distribution, P(X ≤ 1) = P(X = 0) + P(X = 1) =e−2 + 2e−2 = 3e−2 = 0.406.

12.5 a Model the errors in the bytes of the hard disk as a Poisson process, withintensity λ per byte. Given is that λ · 220 = 1, or λ = 2−20. The expected numberof errors in 512 = 29 bytes is λ29 = 2−11 = 0.00049.

12.5 b Let Y be the number of errors on the hard disk. Then Y has a Poissondistribution with parameter µ = 39054015× 0.00049 = 19069.34. Then P(Y ≥ 1) =1−P(Y = 0) = 1− e−19069.34 = 1.00000 · · · . For all practical purposes this happenswith probability 1.

Page 41: Probability and Statistics Book Solutions

29.1 Full solutions 497

12.6 The expected numbers of flaws in 1 meter is 100/40 = 2.5, and hencethe number of flaws X has a Pois(2.5) distribution. The answer is P(X = 2) =12!

(2.5)2e−2.5 = 0.256.

12.7 a It is reasonable to estimate λ with (nr. of cars)/(total time in sec.) = 0.192.

12.7 b 19/120 = 0.1583, and if λ = 0.192 then P(N(10) = 0) = e−0.192·10 = 0.147.

12.7 c P(N(10) = 10) with λ from a seems a reasonable approximation of this prob-ability. It equals e−1.92 · (0.192 · 10)10/10! = 2.71 · 10−5.

12.8 a We have

E[X(X − 1)] =

∞Xk=0

k(k − 1)e−λλk/k! =

∞Xk=2

e−λλk/(k − 2)!

= λ2∞X

k=2

e−λλk−2/(k − 2)! = λ2∞X

j=0

e−λλj/j! = λ2.

12.8 b Since Var(X) = E[X(X − 1)] + E[X] − (E[X])2, we have Var(X) = λ2 +λ− λ2 = λ.

12.9 In a Poisson process with intensity 1, the number of points in a interval oflength t has a Poisson distribution with parameter t. So the number Y1 of pointsin [0, µ1] is Pois(µ1) distributed, and the number Y2 in [µ1 + µ1 + µ2] is Pois(µ2)distributed. But the sum Y1 +Y2 of these is equal to the number of points in [0, µ1 +µ2] , and so is Pois(µ1 + µ2) distributed.

12.10 We have to consider the numbers pk = P(X = k) = µk

k!e−µ. To compare

them, divide: pk+1/pk = µ/(k +1), and note that pk < ( or > )pk+1 is equivalent topk+1/pk > ( or < )1.So it follows immediately that— for µ < 1 the probabilities P(X = k) are decreasing, and

— for µ > 1 the probabilities are increasing as long as µk+1

> 1,and decreasing from the moment where this fraction has become less than 1.

Lastly if µ = 1, then p0 = p1 > p2 > p3 > . . . .

12.11 Following the hint, we obtain:

P(N([0, s] = k, N([0, 2s]) = n) = P(N([0, s]) = k, N((s, 2s]) = n− k)

= P(N([0, s]) = k) · P(N((s, 2s]) = n− k)

= (λs)ke−λs/(k!) · (λs)n−ke−λs/((n− k)!)

= (λs)ne−λ2s/(k!(n− k)!).

So

P(N([0, s]) = k |N([0, 2s]) = n) =P(N([0, s]) = k, N([0, 2s]) = n)

P(N([0, 2s]) = n)

= n!/(k!(n− k)!) · (λs)n/(2λs)n

= n!/(k!(n− k)!) · (1/2)n.

This holds for k = 0, . . . , n, so we find the Bin(n, 12) distribution.

12.12 a The event X2 ≤ t is a disjoint union of the events X1 ≤ s, X2 ≤ t andX1 > s, X2 ≤ t.

Page 42: Probability and Statistics Book Solutions

498 Full solutions from MIPS: DO NOT DISTRIBUTE

12.12 b If X2 ≤ t and Na = 2, then the first two points of the Poisson process mustlie in [0, t], and no points lie in (t, a], and conversely. Therefore

P(X2 ≤ t, Na = 2) = P(N([0, t]) = 2, N((t, a]) = 0)

= λ2t2

2!e−λt · e−λ(a−t)

= 12λ2t2e−λa.

Similarly,

P(X1 > s, X2 ≤ t, Na = 2) = P(N([0, s)) = 0, N([s, t]) = 2, N((t, a]) = 0)

= e−λs · λ2(t−s)2

2!e−λ(t−s) · e−λ(a−t)

= 12λ2(t− s)2e−λa.

The result now follows by using part a.

12.12 c This follows immediately from part b since

P(X1 ≤ s, X2 ≤ t |Na = 2) =P(X1 ≤ s, X2 ≤ t, Na = 2)

P(Na = 2),

where the denominator equals 12λ2a2 e−λa.

12.13 a Given that Nt = n + m, the number of daisies Xt has a binomial dis-tribution with parameters n + m and 1/4. So P(Xt = n, Yt = m|Nt = n + m) =

P(Xt = n|Nt = n + m) =

n+mn

14

n34

m

.

12.13 b Using part a we find thatP(Xt = n, Yt = m) = P(Xt = n, Yt = m|Nt = n + m) P(Nt = n + m) =

n+mn

14

n34

m(λt)n+me−λt

(n+m)!= 1

n!1

m!

14

n34

m

(λt)n+me−λt.

12.13 c Using b we find thatP(Xt = n) =

P∞m=0 P(Xt = n, Yt = m) =

1n!

( 14)n(λt)ne−(λ/4)tP∞

m=01

m!(3λ/4)me−(3λ/4)t = 1

n!((λ/4)t)ne−(λ/4)t,

since the sum over m just adds all the probabilities P(Z = m) where Z has aPois(3λ/4) distribution.

12.14 a This follows since 1− 1/n → 1 and 1/n → 0.

12.14 b This is an easy computation: E[Xn] = (1− 1n) · 0 + ( 1

n) · 7n = 7 for all n.

13.1 For U(−1, 1), µ = 0 and σ = 1/√

3, and we obtain the following table:

k 1 2 3 4

P(|Y − µ| < kσ) 0.577 1 1 1

For U(−a, a), µ = 0 and σ = a/√

3, and we obtain the following table:

k 1 2 3 4

P(|Y − µ| < kσ) 0.577 1 1 1

For N(0, 1), µ = 0 and σ = 1, and we obtain the following table:

k 1 2 3 4

P(|Y − µ| < kσ) 0.6826 0.9544 0.9974 1

Page 43: Probability and Statistics Book Solutions

29.1 Full solutions 499

For N(µ, σ2), P(|Y − µ| < kσ) = P(|Y − µ|/σ < k), and we obtain the same resultas for N(0, 1).For Par(3), µ = 3/2, σ =

p3/4, and

P(|Y − µ| < kσ) =

Z 3/2+k√

3/2

1

3x−4 dx = 1− (3/2 + k√

3/2)−3,

and we obtain the following table:

k 1 2 3 4

P(|Y − µ| < kσ) 0.925 0.970 0.985 0.992

For Geo(3), µ = 2, σ = 2, and P(|Y − µ| < kσ) = P|Y − 2| < k

√2

=

P2− k

√2 <Y <2 + k

√2

= PY < 2 + k

√2, and we obtain the following table:

k 1 2 3 4

P(|Y − µ| < kσ) 0.875 0.9375 0.9844 0.9922

13.2 a From the formulas for the U(a, b) distribution, substituting a = −1/2 andb = 1/2, we derive that E[Xi] = 0 and Var(Xi) = 1/12.

13.2 b We write S = X1 + X2 + · · ·+ X100, for which we find E[S] = E[X1] + · · ·+E[X100] = 0 and, by independence, Var(S) = Var(X1)+ · · ·+Var(X100) = 100 · 1

12=

100/12. We find from Chebyshev’s inequality:

P(|S| > 10) = P(|S − 0| > 10) ≤ Var(S)

102=

1

12.

13.3 We can apply the law of large numbers to the sequence (Yi), with Yi = |Xi|for i = 1, . . . . Since E[Yi] = E[|Xi|] = 2

R 0.5

0x dx = 2 · 1

2· (0.5)2 = 0.25, and since

the Yi have finite variance, it follows that 1n

Pni=1 |Xi| → 0.25 as n →∞.

13.4 a Because Xi has a Ber(p) distribution, E[Xi] = p and Var(Xi) = p (1 − p),and so E

Xn

= p and Var

Xn

= Var(Xi) /n = p (1 − p)/n. By Chebyshev’s

inequality:

P|Xn − p| ≥ 0.2

≤ p (1− p)/n

(0.2)2=

25p(1− p)

n.

The right-hand side should be at most 0.1 (note that we switched to the comple-ment). If p = 1/2 we therefore require 25/(4n) ≤ 0.1, or n ≥ 25/(4 · 0.1) = 62.5,i.e., n ≥ 63. Now, suppose p 6= 1/2, using n = 63 and p(1 − p) ≤ 1/4 we concludethat 25p(1− p)/n ≤ 25 · (1/4)/63 = 0.0992 < 0.1, so (because of the inequality) thecomputed value satisfies for other values of p as well.

13.4 b For arbitrary a > 0 we conclude from Chebyshev’s inequality:

P|Xn − p| ≥ a

≤ p (1− p)/n

a2=

p(1− p)

na2≤ 1

4na2,

where we used p (1− p) ≤ 1/4 again. The question now becomes: when a = 0.1, forwhat n is 1/(4na2) ≤ 0.1? We find: n ≥ 1/(4 · 0.1 · (0.1)2) = 250, so n = 250 is largeenough.

Page 44: Probability and Statistics Book Solutions

500 Full solutions from MIPS: DO NOT DISTRIBUTE

13.4 c From part a we know that an error of size 0.2 or occur with a probabilityof at most 25/4n, regardless of the values of p. So, we need 25/(4n) ≤ 0.05, i.e.,n ≥ 25/(4 · 0.05) = 125.

13.4 d We compute PXn ≤ 0.5

for the case that p = 0.6. Then E

Xn

= 0.6

and VarXn

= 0.6 · 0.4/n. Chebyshev’s inequality cannot be used directly, we need

an intermediate step: the probability that Xn ≤ 0.5 is contained in the event “theprediction is off by at least 0.1, in either direction.” So

PXn ≤ 0.5

≤ P|Xn − 0.6| ≥ 0.1

≤ 0.6 · 0.4/n

(0.1)2=

24

n

For n ≥ 240 this probability is 0.1 or smaller.

13.5 To get P|Mn − c| ≤ 0.5

≥ 0.9, we must have P|Un| ≤ 0.5

≥ 0.9. Now

P|Un| ≥ 0.5

≤ Var(Ui)/n

(0.5)2= 3/n

0.25= 12

n. If we want this below 0.1, we should take

n ≥ 120.

13.6 The probability distribution for an individual game has P(X = −1) = 1837

, andP(X = 1) = 19

37. From this we compute E[X] = (−1) · 18

37+1 · 19

37= 1

37. If the game is

played 365 ·1000 times, and we suppose that the roulette machine is “fair”, the totalgain will, according to the law of large numbers, be close to 365 · 1000 · 1

37= 9865

(Euro). (Using Chebyshev’s inequality one can, for example, find a lower bound forthe probability that the total gain will be between 9865− 250 and 9865 + 250.)

13.7 Following the hint we define Yi = 1 when Xi ∈ (−∞, a], and 0 otherwise.In this way we have written Fn(a) = 1

n

Pni=1 Yi. The expectation of Yi equals

E[Yi] = P(Xi ∈ (−∞, a]) = F (a). Moreover, Var(Yi) = F (a)(1 − F (a)) is finite.Since Fn(a) = Yn, the law of large numbers tells us that

limn→∞

P(|Fn(a)− F (a)| > ε) = 0.

13.8 a We have p = E[Y ] =R a+h

a−hxe−x dx ≈ 2h · 2e−2. (Of course the integral

can be computed exactly, but this approximation is excellent: for h = 0.25 we getp = 0.13534, while the exact result is p = 0.13533.) Now Var(Y ) = p(1 − p) =4he−2(1− 4he−2), and

VarYn/2h

=

1

4h2n4he−2(1− 4he−2).

13.8 b What is required is that

PYn/2h− f(a)

≥ 0.2f(a) ≤ 0.2.

Chebyshev’s inequality yields that

PYn/2h− f(a)

≥ 0.2f(a) ≤ Var

Yn/2h

0.04f(a)2

,

so we want VarYn/2h

≤ 0.008 · (2e−2)2. Using part a this leads to

1

n≤ 0.008 · e−2

1− e−2=⇒ n ≥ e2 − 1

0.008= 798.6 =⇒ n = 799.

Page 45: Probability and Statistics Book Solutions

29.1 Full solutions 501

13.9 a The statement looks like the law of large numbers, and indeed, if we lookmore closely, we see that Tn is the average of an i.i.d. sequence: define Yi = X2

i ,then Tn = Yn. The law of large numbers now states: if Yn is the average of nindependent random variables with expectation µ and variance σ2, then for anyε > 0: limn→∞ P

|Yn − µ| > ε

= 0. So, if a = µ and the variance σ2 is finite, thenit is true.

13.9 b We compute expectation and variance of Yi: E[Yi] = EX2

i

=R 1

−112x2 dx =

1/3. And: EY 2

i

= E

X4

i

=R 1

−112x4 dx = 1/5, so Var(Yi) = 1/5− (1/3)2 = 4/45.

The variance is finite, so indeed, the law of large numbers applies, and the statementis true if a = E

X2

i

= 1/3.

13.10 a For 0 ≤ ε ≤ 1:

P(|Mn − 1| > ε) = P(1− ε < Mn < 1 + ε) = P(1− ε < Mn) = (1− ε)n.

13.10 b For any 0 < ε: limn→∞(1− ε)n = 0. The conclusion is that Mn convergesto 1, as n goes to infinity. This cannot be obtained by a straightforward applicationof Chebyshev’s inequality or the law of large numbers.

13.11 a We have P(|X − 1| > 8) = P(X = 10) = 1/10. On the other hand we getfrom Chebyshev’s inequality P(|X − 1| > 8) ≤ (t− 1)/64 = 9/64.

13.11 b For a = 5 we haveEXACT: P(|X − 1| > 5) = P(X = 10) = 1/10, CHEB: P(|X − 1| > 5) ≤ 9/25,so the Chebyshev gap equals 9/25-1/10=0.26. For a = 10 we haveEXACT: P(|X − 1| > 10) = 0, CHEB: P(|X − 1| > 10) ≤ 9/100,so the Chebyshev gap equals 9/100=0.09.

13.11 c Choosing a = t as in the previous question, we have P(|X − 1| > t) = 0,and P(|X − 1| > t) ≤ (t − 1)/t2 < 1/t. So to answer the question we can simplychoose t = 100, t = 1000 and t = 10 000.

13.11 d As we have seen that the Chebyshev gap can be made arbitrarily small(taking arbitrarily large t in part c) we can not find a closer bound for this familyof probability distributions.

13.12 a Since EXn

= 1

n

Pni=1 µi, and Var

Xn

= 1

n2

Pni=1 Var(Xi), this inequal-

ity follows directly from Chebyshev’s.

13.12 b This follows directly from

VarXn

=

1

n2

nXi=1

Var(Xi) ≤ 1

nM → 0 as n →∞.

14.1 Since µ = 2 and σ = 2, we find that

P(X1 + X2 + · · ·+ X144 > 144) = PX144 > 1

= P

√144

X144 − µ

σ>√

1441− µ

σ

= P

Z144 > 12

1− 2

2

≈ 1− Φ(−6) = 1.

Page 46: Probability and Statistics Book Solutions

502 Full solutions from MIPS: DO NOT DISTRIBUTE

14.2 Since E[Xi] = 1/4 and Var(Xi) = 3/80, using the central limit theorem wefind

P(X1 + X2 + · · ·+ X625 < 170) = P

√625

X625 − 1/4p3/80

<√

625170/625− 1/4p

3/80

!≈ Φ(2.8402) = 1− 0.0023 = 0.9977.

14.3 First note that P|Xn − p| < 0.2

= 1−P

Xn − p ≥ 0.2

−PXn − p ≤ −0.2

.

Because µ = p and σ2 = p(1− p), we find, using the central limit theorem:

PXn − p ≥ 0.2

= P

nXn − ppp(1− p)

≥ √n

0.2pp(1− p)

!= P

Zn ≥

√n

0.2pp(1− p)

!≈ P

Z ≥ √

n0.2p

p(1− p)

!,

where Z has an N(0, 1) distribution. Similarly,

PXn − p ≤ −0.2

≈ P

Z ≥ √

n0.2p

p(1− p)

!,

so we are looking for the smallest positive integer n such that

1− 2P

Z ≥ √

n0.2p

p(1− p)

!≥ 0.9,

i.e., the smallest positive integer n such that

P

Z ≥ √

n0.2p

p(1− p)

!≤ 0.05.

From Table ?? it follows that

√n

0.2pp(1− p)

≥ 1.645.

Since p(1− p) ≤ 1/4 for all p between 0 and 1, we see that n should be at least 17.

14.4 Using the central limit theorem, with µ = 2, and σ2 = 4,

P(T1 + T2 + · · ·+ T30 ≤ 60) = P

√30

T30 − 2

2≤√

302− 2

2

≈ Φ(0) =

1

2.

14.5 In Section 4.3 we have seen that X has the same probability distributionas X1 + X2 + · · · + Xn, where X1, X2, . . . , Xn are independent Ber(p) distributedrandom variables. Recall that E[Xi] = p, and Var(Xi) = p(1−p). But then we havefor any real number a that

P

X − nppnp(1− p)

≤ a

!= P

X1 + X2 + · · ·+ Xn − npp

np(1− p)≤ a

!= P(Zn ≤ a) ;

Page 47: Probability and Statistics Book Solutions

29.1 Full solutions 503

see also (14.1). It follows from the central limit theorem that

P

X − nppnp(1− p)

≤ a

!≈ Φ(a),

i.e., the random variable X−np√np(1−p)

has a distribution that is approximately standard

normal.

14.6 a Using the result of Exercise 14.5 yields that

P(X ≤ 25) = P

X − 25p

75/4≤ 25− 25p

75/4

!= P

X − 25p

75/4≤ 0

!≈ P(Z ≤ 0)

=1

2,

where Z has a standard normal distribution. In the same way,

P(X < 26) = P

X − 25p

75/4≤ 26− 25p

75/4

!= P

X − 25p

75/4≤ 2

5√

3

!≈ P(Z ≤ 0.2309)

= 0.591.

14.6 b P(X ≤ 2) ≈ P(Z ≤ −5.31) ≈ 0 (the table ends with z = 3.49).

14.7 Setting Y = n1/4

σXn, one easily shows that E[Y ] = n1/4

σµ, and Var(Y ) =

1/√

n. Since

P

n 14

Xn − µ

σ

≥ a

= P(|Y − E[Y ]| ≥ a) ,

it follows from Chebyshev’s inequality that

P

n 14

Xn − µ

σ

≥ a

≤ 1

a2√

n.

As a consequence, if n goes to infinity, we see that most of the probability mass

of the random variable n14 Xn−µ

σis concentrated in the interval between −a and a,

for every a > 0. Since we can choose a arbitrarily small, this explains the spike inFigure 14.1.

14.8 a 1 = Var(Xi) = EX2

i

− (E[Xi])2 = E

X2

i

− 0, so EX2

i

= 1.

14.8 b (integration by parts: EX4

i

=R

x4 1√2π

e−12 x2

dx = −1√2π

[x3e−

12 x2

] − R 3x2e−12 x2

dx

=

3R

xe−12 x2

dx = 3EX2

i

= 3)

VarX2

i

= E

X4

i

− (EX2

i

)2 = 3− 1 = 2.

Page 48: Probability and Statistics Book Solutions

504 Full solutions from MIPS: DO NOT DISTRIBUTE

14.8 c P(Y100 > 110) ≈ PZ > 1√

2

≈ 0.242.

14.9 a The probability that for a chain of at least 50 meters more than 1002 linksare needed is the same as the probability that a chain of 1002 chains is shorter than50 meters. Assuming that the random variables X1, X2, . . . , X1002 are independent,and using the central limit theorem, we have that

P(X1 + X2 + · · ·+ X1002 < 5000) ≈ P

Z <

√1002 ·

50001002

− 5√0.04

= 0.0571,

where Z has an N(0, 1) distribution. So about 6% of the customers will receive afree chain.

14.9 b We now have that

P(X1 + X2 + · · ·+ X1002 < 5000) ≈ P(Z < 0.0032) ,

which is slightly larger than 1/2. So about half of the customers will receive a freechain. Clearly something has to be done: a seemingly minor change of expected valuehas major consequences!

14.10 a Note that

P|Mn − c| ≤ 0.5

= P

−√3

6

√n

Mn − c

σ<

√3

6

√n

≈ P

−√3

6

√n < Z <

√3

6

√n

= 1− 2P

Z ≥

√3

6

√n

,

where Z is N(0, 1). Now choose n such that 1 − 2PZ ≥

√3

6

√n

= 0.9, i.e., let n

be such that PZ ≥

√3

6

√n

= 0.05. Then√

36

√n = 2.75 yielding that n = 90.75.

Since n is an integer, we find n = 91.

14.10 b In case the Ui are normally distributed, the random variable (Mn − c)/σhas an N(0, 1) distribution, and the calculations in the answer of a are ‘exact’ (cf.Section 5.5). In all other cases this random variable has a distribution which is byapproximation equal to an N(0, 1) distribution (see also the discussion in Section 14.2on the size of n).

15.1 a For bin (32.5, 33.5], the height equals the number of xj in Bi divided byn|Bi|: 3/(5732 · 1) = 0.00052. Similar computations for the other bins give

bin height bin height

(32.5, 33.5] 0.00052 (40.5, 41.5] 0.16312(33.5, 34.5] 0.00331 (41.5, 42.5] 0.11270(34.5, 35.5] 0.01413 (42.5, 43.5] 0.05461(35.5, 36.5] 0.03297 (43.5, 44.5] 0.02931(36.5, 37.5] 0.07135 (44.5, 45.5] 0.00872(37.5, 38.5] 0.13137 (45.5, 46.5] 0.00314(38.5, 39.5] 0.18526 (46.5, 47.5] 0.00052(39.5, 40.5] 0.18876 (47.5, 48.5] 0.00017

Page 49: Probability and Statistics Book Solutions

29.1 Full solutions 505

15.1 b Symmetric, since the histogram has one mode with more or less the samedecay in both directions.

32 34 36 38 40 42 44 46 48 50

0

0.05

0.10

0.15

0.20

15.2 a For bin (50, 55], the height equals the number of xj in Bi divided by n|Bi|:1/(23 · 5) = 0.0087. The heights of the other bins can be computed similarly.

bin height

(50,55] 0.0087(55,60] 0.0174(60,65] 0.0087(65,70] 0.0870(70,75] 0.0348(75,80] 0.0348(80,85] 0.0087

15.2 b No, 31 degrees is extremely low compared to the temperature experiencedon earlier takeoffs.

15.3 a For bin (0, 250], the height equals the number of xj in Bi divided by n|Bi|:141/(190 · 250) = 0.00297. The heights of the other bins can be computed similarly.

Bin Height

(0,250] 0.00297(250,500] 0.00067(500,750] 0.00015(750,1000] 0.00008(1000,1250] 0.00002(1250,1500] 0.00004(1500,1750] 0.00004(1750,2000] 0(2250,2500] 0(2250,2500] 0.00002

Page 50: Probability and Statistics Book Solutions

506 Full solutions from MIPS: DO NOT DISTRIBUTE

15.3 b Skewed, since the histogram has one mode and only decays to the right.

0 500 1000 1500 2000 2500

0

0.001

0.002

0.003

15.4 a For bin (0, 500], the height equals the number of xj in Bi divided by n|Bi|:86/(135 ·500) = 0.0012741. The heights of the other bins can be computed similarly.

Bin Height

[0,500] 0.0012741(500,1000] 0.0003556(1000,1500] 0.0001778(1500,2000] 0.0000741(2000,2500] 0.0000148(2500,3000] 0.0000148(3000,3500] 0.0000296(3500,4000] 0(4000,4500] 0.0000148(4500,5000] 0(5000,5500] 0.0000148(5500,6000] 0.0000148(6000,6500] 0.0000148

15.4 b Since all elements (not rounded) are strictly positive, the value of Fn atzero is zero. At 500 the value of Fn is the number of xi ≤ 500 divided by n:86/135 = 0.6370. The values at the other endpoints can be computed similarly:

t Fn(t) t Fn(t)

0 0 3500 0.9704500 0.6370 4000 0.9704

1000 0.8148 4500 0.97781500 0.9037 5000 0.97782000 0.9407 5500 0.98522500 0.9481 6000 0.99263000 0.9556 6500 1

15.4 c The area under the histogram on bin (1000, 1500] is 500 ·0.000178 = 0.0889,and Fn(1500)− Fn(1000) = 0.9037− 0.8148 = 0.0889.

Page 51: Probability and Statistics Book Solutions

29.1 Full solutions 507

15.5 Since the increase of Fn on bin (0, 1] is equal to the area under the histogramon the same bin, the height on this bin must be 0.2250/(1− 0) = 0.2250. Similarly,the height on bin (1, 3] is (0.445-0.225)/(3-1)=0.1100, and so on:

Bin Height

(0, 1] 0.2250(1, 3] 0.1100(3, 5] 0.0850(5, 8] 0.0400(8, 11] 0.0230(11, 14] 0.0350(14, 18] 0.0225

15.6 Because (2 − 0) · 0.245 + (4 − 2) · 0.130 + (7 − 4) · 0.050 + (11 − 7) · 0.020 +(15− 11) · 0.005 = 1, there are no data points outside the listed bins. Hence

Fn(7) =number of xi ≤ 7

n

=number of xi in bins (0, 2], (2, 4] and (4, 7]

n

=n · (2− 0) · 0.245 + n · (4− 2) · 0.130 + n · (7− 4) · 0.050

n= 0.490 + 0.260 + 0.150 = 0.9.

15.7 Note that the increase of Fn on each bin is equal to the area under thehistogram on the same bin. Since all bins have the same width, it follows from theresults of Exercise 15.2 that the increase of Fn is the largest (5 · 0.0.870 = 0.435) on(65, 70].

15.8 K does not satisfy (K1), since it is negative between 0 and 1 and does notintegrate to one.

15.9 a The scatterplot indicates that larger durations correspond to longer waitingtimes.

15.9 b By judgement by the eye, the authors predicted a waiting time of about 80.

15.9 c If you project the points on the vertical axes you get the dataset of waitingtimes. Since the two groups of points in the scatterplot are separated in North Eastdirection (both vertical as well as horizontal), you will see two modes.

15.10 Each steep part of Fn corresponds to a mode of the dataset. One counts fourof such parts.

15.11 The height of the histogram on a bin (a, b] is

number of xi in (a, b]

n(b− a)=

(number of xi ≤ b) – (number of xi ≤ a)

n(b− a)

=Fn(b)− Fn(a)

b− a.

Page 52: Probability and Statistics Book Solutions

508 Full solutions from MIPS: DO NOT DISTRIBUTE

15.12 a By inserting the expression for fn,h(t), we getZ ∞

−∞t · fn,h(t) dt =

Z ∞

−∞t · 1

nh

nXi=1

K

t− xi

h

dt

=1

n

nXi=1

Z ∞

−∞

t

hK

t− xi

h

dt.

For each i fixed we find with change of integration variables u = (t− xi)/h,Z ∞

−∞

t

hK

t− xi

h

dt =

Z ∞

−∞(xi + hu)K (u) du

= xi

Z ∞

−∞K (u) du + h

Z ∞

−∞uK (u) du = xi,

using that K integrates to one and thatR∞−∞ uK (u) du = 0, because K is symmetric.

Hence Z ∞

−∞t · fn,h(t) dt =

1

n

nXi=1

Z ∞

−∞

t

hK

t− xi

h

dt =

1

n

nXi=1

xi.

15.12 b By means of a similar reasoningZ ∞

−∞t2 · fn,h(t) dt =

Z ∞

−∞t2 · 1

nh

nXi=1

K

t− xi

h

dt

=1

n

nXi=1

Z ∞

−∞

t2

hK

t− xi

h

dt.

For each i:Z ∞

−∞

t2

hK

t− xi

h

dt

=

Z ∞

−∞(xi + hu)2K (u) du =

Z ∞

−∞(x2

i + 2xihu + h2u2)K (u) du

= x2i

Z ∞

−∞K (u) du + 2xih

Z ∞

−∞uK (u) du + h2

Z ∞

−∞u2K (u) du

= x2i + h2

Z ∞

−∞u2K (u) du,

again using that K integrates to one and that K is symmetric.

16.1 a The dataset has 135 elements, so the sample median is the 68th element inorder of magnitude. From the table in Exercise 15.4 we see that this is 290.

16.1 b The dataset has n = 135 elements. The lower quartile is the 25th empiricalpercentile. We have k = b0.25 · (135 + 1)c = 34, so that α = 0, and qn(0.25) =x(34) = 81, according to the table in Exercise 15.4. Similarly, the upper quartile isqn(0.75) = x(102) = 843, and the IQR is 843− 81 = 762.

16.1 c We have k = b0.37 · (135 + 1)c = b50.32c = 50, so that α = 0.32, and

qn(0.37) = x(50) + 0.32 · (x(51) − x(50)) = 143 + 0.32 · (148− 143) = 144.6.

Page 53: Probability and Statistics Book Solutions

29.1 Full solutions 509

16.2 To compute the lower quartile, we use that n = 272, so that k = b0.25 ·273c =b68.25c = 68 and α = 0.25, and hence from Table 15.2:

qn(0.25) = x(68) + 0.25 · (x(69) − x(68)) = 129 + 0.25 · (130− 129) = 129.25.

Similarly, qn(0.5) = 240 and qn(0.75) = 267.75. Hence, |qn(0.75)− qn(0.5)| = 27.75and |qn(0.25)− qn(0.5)| = 110.75. The kernel density estimate (see Figure 15.3) hasa ‘peak’ just right of 240, so there is relatively a lot of probability mass just rightof the sample median, namely about 25% between 240 and 110.75.

16.3 a Because n = 24, the sample median is the average of the 12th and 13thelement. Since these are both equal to 70, the sample median is also 70. The lowerquartile is the pth empirical quantile for p = 1/4. We get k = bp(n + 1)c = b0.25 ·(24 + 1)c = b6.25c = 6, so that

qn(0.25) = x(6) + 0.25 · (x(7) − x(6)) = 66 + 0.25 · (67− 66) = 66.25.

Similarly, the upper quartile is the pth empirical quantile for p = 3/4:

qn(0.75) = x(18) + 0.75 · (x(19) − x(18)) = 75 + 0.75 · (75− 75) = 75.

16.3 b In part a we found the sample median and the two quartiles. From this wecompute the IQR: qn(0.75)− qn(0.25) = 75− 66.25 = 8.75. This means that

qn(0.25)− 1.5 · IQR = 66.25− 1.5 · 8.75 = 53.125,

qn(0.75) + 1.5 · IQR = 75 + 1.5 · 8.75 = 88.125.

Hence, the last element below 88.125 is 88, and the first element above 53.125 is 57.Therefore, the upper whisker runs until 88 and the lower whisker until 57, with twoelements 53 and 31 below. This leads to the following boxplot:

31

57

66.257075

81

16.3 c The values 53 and 31 are outliers. Value 31 is far away from the bulk of thedata and appears to be an extreme outlier.

16.4 a Check that yn = 700/99 and xn = 492/11, so that

5

9(xn − 32) =

5

9

492

11− 32

=

700

99.

16.4 b Since n = 11, for both datasets the sample median is 6th element in order ofmagnitude. Check that Med(y1, . . . , yn) = 50/9 and Med(x1, . . . , xn) = 42, so that

5

9(Med(x1, . . . , xn)− 32) =

5

9(42− 32) =

50

9,

which is equal to Med(y1, . . . , yn).

Page 54: Probability and Statistics Book Solutions

510 Full solutions from MIPS: DO NOT DISTRIBUTE

16.4 c For any real number a, we have

yn =1

n

nXi=1

yi =1

n

nXi=1

(axi + b) = a

1

n

nXi=1

xi

!+ b = axn + b.

For a ≥ 0, the order of magnitude of yi = axi + b is the same as that of the xi’s,and therefore

Med(y1, . . . , yn) = aMed(x1, . . . , xn) + b.

When a < 0, the order of yi = axi + b is reverse to that of the xi’s, but the positionof the middle number in order of magnitude (or average of the two middle numbers)remains the same, so that the above rule still holds.

16.5 a Check that the sample variance of the yi’s is (sC)2 = 132550/992, so that5

9

2

(sF )2 =

5

9

2482

11=

132550

992,

which is equal to (sC)2, and hence sC = 59sF .

16.5 b For the xi’s we have Med(x1, . . . , xn) = 42. This leads to the following table

xi 43 43 41 41 41 42 43 58 58 41 41|xi − 42| 1 1 1 1 1 0 1 16 16 1 1|xi − 42| ordered 0 1 1 1 1 1 1 1 1 16 16

so that MADF = Med|xi − 42| = 1. Similarly, we have Med(y1, . . . , yn) = 509

, and

yi559

559

5 5 5 509

559

1309

1309

5 5

|yi − 509| 5

959

59

59

59

0 59

809

809

59

59

|yi − 509| ordered 0 5

959

59

59

59

59

59

59

809

809

so that MADC = Med|yi − 509| = 5

9. Therefore, MADC = 5

9MADF .

16.5 c Since, yn = axn + b, for any real number a we have for the sample varianceof the yi’s

s2Y =

1

n− 1

nXi=1

(yi − yn)2 =1

n− 1

nXi=1

((axi + b)− (axn + b))2

= a2 1

n− 1

nXi=1

(xi − xn)2 = a2s2X

,

so that sY = |a| sX . Because MedY = aMedX + b, it follows that

MADY = Med|yi −MedY | = Med|(axi + b)− (aMedX + b)|= |a|Med|xi −MedX | = |a|MADX .

16.6 a Yes, we find x = (1+5+9)/3 = 15/3 = 5, y = (2+4+6+8)/4 = 20/4 = 5,so that (x + y)/2 = 5. The average for the combined dataset is also equal to 5:(15 + 20)/7 = 5.

Page 55: Probability and Statistics Book Solutions

29.1 Full solutions 511

16.6 b The mean of x1, x2, . . . , xn, y1, y2, . . . , ym equals

x1 + · · ·+ xn + y1 + · · ·+ ym

n + m=

nxn + mym

n + m=

n

n + mxn +

m

n + mym.

In general, this is not equal to (xn+ym)/2. For instance, replace 1 in the first datasetby 4. Then xn = 6 and ym = 5, so that (xn + ym)/2 = 5 1

2. However, the average of

the combined dataset is 38/7 = 5 27.

16.6 c Yes, m = n implies n/(n + m) = m/(n + m) = 1/2. From the expressionsfound in part b we see that the sample mean of the combined dataset equals (xn +ym)/2.

16.7 a We have Medx = 5 and Medy = (4 + 6)/2 = 5, whereas for the combineddataset

1 2 4 5 6 8 9

the sample median is the fourth number in order of magnitude: 5.

16.7 b This will not be true in general: take 1, 2, 3 and 5, 7 with sample medians 2and 6. The combined dataset has sample median 3, whereas (2 + 6)/2 = 4.

16.7 c This will not be true in general: take 1, 9 and 2, 4 with sample medians 5and 3. The combined dataset has sample median 3, whereas (5 + 3)/2 = 4.

16.8 The ordered combined dataset is 1, 2, 4, 5, 6, 8, 9, so that the sample medianequals 5. The absolute deviations from 5 are: 4, 3, 1, 0, 1, 3, 4, and if we put them inorder: 0, 1, 1, 3, 3, 4, 4. The MAD is the sample median of the absolute deviations,which is 3.

16.9 a One can easily check that xn = 10/3. The average of the yi’s:

1

3

−1

6+ 1 +

1

15

=

1

3· 54

60=

3

10,

so that y7 = 1/x7.

16.9 b No, for instance take 1, 2, and 3. Then xn = 2, and yn = (1 + 12

+ 13)/3 =

11/18.

16.9 c First note that the transformation y = 1/x puts the yi’s in reverse ordercompared to the xi’s. If n is odd, then the middle element in order of magnituderemains the middle element under the transformation y = 1/x. Therefore, if n isodd, the answer is yes. If n is even, the answer is no, for example take 1 and 2. ThenMedX = 3

2and MedY = Med(1, 3

2) = (1 + 3

2)/2 = 5

4.

16.10 a The sum of the elements is 16.8 + y which goes to infinity as y →∞, andtherefore the sample mean also goes to infinity, because we only divide the sum bythe sample size n. When y →∞, the ordered data will be

3.0 4.2 4.6 5.0 y

Therefore, no matter how large y gets, the sample median, which is the middlenumber, remains 4.6.

16.10 b In order to let the middle number go to infinity, we must replace at leastthree numbers. For instance, replace 3.2, 4.2, and 5.0 by some real number y thatgoes to infinity. In that case, the ordered data will be

3.0 4.6 y y y

of which the middle number y goes to infinity.

Page 56: Probability and Statistics Book Solutions

512 Full solutions from MIPS: DO NOT DISTRIBUTE

16.10 c Without loss of generality suppose that the dataset is already ordered. Bythe same reasoning as in part a, one can argue that we only have to replace oneelement in order to let the sample mean go to infinity.When n = 2k + 1 is odd, the middle number is the (k + 1)st number. By the samereasoning as in part b one can argue that in order to let that go to infinity, we mustreplace the middle number, as well as k other elements by y, which means we haveto replace k + 1 = (n− 1)/2 = b(n + 1)/2c elements.When n = 2k is even, the middle number is the average of the kth and (k + 1)stnumber. In order to let that go to infinity, it suffices to replace k elements, thatinclude either the kth or the (k + 1)st number, by y, This means we have to replacek = n/2 = b(n + 1)/2c elements.

16.11 a. From Exercise 16.10 we already know that the sample mean goes toinfinity. This implies that (4.6 − xn)2 also goes to infinity and therefore also thesample variance, as well as the sample standard deviation.From Exercise 16.10 we know that the sample median remains 4.6. Hence, the or-dered absolute deviations are

0 0.4 0.4 1.6 |y − 4.6|Therefore, no matter how large y gets, the MAD, which is the middle number of theordered absolute deviations, remains 0.4.b. In order to let the middle number the ordered absolute deviations go to infinity,we must at least replace three numbers. For instance, replace 3.2 and 4.2 by y and5.0 by −y, where y is some real number that goes to infinity. In that case, the ordereddata will be

−y 3.0 4.6 y y

of which the middle number is 4.6. The ordered absolute deviations are

0 0.4 |y − 4.6| |y − 4.6| |4.6 + y|The MAD is the middle number, which goes to infinity.c. Without loss of generality suppose that the dataset is already ordered. By thesame reasoning as in part a we only need to replace one element in order to let thesample standard deviation go to infinity.When n = 2k + 1 is odd, the middle number is the (k + 1)st number. By the samereasoning as in part b one can argue that in order to let the MAD go to infinity, wemust replace the middle number by y, as well as k other numbers. This means wehave to replace k + 1 = (n − 1)/2 = b(n + 1)/2c elements, for instance one by −yand the rest by y. In that case, the sample median remains finite and the majorityof the absolute deviations from the median tends to infinity.When n = 2k is even, the middle number is the average of the kth and (k + 1)stnumber. In order to let the MAD go to infinity, it suffices to replace k elements, thatinclude either the kth or the (k + 1)st number, by y. This means we have to replacek = n/2 = b(n + 1)/2c elements, for instance one by −y and the rest by y. In thatcase, the sample median remains finite and half of the absolute deviations from themedian tends to infinity.

16.12 The sample mean is

1

N(1 + 2 + · · ·+ N) =

1

N· N(N + 1)

2=

N + 1

2

Page 57: Probability and Statistics Book Solutions

29.1 Full solutions 513

When N = 2k + 1 is odd, the sample median is the (k + 1)st number, which isk + 1 = (N − 1)/2 + 1 = (N + 1)/2. When N = 2k is even, the sample median isthe average of the kth and (k + 1)st number:

(N/2 + (N/2 + 1))

2=

N + 1

2.

16.13 First note that the sample mean is zero, and n = 2N + 1. Therefore thesample variance is

s2n =

1

n− 1

(−N)2 + · · ·+ (−1)2 + 02 + 12 + · · ·N2

=1

2N· 2 · 12 + · · ·N2 =

1

N· N(N + 1)(2N + 1)

6=

(N + 1)(2N + 1)

6,

so that sn =q

16(N + 1)(2N + 1). Since n = 2N + 1 is always odd, the sample

median is the middle number, which is zero. Therefore we have the following

xi −N · · · −1 0 1 · · · N|xi| N · · · 1 0 1 · · · N|xi| ordered 0 1 1 · · · · · · N N

Since n = 2N + 1, which is odd, MAD = Med|xi| is the (N + 1)st number in thebottom row: (N + 1)/2.

16.14 When n = 2i + 1 is odd, then in the formula qn( 12) = x(k) + α[x(k+1) − x(k)]

for the 50th empirical percentile, we find k = b 12(n+1)c = bi+1c = i+1 and α = 0,

so that qn( 12) = x(i+1), which is the middle number in order of magnitude.

When n = 2i is even, we find k = b 12(n + 1)c = bi + 1

2c = i and α = 1

2, so that

qn( 12) = x(i) + 1

2

x(i+1) − x(i)

=

x(i+1)+x(i)2

,

which is the average of the two middle numbers in order of magnitude.

16.15 First write

1

n

nXi=1

(xi − xn)2 =1

n

nXi=1

x2

i − 2xnxi + x2n

=

1

n

nXi=1

x2i − 2xn

1

n

nXi=1

xi +1

n

nXi=1

x2n.

Next, by inserting

1

n

nXi=1

xi = xn and1

n

nXi=1

x2n =

1

n· n · x2

n = x2n,

we find1

n

nXi=1

(xi − xn)2 =1

n

nXi=1

x2i − 2x2

n + x2n =

1

n

nXi=1

x2i − x2

n.

16.16 According to Exercise 15.12Z ∞

−∞tfn,h(t) dt = xn

and

Page 58: Probability and Statistics Book Solutions

514 Full solutions from MIPS: DO NOT DISTRIBUTEZ ∞

−∞t2fn,h(t) dt =

1

n

nXi=1

x2i + h2

Z ∞

−∞u2K(u) du.

Therefore by the rule for the variance, Var(X) = EX2 − (E[X])2, the variance

corresponding to fn,h isZ ∞

−∞t2fn,h(t) dt−

Z ∞

−∞tfn,h(t) dt

2

=

Z ∞

−∞t2fn,h(t) dt− (xn)2

=1

n

nXi=1

x2i − (xn)2 + h2

Z ∞

−∞u2K(u) du

=1

n

nXi=1

(xi − xn)2 + h2

Z ∞

−∞u2K(u) du,

according to Exercise 16.15.

16.17 For p = i/(n + 1) we find that

k = bp(n + 1)c = i and α = p(n + 1)− k = 0,

so thatqn(p) = x(k) + α

x(k+1) − x(k)

= x(i).

17.1 Before one starts looking at the figures, it is useful to recall a couple featuresof the two distributions involved:

1. most of the probability mass of the N(0, 1) distribution lies between ±3, andbetween ±9 for the N(0, 9) distribution.

2. the height at zero of the exponential density is λ and the median of the Exp(λ)is ln(2)/λ (see Exercise 5.11).

We then have the following:

1. N(3, 1), since the mode is about 3.2. N(0, 1), since Fn is 0.5 at about zero and all elements are between about ±2.3. N(0, 1), since the mode is about zero and all elements are between about −4

and 3.4. N(3, 1), since the mode is about 3.5. Exp(1/3), since the sample median is about 2.6. Exp(1), since the sample median is less than 1.7. N(0, 1), since the mode is about zero and all elements are between about ±4.8. N(0, 9), since the mode is about zero, and all elements are between about −6

and 9.9. Exp(1), since the height of the histogram at zero is about 0.7. Moreover, almost

all elements are less than 6, whereas the probability of exceeding 6 for theExp(1/3) distribution is 0.135.

10. N(3, 1), since Fn is 0.5 at about 3.11. N(0, 9), since the mode is about zero and all elements are between about ±12.12. Exp(1/3), since the height of the histogram at zero is about 0.24. Moreover,

there are several elements beyond 10, which has probability 0.000045 for theExp(1) distribution.

13. N(0, 9), since the mode is about zero and all elements are between about −5and 8.

Page 59: Probability and Statistics Book Solutions

29.1 Full solutions 515

14. Exp(1/3), since the height of the kernel density estimate at zero is about 0.35.15. Exp(1), since the sample median is about 1.

17.2 We continue as in Exercise 17.1:

1. Exp(1/3), since the sample median is about 2.2. N(0, 9), since the sample median is about zero and all elements are between

about −6 and 8.3. Exp(1/3), since the sample median is about 2.4. N(0, 1), since the sample median is about zero and all elements are between

about −2 and 3.5. N(3, 1), since the sample median is about 3.6. Exp(1), since the sample median is less than one.7. N(0, 9), since the sample median is about zero and all elements are between

about −9 and 7.8. N(0, 9), since the sample median is about zero and all elements are between

about −7 and 9.9. N(3, 1), since the sample median is about 3.

10. Exp(1), since the sample median is less than one.11. N(3, 1), since the sample median is about 3.12. Exp(1), since the sample median is less than one.13. N(0, 1), since the sample median is about zero and all elements are between

about ±3.14. N(0, 1), since the sample median is about zero and all elements are between

about −3 and 4.15. Exp(1/3), since the sample median is about 2.

17.3 a The model distribution corresponds to the number of women in a queue. Aqueue has 10 positions. The occurrence of a woman in any position is independentof the occurrence of a woman in other positions. At each position a woman occurswith probability p. Counting the occurrence of a woman as a “success,” the numberof women in a queue corresponds to the number of successes in 10 independentexperiments with probability p of success and is therefore modeled by a Bin(10, p)distribution.

17.3 b We have 100 queues and the number of women xi in the ith queue is arealization of a Bin(10, p) random variable. Hence, according to Table 17.2, theaverage number of women x100 resembles the expectation 10p of the Bin(10, p)distribution. We find x100 = 435/100 = 4.35, so an estimate for p is 4.35/10 = 0.435.

17.4 a Recall that the parameter µ is the expectation of the Pois(µ) distribution.Hence, according to Table 17.2 the sample mean seems a reasonable estimate. Since,the dataset contains 229 zero’s, 211 ones, etc.,

xn =211 · 1 + 93 · 2 + 35 · 3 + 7 · 4 + 1 · 7

576=

537

576= 0.9323.

17.4 b From part a the parameter µ = 0.9323. Then P(X = 0) = e−0.9323 = 0.393,and the other Poisson probabilities can be computed similarly. The following tablecompares the relative frequencies to the Poisson probabilities.

hits 0 1 2 3 4 5 6 7

rel.freq. 0.397 0.367 0.162 0.06 0.012 0.0000 0.0000 0.0017prob. 0.393 0.367 0.171 0.053 0.012 0.0023 0.0004 0.0000

Page 60: Probability and Statistics Book Solutions

516 Full solutions from MIPS: DO NOT DISTRIBUTE

17.5 a One possibility is to use the fact that the geometric distribution has expec-tation 1/p, so that the sample mean will be close to 1/p. From the average numberof cycles needed, 331/93, we then estimate p by 93/331 = 0.2809.Another possibility is to use that fact that p is equal to the probability of gettingpregnant in the first cycle. We can estimate this by the relative frequency of womenthat got pregnant in the first cycle: 29/93 = 0.3118.

17.5 b From the average number of cycles needed, 1285/474, we could estimate pby 474/1285 = 0.3689. Another possibility is to estimate p by the relative frequencyof women that got pregnant in the first cycle: 198/474 = 0.4177.

17.5 c

p p + p(1− p) + p(1− p)2

smokers 0.2809 0.62810.3118 0.6741

nonsmokers 0.3689 0.74860.4177 0.8026

17.6 a The parameters µ and σ are the expectation and standard deviation of theN(µ, σ2) distribution. Hence, according to Table 17.2 we can estimate µ by thesample mean:

µ =1

n

nXi=1

xi =2283772

5732= 39.84,

and σ by the sample standard deviation sn. To compute this, use Exercise 16.15 toget

s2n =

1

n− 1

nXi=1

(xi − xn)2 =n

n− 1

"1

n

nXi=1

(xi − xn)2#

=n

n− 1

" 1

n

nXi=1

x2i

!2

− (xn)2#

=5732

5731

9124064

5732− (39.8)2

= 4.35.

Hence, take sn =√

4.35 = 2.09 as an estimate for σ.

17.6 b Since we model the chest circumference by means of a random variable Xwith a N(µ, σ2) distribution, the required probability is equal to

P(38.5 < X < 42.5) = P(X < 42.5)− P(X < 38.5)

= P

Z <

42.5− µ

σ

− P

Z <

38.5− µ

σ

= Φ

42.5− µ

σ

− Φ

38.5− µ

σ

where Z = (X − µ)/σ has a standard normal distribution. We can estimate this byplugging in the estimates 39.84 and 2.09 for µ and σ. This gives

Φ (1.27)− Φ(−0.64) = 0.8980− 0.2611 = 0.6369.

Another possibility is to estimate P(38.5 < X < 42.5) by the relative frequency ofchest circumferences between 38.5 and 42.5. Using the information in Exercise 15.1we find 3725/5732 = 0.6499.

Page 61: Probability and Statistics Book Solutions

29.1 Full solutions 517

17.7 a If we model the series of disasters by a Poisson process, then as a property ofthe Poisson process, the interdisaster times should follow an exponential distribution(see Section 12.3). This is indeed confirmed by the histogram and empirical distri-bution of the observed interdisaster times; they resemble the probability density anddistribution function of an exponential distribution.

17.7 b The average length of a time interval is 40 549/190 = 213.4 days. FollowingTable 17.2 this should resemble the expectation of the Exp(λ) distribution, whichis 1/λ. Hence, as an estimate for λ we could take 190/40 549 = 0.00469.

17.8 a The distribution function of Y is given by

FY (a) = P(Y ≤ a) = PX ≤ y1/α

= 1− eλαy.

This is the distribution function of the Exp(λα) distribution, which has expectation1/λα. Therefore, E[Xα] = E[Y ] = 1/λα.

17.8 b Take α-powers of the data: yi = xαi . Then, according to part a, the yi’s

are a realization of a sample from an Exp(λα) distribution, with expectation 1/λα.Hence, 1/λα can be estimated by the average of these numbers:

1

λα≈ 1

n

nXi=1

xαi .

Next, solve for λ to get an estimate for λ:

λ =

1

n

nXi=1

xαi

!−1/α

.

When we plug in α = 2.102 and apply this formula to the dataset, we get λ =(10654.85)−1/2.102 = 0.0121.

17.9 a A (perfect) cylindrical cone with diameter d (at the base) and height h hasvolume πd2h/12, or about 0.26d2h. The effective wood of a tree is the trunk withoutthe branches. Since the trunk is similar to a cylindrical cone, one can expect a linearrelation between the effective wood and d2h.

17.9 b We find

zn =

Pyi/xi

n=

9.369

31= 0.3022

y/x =(P

yi)/n

(P

xi)/n=

26.486/31

87.456/31= 0.3028

least squares =

PxiyiPx2

i

=95.498

314.644= 0.3035.

17.10 a

G(y) = P(Y ≤ y)

= P(|X −m| ≤ y)

= P(m− y ≤ X ≤ m + y)

= P(X ≤ m + y)− P(X ≤ m− y)

= F (m + y)− F (m− y).

Page 62: Probability and Statistics Book Solutions

518 Full solutions from MIPS: DO NOT DISTRIBUTE

17.10 b If f is symmetric around its median, then

f(m− y) = f(m + y)

for all y. This implies that the area under f on (−∞, m−y] is equal to the area underf on [m+y,∞), i.e. F (m−y) = 1−F (m+y). By a this means G(y) = 2F (m+y)−1.To derive Ginv( 1

2) put 1

2= G(y). Then, since m = F inv( 1

2)

1

2= 2F (m + y)− 1 ⇔ 3

4= F (m + y)

⇔ y = F inv( 34)−m = F inv( 3

4)− F inv( 1

2).

17.10 c First determine an expression for F inv(x), by putting x = F (u). Then

F inv(x) = µ + σΦinv(x).

With b it follows that the MAD of a N(µ, σ2) is equal to

F inv( 34)− F inv( 1

2) = σΦinv( 3

4)− σΦinv( 1

2) = σΦinv( 3

4),

using that Φinv( 12) = 0.

For the N(5, 4) distribution the MAD is 2Φinv( 34) = 2 · 0.6745 = 1.3490.

17.11 a Using Exercise 17.10 a it follows that

G(y) = F (m + y)− F (m− y)

=1− e−λ(m−y)

−1− e−λ(m+y)

= e−λm

eλy − e−λy

=

1

2

eλy − e−λy

,

using that m is the median of the Exp(λ) distribution, and satisfies 1− e−λm = 12.

17.11 b Combining a with Exercise 17.10b the MAD of the Exp(λ) distribution isa solution of G(y) = 1

2, so that 1

2

eλy − e−λy

= 1

2. Multiplying this equation with

eλy, yields that the MAD must satisfy e2λy − eλy − 1 = 0.

17.11 c Put x = eλy and solve x2 − x− 1 = 0 for x. This gives

x =1±√5

2.

Since the MAD must be positive, it can be found from the relation

eλy =1 +

√5

2.

If follows that the MAD of the Exp(λ) distribution is equal to

ln(1 +√

5)− ln 2

λ=

ln(1 +√

5)

λ− F inv( 1

2).

18.1 If we view a bootstrap dataset as a vector (x∗1, . . . , x∗n), we have five possibil-

ities 1, 2, 3, 4, and 6 for each of the six positions. Therefore there are 56 differentvectors possible. They are not equally likely. For instance, (1, 1, 1, 1, 1, 1) has prob-ability ( 1

3)6, whereas (2, 2, 2, 2, 2, 2) has probability ( 1

6)6 to occur.

Page 63: Probability and Statistics Book Solutions

29.1 Full solutions 519

18.2 a Because the bootstrap sample can only be 1 if and only if all elements in thebootstrap random sample are 1, we get:

PX∗

n = 1

= P(X∗1 = 1, . . . , X∗

4 = 1)

= P(X∗1 = 1) · · ·P(X∗

4 = 1) =

1

4

4

= 0.0039.

18.2 b Because the maximum is less than 4, is equivalent to all numbers being lessthan 4, we get

P(max X∗i = 6) = 1− P(max X∗

i ≤ 4) = 1− P(X∗1 ≤ 4, . . . , X∗

4 ≤ 4)

= 1− P(X∗1 ≤ 4) · · ·P(X∗

4 ≤ 4) = 1−

3

4

4

= 0.6836

18.3 a Note that generating from the empirical distribution function is the same aschoosing one of the elements of the original dataset with equal probability. Hence,an element in the bootstrap dataset equals 0.35 with probability 0.1. The numberof ways to have exactly three out of ten elements equal to 0.35 is

103

, and each has

probability (0.1)3(0.9)7. Therefore, the probability that the bootstrap dataset hasexactly three elements equal to 0.35 is equal to

103

(0.1)3(0.9)7 = 0.0574.

18.3 b Having at most two elements less than or equal to 0.38 means that 0, 1,or 2 elements are less than or equal to 0.38. Five elements of the original datasetare smaller than or equal to 0.38, so that an element in the bootstrap dataset isless than or equal to 0.38 with probability 0.5. Hence, the probability that thebootstrap dataset has at most two elements less than or equal to 0.38 is equal to(0.5)10 +

101

(0.5)10 +

102

(0.5)10 = 0.0547.

18.3 c Five elements of the dataset are smaller than or equal to 0.38 and twoare greater than 0.42. Therefore, obtaining a bootstrap dataset with two elementsless than or equal to 0.38, and the other elements greater than 0.42 has probabil-ity (0.5)2 (0.2)8. The number of such bootstrap datasets is

102

. So the answer is

102

(0.5)2 (0.2)8 = 0.000029.

18.4 a There are 9 elements strictly less than 0.46, so that P(M∗10 < 0.46) =

( 910

)10 = 0.3487.

18.4 b There are n − 1 elements strictly less than mn, so that P(M∗n < mn) =

(n−1n

)n = (1− 1/n)n.

18.5 Since each X∗i is either 0, 3, or 6, it is not so difficult to see that X∗

1 +X∗2 +X∗

3

can only take the values 0, 3, 6, 9, 12, 15, and 18. The sample mean of thedataset is xn = 3. Therefore X∗

3 − xn = (X∗1 + X∗

2 + X∗3 )/3 − 3 can only take

the values −3, −2, −1, 0, 1, 2, and 3. The value X∗3 − xn = −3 corresponds to

(X∗1 , X∗

2 , X∗3 ) = (0, 0, 0). Because there are 33 = 27 possibilities for (X∗

1 , X∗2 , X∗

3 ),the probability P

X∗

3 − xn = −3

= 1/27. Similarly, the value X∗3 − xn = −2

corresponds to (X∗1 , X∗

2 , X∗3 ) being equal to (3, 0, 0), (0, 3, 0), or (0, 0, 3), so that

PX∗

3 − xn = −2

= 3/27. The other probabilities can be computed in the sameway, which leads to

a −3 −2 −1 0 1 2 3

PX∗

n − xn = a 1

27

3

27

6

27

7

27

6

27

3

27

1

27

Page 64: Probability and Statistics Book Solutions

520 Full solutions from MIPS: DO NOT DISTRIBUTE

18.6 a We have

Fλ(m) = 12⇔ 1− e−λm = 1

2⇔ −λm = ln( 1

2) = − ln(2) ⇔ m =

ln 2

λ.

18.6 b Since we know that the dataset is a realization of a sample from an Exp(λ)distribution, we are dealing with a parametric bootstrap. Therefore we must generatethe bootstrap datasets from the distribution function of the Exp(λ) distribution withthe parameter λ estimated by 1/xn = 0.2. Because the bootstrap simulation is forMed(X1, X2, . . . , Xn)−mλ, in each iteration we must compute Med(x∗1, x

∗2, . . . , x

∗n)−

m∗, where m∗ denotes the median of the estimated exponential distribution: m∗ =ln(2)/(1/xn) = 5 ln(2). This leads to the following parametric bootstrap simulationprocedure:

1. Generate a bootstrap dataset x∗1, x∗2, . . . , x

∗n from F (x) = 1− e−0.2x.

2. Compute the centered sample median for the bootstrap dataset:

Med(x∗1, x∗2, . . . , x

∗n)− 5 ln(2),

where Med(x∗1, x∗2, . . . , x

∗n) is the sample median of x∗1, x

∗2, . . . , x

∗n.

Repeat steps 1 and 2 many times.

18.7 For the parametric bootstrap, we must estimate the parameter θ by θ =(n + 1)mn/n, and generate bootstrap samples from the U(0, θ) distribution. Thisdistribution has expectation µθ = θ/2 = (n +1)mn/(2n). Hence, for each bootstrapsample x∗1, x

∗2, . . . , x

∗n compute x∗n − µθ = x∗n − (n + 1)mn/(2n).

Note that this is different from the empirical bootstrap simulation, where one wouldestimate µ by xn and compute x∗n − xn.

18.8 a Since we know nothing about the distribution of the interfailure times, weestimate F by the empirical distribution function Fn of the software data and weestimate the expectation µ of F by the expectation µ∗ = xn = 656.8815 of Fn.The bootstrapped centered sample mean is the random variable X∗

n−656.8815. Thecorresponding empirical bootstrap simulation is described as follows:

1. Generate a bootstrap dataset x∗1, x∗2, . . . , x

∗n from Fn, i.e., draw with replacement

135 numbers from the software data.2. Compute the centered sample mean for the bootstrap dataset:

x∗n − 656.8815

where x∗n is the sample mean of x∗1, x∗2, . . . , x

∗n.

Repeat steps 1 and 2 one thousand times.

18.8 b Because the interfailure times are now assumed to have an Exp(λ) distribu-tion, we must estimate λ by λ = 1/xn = 0.0015 and estimate F by the distributionfunction of the Exp(0.0015) distribution. Estimate the expectation µ = 1/λ of theExp(λ) distribution by µ∗ = 1/λ = xn = 656.8815. Also now, the bootstrappedcentered sample mean is the random variable X∗

n − 656.8815. The correspondingparametric bootstrap simulation is described as follows:

1. Generate a bootstrap dataset x∗1, x∗2, . . . , x

∗n from the Exp(0.0015) distribution.

Page 65: Probability and Statistics Book Solutions

29.1 Full solutions 521

2. Compute the centered sample mean for the bootstrap dataset:

x∗n − 656.8815,

where x∗n is the sample mean of x∗1, x∗2, . . . , x

∗n.

Repeat steps 1 and 2 one thousand times. We see that in this simulation the boot-strapped centered sample mean is the same in both cases: X∗

n − xn, but the corre-sponding simulation procedures differ in step 1.

18.8 c Estimate λ by λ = ln 2/mn = 0.0024 and estimate F by the distributionfunction of the Exp(0.0024) distribution. Estimate the expectation µ = 1/λ of theExp(λ) distribution by µ∗ = 1/λ = 418.3816. The corresponding parametric boot-strap simulation is described as follows:

1. Generate a bootstrap dataset x∗1, x∗2, . . . , x

∗n from the Exp(0.0024) distribution.

2. Compute the centered sample mean for the bootstrap dataset:

x∗n − 418.3816,

where x∗n is the sample mean of x∗1, x∗2, . . . , x

∗n.

Repeat steps 1 and 2 one thousand times. We see that in this parametric bootstrapsimulation the bootstrapped centered sample mean is different from the one in theempirical bootstrap simulation: X∗

n − (ln 2)/mn instead of X∗n − xn.

18.9 Estimate µ by xn = 39.85, σ by sn = 2.09, and estimate F by the distributionfunction of the N(39.85, 4.37) distribution.

1. Generate a bootstrap dataset x∗1, x∗2, . . . , x

∗n from the N(39.85, 4.37) distribution.

2. Compute the centered sample mean for the bootstrap dataset:

x∗n − 39.85

where xn is the sample mean of x∗1, x∗2, . . . , x

∗n.

Repeat steps 1 and 2 one thousand times. Estimate P|Xn − µ| > 1

by the relative

frequency of bootstrapped centered sample means that are greater than 1 in absolutevalue:

number of x∗n with |x∗n − 39.85| greater than 1

1000.

18.10 a Perform the empirical bootstrap simulation as in part a of Exercise 18.8.

1. Generate a bootstrap dataset x∗1, x∗2, . . . , x

∗n from Fn, i.e., draw with replacement

135 numbers from the software data.2. Compute the centered sample mean for the bootstrap dataset:

x∗n − 656.8815

where x∗n is the sample mean of x∗1, x∗2, . . . , x

∗n.

Repeat steps 1 and 2 one thousand times.Estimate P

|Xn − µ| > 10

by the relative frequency of bootstrapped centered sam-ple means that are greater than 1 in absolute value:

number of x∗n with |x∗n − 656.8815| greater than 10

1000.

Page 66: Probability and Statistics Book Solutions

522 Full solutions from MIPS: DO NOT DISTRIBUTE

18.10 b Perform the parametric bootstrap simulation as in part b of Exercise 18.8

1. Generate a bootstrap dataset x∗1, x∗2, . . . , x

∗n from the Exp(0.0015) distribution.

2. Compute the centered sample mean for the bootstrap dataset:

x∗n − 656.8815,

where x∗n is the sample mean of x∗1, x∗2, . . . , x

∗n.

Repeat steps 1 and 2 one thousand times.Estimate P

|Xn − µ| > 10

as in part a.

18.11 Estimate µ by xn = 39.85, σ by sn = 2.09, and estimate F by the distributionfunction of the N(39.85, 4.37) distribution.

1. Generate a bootstrap dataset x∗1, x∗2, . . . , x

∗n from the N(39.85, 4.37) distribution.

2. Compute the sample mean x∗n, sample standard deviation s∗n and empirical dis-tribution function F ∗n of x∗1, x

∗2, . . . , x

∗n. Use these to compute the bootstrapped

KS distancet∗ks = sup

a∈R|F ∗n(a)− Fx∗n,s∗n(a)|

Repeat steps 1 and 2 a large number of times. To investigate to which degree thevalue 0.0987 agrees with the assumed normality of the dataset, one may computethe percentage of t∗ks values that are greater then 0.0987. The closer this percentageis to one, the better the normal distribution fits the data.

18.12 a Note that P(Tn ≤ 0) = P(Mn ≥ θ) = 0, because all Xi are less than θwith probability one. To compute P(T ∗n ≤ 0), note that, since the bootstrap randomsample is from the original dataset, we always have that M∗

n ≤ mn. Hence

P(T ∗n ≤ 0) = P(M∗n ≥ mn) = P(M∗

n = mn) = 1− P(M∗n < mn) .

Furthermore, from Exercise 18.4 we know that P(M∗n < mn) =

1− 1

n

n.

18.12 b Note that

Gn(0) = P(Tn ≤ 0) = P(Mn ≤ 0) = P(X1 ≤ 0) · · ·P(Xn ≤ 0) ,

which is zero because the Xi have a U(0, θ) distribution. With part a it follows that

supt∈R

|G∗n(t)−Gn(t)| ≥ |G∗n(0)−Gn(0)| = P(T ∗n ≤ 0) = 1−

1− 1

n

n

.

18.12 c From the inequality e−x ≥ 1− x, it follows that

1−

1− 1

n

n

≥ 1−e−1/n

n

= 1− e−1.

18.13 a The density of a U(0, θ) distribution is given by fθ(x) = 1/θ, for 0 ≤ x ≤ θ.Hence for 0 ≤ a ≤ θ,

Fθ(a) =

Z a

0

1

θdx =

a

θ.

Page 67: Probability and Statistics Book Solutions

29.1 Full solutions 523

18.13 b We have

P(Tn ≤ t) = P(1−Mn/θ ≤ t) = P(Mn ≥ θ(1− t)) = 1− P(Mn ≤ θ(1− t)) .

Using the rule on page 115 about the distribution function of the maximum, itfollows that for 0 ≤ t ≤ 1,

Gn(t) = 1− P(Mn ≤ θ(1− t)) = 1− Fθ(θ(1− t))n = 1− (1− t)n.

18.13 c By the same argument as before

G∗n(t) = P(T ∗n ≤ t) = 1− PM∗

n ≤ θ(1− t)

.

Since M∗n is the maximum of a random sample X∗

1 , X∗2 , . . . , X∗

n from a U(0, θ) distri-bution, again the rule on page 115 about the distribution function of the maximumyields that for 0 ≤ t ≤ 1,

G∗n(t) = 1− PM∗

n ≤ θ(1− t)

= 1− Fθ(θ(1− t))n = 1− (1− t)n.

19.1 a We must show that E[T ] = θ2. From the formulas for the expectation andvariance of uniform random variables we deduce that E[Xi] = 0 and Var(Xi) =(2θ)2/12 = θ2/3. Hence E

X2

i

= Var(Xi)+(E[Xi])

2 = θ2/3. Therefore, by linearityof expectations

E[T ] =3

n(EX2

1

+ E

X2

2

+ · · ·+ E

X2

n

)

=3

n

θ2

3+ · · ·+ θ2

3

=

3

n· n · θ2

3= θ2.

Since E[T ] = θ2, the random variable T is an unbiased estimator for θ2.

19.1 b The function g(x) = −√x is a strictly convex function, because g′′(x) =

(x−3/2)/4 > 0. Therefore, by Jensen’s inequality, −p

E[T ] < −Eh√

Ti. Since, from

part a we know that E[T ] = θ2, this means that Eh√

Ti

< θ. In other words,√

T

is a biased estimator for θ, with negative bias.

19.2 a We must check whether E[S] = µ. By linearity of expectations, we have

E[S] = E

1

2X1 +

1

3X2 +

1

6X3

=

1

2E[X1] +

1

3E[X2] +

1

6E[X3]

=1

2µ +

1

3µ +

1

6µ =

1

2+

1

3+

1

6

µ = µ.

So indeed, S is an unbiased estimator for µ.

19.2 b We must check under what conditions E[T ] = µ for all µ. By linearity ofexpectations, we have

E[T ] = E[a1X1 + a2X2 + · · ·+ anXn] = a1E[X1] + a2E[X2] + · · ·+ anE[Xn]

= a1µ + a2µ + · · ·+ anµ = (a1 + a2 + · · ·+ an) µ.

This is equal to µ if and only if a1 + a2 + · · ·+ an = 1. Therefore, T is an unbiasedestimator for µ if and only if a1 + a2 + · · ·+ an = 1.

Page 68: Probability and Statistics Book Solutions

524 Full solutions from MIPS: DO NOT DISTRIBUTE

19.3 We must check under what conditions E[T ] = µ for all µ. By linearity ofexpectations, we have

E[T ] = E[a(X1 + X2 + · · ·+ Xn) + b] = a (E[X1] + E[X2] + · · ·+ E[Xn]) + b

= a (µ + µ + · · ·+ µ) + b = anµ + b.

For T to be an unbiased estimator for µ, one must have anµ + b = µ for all µ, witha and b being constants not depending on µ. This can only be the case, when b = 0and a = 1/n.

19.4 a The function g(x) = 1/x is convex, so that according to Jensen’s inequality:

E[T ] = E

1

Xn

>

1

EXn

.Furthermore, E

Xn

= E[X1] = 1/p, so that E[T ] > 1/(1/p) = p. We conclude that

T is a biased estimator for p with positive bias.

19.4 b For each i label ‘Xi ≤ 3’ as a success, then the total number Y of allXi ≤ 3 has a Bin(n, θ) distribution where θ = p + (1 − p)p + (1 − p)2p representsthe probability that a woman becomes pregnant within three or fewer cycles. Thisimplies that E[Y ] = nθ and therefore E[S] = E[Y/n] = E[Y ] /n = θ. This meansthat S is an unbiased estimator for θ.

19.5 We must find out for which c, E[T ] = µ, where µ = 1/λ is the expectation ofthe Exp(λ) distribution. Because Mn has an Exp(nλ) distribution with expectation1/(nλ), we have

E[T ] = E[cMn] = cE[Mn] = c · 1

nλ.

Hence, with the choice c = n, we get E[T ] = 1/λ = µ, which means that T is anunbiased estimator for µ.

19.6 a We must show that E[T ] = 1/λ. By linearity of expectations

E[T ] =n

n− 1

EXn

− E[Mn]

=n

n− 1

δ +

1

λ

δ +1

=

n

n− 1

1

λ− 1

=

n

n− 1

1− 1

n

· 1

λ=

1

λ.

19.6 b Find a linear combination of Xn and Mn of which the expectation is δ. Fromthe expressions for E

Xn

and E[Mn] we see that we can eliminate λ by subtracting

EXn

from nE[Mn]. Therefore, first consider nMn − Xn, which has expectation

EnMn − Xn

= nE[Mn]− E

Xn

= n

δ +

1

δ +1

λ

= (n− 1)δ.

This means that

T =nMn − Xn

n− 1

has expectation δ: E[T ] = EnMn − Xn

/(n − 1) = δ, so that T is an unbiased

estimator for δ.

Page 69: Probability and Statistics Book Solutions

29.1 Full solutions 525

19.6 c Plug in xn = 8563.5, mn = 2398 and n = 20 in the estimator of part b:

t =20 · 2398− 8563.5

19= 2073.5.

19.7 a Note that by linearity of expectations

E[T1] =4

nE[N1]− 2.

Because N1 has a Bin(n, p1) distribution, with p1 = (θ+2)/4, it follows that E[N1] =np1 = n(θ + 2)/4, so that E[T1] = θ. The argument for T2 is similar.

19.7 b Plug in n = 3839, n1 = 1997, and n2 = 32 in the estimators T1 and T2:

t1 =4

3839· 1997− 2 = 0.0808,

t2 =4

3839· 32 = 0.0333.

19.8 From the model assumptions it follows that E[Yi] = βxi for each i. Usinglinearity of expectations, this implies that

E[B1] =1

n

E[Y1]

x1+ · · ·+ E[Yn]

xn

=

1

n

βx1

x1+ · · ·+ βxn

xn

= β,

E[B2] =E[Y1] + · · ·+ E[Yn]

x1 + · · ·+ xn=

βx1 + · · ·+ βxn

x1 + · · ·+ xn= β,

E[B3] =x1E[Y1] + · · ·+ xnE[Yn]

x21 + · · ·+ x2

n

=βx2

1 + · · ·+ βx2n

x21 + · · ·+ x2

n

= β.

19.9 Write T = e−Z/n, where Z = X1 +X2 + · · ·+Xn has a Pois(nµ) distribution,with probabilities P(Z = k) = e−nµ(nµ)k/k!. Therefore

E[T ] = Ehe−Z/n

i=

∞Xk=0

e−k/n · P(Z = k)

= e−nµ∞X

k=0

e−k/n · (nµ)k

k!= e−nµ

∞Xk=0

nµe−1/n

k

k!

Use thatP∞

k=0 xk/k! = ex, with x = nµe−1/n, and conclude that

E[T ] = e−nµ · enµe−1/n

= e−nµ(1−e−1/n).

20.1 We have VarXn

= MSE(Xn) = σ2/n, which is decreasing in n, so the

larger the sample, the more efficient Xn is. In particular VarXn

/Var

X2n

=

(σ2/n)/(σ2/2n) = 2 shows that X2n is twice as efficient as Xn.

20.2 a Compute the mean squared errors of S and T : MSE(S) = Var(S) +[bias(S)]2 = 40 + 0 = 40; MSE(T ) = Var(T )+ [bias(T )]2 = 4+ 9 = 13. We prefer T ,because it has a smaller MSE.

20.2 b Compute the mean squared errors of S and T : MSE(S) = 40, as in a;MSE(T ) = Var(T ) + [bias(T )]2 = 4 + a2. So, if a < 6: prefer T . If a ≥ 6: prefer S.The preferences are based on the MSE criterion.

Page 70: Probability and Statistics Book Solutions

526 Full solutions from MIPS: DO NOT DISTRIBUTE

20.3 Var(T1) = 1/(nλ2), Var(T2) = 1/λ2; hence we prefer T1, because of its smallervariance.

20.4 a Since L has the same distribution as N + 1−M , we find

E[T3] = E[3L−1]= E[3N+3−3M−1] = 3N+2− E[2T2+ 2] = 3N −2N =N.

Here we use that E[2T2] = 2N , since T2 is unbiased.

20.4 b We have

Var(T3) = Var(3L− 1) = 9Var(L) = 9Var(M) =1

2(N + 1)(N − 2).

20.4 c We compute:

Var(T3) /Var(T2) = 9Var(M) /Var

32M − 1

= 9Var(M) / 9

4Var(M) = 4.

So using the maximum is 4 times as efficient as using the minimum!

20.5 From the variance of the sum rule: Var((U + V )/2) = 14(Var(U) + Var(V ) +

2Cov(U, V )). Using that Var(V )=Var(U), we get from this that that the relativeefficiency of U with respect to W is equal to

Var(W )

Var(U)=

Var((U + V )/2)

Var(U)=

1

4

1 + 1 + 2

Cov(U, V )pVar(V )

pVar(U)

=

1

2+

1

2ρ(U, V ) .

Since the correlation coefficient is always between -1 and 1, it follows that the relativeefficiency of U with respect to W is always between 0 and 1. Hence it ia always better(in the MSE sense) to use W .

20.6 a By linearity of expectations, E[U1] = E[T1] +13(π−E[T1]−E[T2]−E[T3]),

which equals α1 + 13(π − α1 − α2 − α3) = α1.

To compute the variances, rewrite U1 as U1 = 23T1 − 1

3T2 − 1

3T3 + 1

3π. Then, by

independence, Var(U1) = 49Var(T1) + 1

9Var(T2) + 1

9Var(T3) = 2

3σ2.

20.6 b We have Var(T1) /Var(U1) = 3/2, so U1 is 50% more efficient than T1.

20.6 c There are at least two ways to obtain an efficient estimator for α1 = α2. Thefirst is via the insight that T1 and T2 both estimate α1, so (T1 +T2)/2 is an efficientestimator for α1 (c.f. Exercise 20.1). Then we can improve the efficiency in the sameway as in part a. This yields the estimator

V1 = 12(T1 + T2) + 1

3(π − T1 − T2 − T3) = 1

6T1 + 1

6T2 − 1

3T3 + 1

3π.

The second way is to find the linear estimator V1 = uT1 + vT2 + wT3 + t with thesmallest MSE, optimising over u, v, w and t. This will result in the same estimatoras obtained with the first way.

20.7 We compare the MSE’s, which by unbiasedness are equal to the variances.Both N1 and N2 are binomally distributed, so Var(N1) = np1(1−p1) = n(θ+2)(1−(θ + 2)/4)/4, and Var(N2) = np2(1− p2) = nθ(1− θ)/4. It follows that

Var(T1) =16

n2Var(N1) =

1

n(4− θ2); Var(T2) =

16

n2Var(N2) =

1

nθ(4− θ).

Since (4− θ2) > θ(4− θ) for all θ, we prefer T2.

Page 71: Probability and Statistics Book Solutions

29.1 Full solutions 527

20.8 a This follows directly from linearity of expectations:

E[T ] = ErXn+(1− r)Ym

= rE

Xn

+ (1− r)E

Ym

= rµ + (1− r)µ = µ.

20.8 b Using that Xn and Ym are independent, we find MSE(T )=Var(T ) =r2Var

Xn

+ (1− r)2Var

Ym

= r2 · σ2/n + (1− r)2 · σ2/m.

To find the minimum of this parabola we differentiate with respect to r andequate the result to 0: 2r/n − 2(1 − r)/m = 0. This gives the minimum value:2rm− 2n(1− r) = 0 or r = n/(n + m).

20.9 a Since E[T1] = p, T1 is unbiased. The estimator T2 takes only the values 0and 1, the latter with probability pn. So E[T2] = pn, and T2 is biased for all n > 1.

20.9 b Since T1 is unbiased, MSE(T1) = Var(T1) = np(1− p)/n2 = 1np(1− p). Now

for T2: this random variable has a Ber(pn) distribution, henceMSE(T2) = E

(T2 − θ)2

= E

(T2 − p)2

= p2 · P(T2 = 0) + (1 − p)2 · P(T2 = 1) =

p2(1− pn) + (1− p)2pn = pn − 2pn+1 + p2.

20.9 c For n = 2: MSE(T2) = 4p MSE(T1), so for p < 14

T2 is more efficient thanT1, but otherwise T1 is more efficient.

20.10 a Recall that the variance of an Exp(λ) distribution equals 1/λ2, hence themean squared error of T equals

MSE(T ) = Var(T ) +E[T ]− λ−12

= c2 · n · λ−2 +c · n · λ−1 − λ−12

= λ−2 c2 · n + (c · n− 1)2

= λ−2 c2 · (n2 + n)− 2cn + 1.

20.10 b This is a parabola in c, taking its smallest value when 2c(n2 + n) = 2n,which happens for c = 1/(n + 1). So the estimator with the smallest MSE is

U =1

n + 1· (X1 + X2 + · · ·+ Xn) .

Substituting c = 1/n, and c = 1/(n + 1) in the formula for the MSE, we find that

MSE(Xn) =1

n, and MSE(U) =

1

n + 1.

So U performs better in terms of MSE, but not much.

20.11 We have MSE(T1) = Var(T1), which equals

Var(T1) = Var

nX

i=1

xiYi

. nXi=1

x2i

!=

nXi=1

x2i Var(Yi)

. nXi=1

x2i

!2

= σ2. nX

i=1

x2i

!.

Also, MSE(T2) = Var(T2), which equals

Var(T2) = Var

1

n

nXi=1

Yi

xi

!=

1

n2

nXi=1

Var

Yi

xi

=

σ2

n2

nXi=1

1/x2

i

.

Finally, MSE(T3) = Var(T3), which equals

Page 72: Probability and Statistics Book Solutions

528 Full solutions from MIPS: DO NOT DISTRIBUTE

Var(T3) = Var

nX

i=1

Yi

. nXi=1

xi

!=

nXi=1

Var(Yi). nX

i=1

xi

!2

= nσ2. nX

i=1

xi

!2

.

To show that Var(T3) < Var(T2): introduce a random variable X by P(X = xi) = pi

for i = 1 . . . n, and apply Jensen with the function g(x) = 1/x2, which is strictlyconvex on (0,∞) (note that all xi in the cherry tree example are positive).

20.12 a Following the hint:

P(Mn ≤ k) = P(X1 ≤ k, X2 ≤ k, . . . , Xn ≤ k)

=k

N· k − 1

N − 1, · · · · · k − n + 1

N − n + 1

=k!

(k − n)!

(N − n)!

N !.

20.12 b To have Mn = n we should have drawn all numbers 1, 2 . . . , n, and con-versely, so

P(Mn = n) = P(Mn ≤ n) =n!(N − n)!

N !.

20.12 c This also follows directly from part a:

P(Mn = k) = P(Mn ≤ k)− P(Mn ≤ k − 1)

=k!

(k − n)!

(N − n)!

N !− (k − 1)!

(k − 1− n)!

(N − n)!

N !

=(k − 1)![k − (k − n)]

(k − 1− n)!

(N − n)!

N !

= n · (k − 1)!

(k − n)!

(N − n)!

N !.

21.1 Setting Xi = j if red appears in the ith experiment for the first time on thejth throw, we have that X1, X2, and X3 are independent Geo(p) distributed randomvariables, where p is the probability that red appears when throwing the selecteddie. The likelihood function is

L(p) = P(X1 = 3, X2 = 5, X3 = 4) = (1− p)2p · (1− p)4p · (1− p)3p

= p3(1− p)9,

so for D1 one has that L(p) = L( 56) =

56

3 1− 5

6

9, whereas for D2 one has that

L(p) = L( 16) =

16

3 1− 1

6

9= 56 · L( 5

6). It is very likely that we picked D2.

21.2 As in the solution of Exercise 21.1, the likelihood is given by L(p) = p3(1−p)9,where p is between 0 and 1. So the loglikelihood is given by `(p) = 3 ln p+9 ln(1−p),and differentiating the loglikelihood with respect to p gives:

d

dp

`(p)

=

3

p− 9

1− p.

We find that ddp

`(p)

= 0 if and only if p = 1/4, and since `(p) (and also L(p)!) at-

tains its maximum for this value of p, we find that the maximum likelihood estimateof p is p = 1/4.

Page 73: Probability and Statistics Book Solutions

29.1 Full solutions 529

21.3 a The likelihood L(µ) is given by

L(µ) = CP(X = 0)229 P(X = 1)211 P(X = 2)93 P(X = 3)35 P(X = 4)7 P(X = 7)

= Cµ537e−576µ.

But then the loglikelihood `(µ) satisfies

`(µ) = ln C + 537 ln µ− 576µ.

But then we have, that `′(µ) = 537/µ− 576, from which it follows that `′(µ) = 0 ifand only if µ = 537/576 = 0.93229.

21.3 b Now the likelihood L(µ) is given by

L(µ) = CP(X = 0, 1)440 P(X = 2)93 P(X = 3)35 P(X = 4)7 P(X = 7)

= C(1 + µ)440µ316e−576µ.

But then the loglikelihood `(µ) satisfies

`(µ) = ln C + 440 ln(1 + µ) + 316 ln µ− 576µ.

But then we have, that

`′(µ) =440

1 + µ+ 316/µ− 576,

from which it follows that `′(µ) = 0 if and only if 576µ2 − 180µ + 260 = 0. We findthat µ = 0.9351086.

21.4 a The likelihood L(µ) is given by

L(µ) = P(X1 = x1, . . . , Xn = xn) = P(X1 = x1) · · ·P(Xn = xn)

=µx1

x1!· e−µ · · · µ

xn

xn!· e−µ =

e−nµ

x1! · · ·xn!µx1+x2+···+xn .

21.4 b We find that the loglikelihood `(µ) is given by

`(µ) =

nX

i=1

xi

!ln(µ)− ln (x1! · · ·xn!)− nµ.

Henced`

dµ=

Pxi

µ− n,

and we find—after checking that we indeed have a maximum!—that xn is the max-imum likelihood estimate for µ.

21.4 c In b we have seen that xn is the maximum likelihood estimate for µ. Due tothe invariance principle from Section 21.4 we thus find that e−xn is the maximumlikelihood estimate for e−µ.

21.5 a By definition, the likelihood L(µ) is given by

L(µ) = fµ(x1) · · · fµ(xn)

=1√2π

e−12 (x1−µ)2 · · · 1√

2πe−

12 (xn−µ)2

= (2π)−n/2 · e− 12Pn

i=1(xi−µ)2 .

Page 74: Probability and Statistics Book Solutions

530 Full solutions from MIPS: DO NOT DISTRIBUTE

But then the loglikelihood `(µ) satisfies

`(µ) = −n

2ln(2π)− 1

2

nXi=1

(xi − µ)2.

Differentiating `(µ) with respect to µ yields

`′(µ) =

nXi=1

(xi − µ) = −nµ +

nXi=1

xi.

So `′(µ) = 0 if and only if µ = xn. Since `(µ) attains a maximum at this value of µ(check this!), we find that xn is the maximum likelihood estimate for µ.

21.5 b Again by the definition of the likelihood, we find that L(σ) is given by

L(σ) = fσ(x1) · · · fσ(xn)

=1

σ√

2πe−

12 x2

1/σ2 · · · 1

σ√

2πe−

12 x2

n/σ2

= σ−n(2π)−n/2e− 1

2σ2Pn

i=1 x2i .

But then the loglikelihood `(σ) satisfies

`(σ) = −n ln σ − n

2ln(2π)− 1

2σ2

nXi=1

x2i .

Differentiating `(σ) with respect to σ yields

`′(σ) = −n

σ+

1

σ3

nXi=1

x2i .

So `′(σ) = 0 if and only if σ =

r1

n

Xn

i=1x2

i . Since `(σ) attains a maximum at

this value of σ (check this!), we find that

r1

n

Xn

i=1x2

i is the maximum likelihood

estimate for σ.

21.6 a In this case the likelihood function L(δ) = 0 for δ > x(1), and

L(δ) = enδ−P xi for δ ≤ x(1).

21.6 b It follows from the graph of L(δ) in a that x(1) is the maximum likelihoodestimate for δ.

21.7 By definition, the likelihood L(θ) is given by

L(θ) = fθ(x1) · · · fθ(xn)

=x1

θ2e−

12 x2

1/θ2 · · · xn

θ2e−

12 x2

n/θ2

= θ−2n

nY

i=1

xi

!e− 1

2θ2Pn

i=1 x2i .

But then the loglikelihood `(θ) is equal to

Page 75: Probability and Statistics Book Solutions

29.1 Full solutions 531

`(θ) = −2n ln θ + ln

nY

i=1

xi

!− 1

2θ2

nXi=1

x2i .

Differentiating `(θ) with respect to θ yields

`′(θ) =−2n

θ+

1

θ3

nXi=1

x2i ,

and we find that `′()θ) = 0 if and only if

θ =

vuut 1

2n

nXi=1

x2i .

Since `(θ) attains a maximum at this value of θ (check this!), we find that the

maximum likelihood estimate for θ is given byq

12n

Pni=1 x2

i .

21.8 a The likelihood L(θ) is given by

L(θ) = C ·

1

4(2 + θ)

1997

·

1

32

·

1

4(1− θ)

906

·

1

4(1− θ)

904

=C

43839· (2 + θ)1997 · θ32 · (1− θ)1810,

where C is the number of ways we can assign 1997 starchy-greens, 32 sugary-whites,906 starchy-whites, and 904 sugary-greens to 3839 plants. Hence the loglikelihood`(θ) is given by

`(θ) = ln(C)− 3839 ln(4) + 1997 ln(2 + θ) + 32 ln(θ) + 1810 ln(1− θ).

21.8 b A short calculation shows that

d`(θ)

dθ= 0 ⇔ 3810θ2 − 1655θ − 64 = 0,

so the maximum likelihood estimate of θ is (after checking that L(θ) indeed attainsa maximum for this value of θ):

−1655 +√

3714385

7620= 0.0357.

21.8 c In this general case the likelihood L(θ) is given by

L(θ) = C ·

1

4(2 + θ)

n1

·

1

n2

·

1

4(1− θ)

n3

·

1

4(1− θ)

n4

·

=C

4n· (2 + θ)n1 · θn2 · (1− θ)n3+n4 ,

where C is the number of ways we can assign n1 starchy-greens, n2 sugary-whites,n3 starchy-whites, and n4 sugary-greens to n plants. Hence the loglikelihood `(θ) isgiven by

`(θ) = ln(C)− n ln(4) + n1 ln(2 + θ) + n2 ln(θ) + (n3 + n4) ln(1− θ).

Page 76: Probability and Statistics Book Solutions

532 Full solutions from MIPS: DO NOT DISTRIBUTE

A short calculation shows that

d`(θ)

dθ= 0 ⇔ nθ2 − (n1 − n2 − 2n3 − 2n4)θ − 2n2 = 0,

so the maximum likelihood estimate of θ is (after checking that L(θ) indeed attainsa maximum for this value of θ):

n1 − n2 − 2n3 − 2n4 +p

(n1 − n2 − 2n3 − 2n4)2 + 8nn2

2n.

21.9 The probability density of this distribution is given by fα,β(x) = 0 if x is notbetween α and β, and

fα,β(x) =1

β − αfor α ≤ x ≤ β.

Since the xi must be in the interval between α and β, the likelihood (which is afunction of two variables!) is given by

L(α, β) =

1

β − α

n

for α ≤ x(1) and β ≥ x(n),

and L(α, β) = 0 for all other values of α and β. So outside the ‘rectangle’ (−∞, x(1)]×[x(n),∞) the likelihood is zero, and clearly on this ‘rectangle’ it attains its maximumin (x(1), x(n)). The maximum likelihood estimates of α and β are therefore α = x(1)

and β = x(n).

21.10 The likelihood is

L(α) =α

xα+11

α

xα+12

· · · α

xα+1n

= αn

nYi=1

xi

−(α+1)

,

so the loglikelihood is

`(α) = n ln α− (α + 1) ln

nYi=1

xi

.

Differentiating `(α) to α yields as maximum likelihood α = n/ ln

Qni=1 xi

.

21.11 a Since the dataset is a realization of a random sample from a Geo(1/N)distribution, the likelihood is L(N) = P(X1 = x1, X2 = x2, . . . , Xn = xn), whereeach Xi has a Geo(1/N) distribution. So

L(N) =

1− 1

N

x1−11

N

1− 1

N

x2−11

N· · ·

1− 1

N

xn−11

N

=

1− 1

N

−n+Pn

i=1 xi

1

N

n

.

But then the loglikelihood is equal to

`(N) = −n ln N +

− n +

nXi=1

xi

ln

1− 1

N

.

Page 77: Probability and Statistics Book Solutions

29.1 Full solutions 533

Differentiating to N yields

d

dN

`(N)

=−n

N+

− n +

nXi=1

xi

1

N(N − 1),

Now ddN

`(N)

= 0 if and only if N = xn. Because `(N) attains its maximum at

xn, we find that the maximum likelihood estimate of N is N = xn.

21.11 b Since P(Y = k) = 1/N for k = 1, 2, . . . , N , the likelihood is given by

L(N) =

1

N

n

for N ≥ y(n),

and L(N) = 0 for N < y(n). So L(N) attains its maximum at y(n); the maximum

likelihood estimate of N is N = y(n).

21.12 Since L(N) = P(Z = k), it follows from Exercise 4.13c that

L(N) =

mk

N−mr−k

Nr

.

In order to see that L(N) increases for N < mrk

, and decreases for N > mrk

, considerthe ratio L(N)/L(N − 1). After some elementary calculations one finds that

L(N)

L(N − 1)=

(N −m)(n− r)

N(N −m− r + k).

So L(N) is increasing if L(N)L(N−1)

= (N−m)(N−r)N(N−m−r+k)

> 1, and another elementary calcu-lation shows that this is when N < mr

k.

21.13 Let N(t1, t2) be the number of customers arriving in the show between timet1 and time t2. Then it follows from the assumption that customers arrive at theshop according to a Poisson process with rate λ, that

P(N(t1, t2) = k) =(λ(t2 − t1))

k

k!e−λ(t2−t1), for k = 0, 1, 2, . . . .

The likelihood L(λ) is given by

L(λ) = P(N(12.00, 12.15) = 2, N(12.15, 12.45) = 0, N(12.45, 13.00) = 1)

+P(N(12.00, 12.15) = 1, N(12.15, 12.45) = 1, N(12.45, 13.00) = 0) .

Since N(12.00, 12.15), N(12.15, 12.45), and N(12.45, 13.00) are independent randomvariables, we find that

L(λ) =( 14λ)2

2!e−

14 λ · ( 1

2λ)0

0!e−

12 λ · ( 1

4λ)1

1!e−

14 λ

+( 14λ)1

1!e−

14 λ · ( 1

2λ)1

1!e−

12 λ · ( 1

4λ)0

0!e−

14 λ

=

1

128λ3 +

1

8λ2

e−λ.

Now L′(λ) = 0 if and only if λ3 + 13λ2 − 32λ = 0. Since λ > 0, we find that

λ = −13+√

2972

= 2.1168439.

Page 78: Probability and Statistics Book Solutions

534 Full solutions from MIPS: DO NOT DISTRIBUTE

21.14 For i = 1, 2, . . . , n, ri is the realization of a continuous random variable Ri.Since the shots at the disc do not influence each other, the Ri are independent, andall have the same distribution function Fθ(x) = x2/θ2 if x is between 0 and θ. Butthen fθ(x) = 2x/θ2 for x between 0 and θ (and f(x) = 0 otherwise). Since the discis hit each of the n shots, the likelihood is:

L(θ) = fθ(r1)fθ(r2) · · · fθ(rn) =2r1

θ2

2r2

θ2· · · 2rn

θ2=

2nQni=1 ri

θ2n,

for θ ≥ r(n), and L(θ) = 0 otherwise. But then we at once see that L(θ) attains its

maximum at θ = r(n), i.e., the maximum likelihood estimate for θ is θ = r(n).

21.15 At temperature t, the probability of failure of an O-ring is given by p(t), sothe probability of a failure of k O-rings at this temperature, for k = 0, 1, . . . , 6, isgiven by

6

k

!(p(t))k(1− p(t))6−k.

Setting

C =

6

0

!16 6

1

!5 6

2

!2

,

we find that the likelihood L(a, b) is given by

L(a, b) = C · (p(53))2(1− p(53))4 · · · (p(53))0(1− p(53))6

= C ·23Y

i=1

(p(ti))ni(1− p(ti))

6−ni ,

where ti is the temperature and ni is the number of failing O-rings during the ithlaunch, for i = 1, 2 . . . , 23. But then the loglikelihood `(a, b) is given by

`(a, b) = ln C +

23Xi=1

ni ln p(ti) +

23Xi=1

(6− ni) ln(1− p(ti)).

21.16 Since sn is the realization of a Bin(n, p) distributed random variable Sn, wefind that the likelihood L(p) is given by

L(p) =

n

sn

!psn(1− p)n−sn ,

from which we see that the loglikelihood `(p) satisfies

`(p) = ln

n

sn

!+ sn ln p + (n− sn) ln(1− p).

But then differentiating `(p) with respect to p yields that

`′(p) =sn

p− n− sn

1− p,

and we find that `′(p) = 0 if and only if p = sn/n. Since `(p) attains a maximum atthis value of p (check this!), we have obtained that sn/n is the maximum likelihood

Page 79: Probability and Statistics Book Solutions

29.1 Full solutions 535

estimate (and Sn/n the maximum likelihood estimator) for p. Due to the invarianceprinciple from Section 21.4 we find that 2n/sn is the maximum likelihood estimatefor π, and that

T =2n

Sn

is the maximum likelihood estimator for π.

22.1 a SinceP

xiyi = 12.4,P

xi = 9,P

yi = 4.8,P

x2i = 35, and n = 3, we find

(c.f. (22.1) and (22.2)), that

β =nP

xiyi − (P

xi)(P

yi)

nP

x2i − (

Pxi)2

=3 · 12.4− 9 · 4.8

3 · 35− 92= −1

4,

and α = yn − βxn = 2.35.

22.1 b Since ri = yi− α− βxi, for i = 1, . . . , n, we find r1 = 2−2.35+0.25 = −0.1,r2 = 1.8 − 2.35 + 0.75 = 0.2, r3 = 1 − 2.35 + 1.25 = −0.1, and r1 + r2 + r3 =−0.1 + 0.2− 0.1 = 0.

22.1 c See Figure 29.2.

−1 0 1 2 3 4 5 6

0

1

2

3

· ··

(x3, y3)

...........................................................................................................................................................................................................................................................................................................................................................................................................................................................................................................................................................................................................................................................................

Fig. 29.2. Solution of Exercise 22.1 c.

22.2 As in the previous exercise, we have thatP

xiyi = 12.4,P

xi = 9,P

yi = 4.8,Px2

i = 35. However, now n = 4, and we find that

β =nP

xiyi − (P

xi)(P

yi)

nP

x2i − (

Pxi)2

=4 · 12.4− 9 · 4.8

4 · 35− 92= 0.10847.

Since we now have that xn = 9/4, and yn = 1.2, we find that α = yn−βxn = 0.9559.

22.3 a Ignoring the subscripts to the sums, we haveXxi = 10,

Xx2

i = 21.84,X

yi = 20, andX

xiyi = 41.61

From (22.1) we find that the least squares estimate β of β is given by

Page 80: Probability and Statistics Book Solutions

536 Full solutions from MIPS: DO NOT DISTRIBUTE

β =5 · 41.61− 10 · 20

5 · 21.84− 102= 0.875,

while from (22.2) it follows that the least squares estimate α of α is given by

α =20

5− 0.875 · 10

5= 2.25

22.3 b See Figure 29.3.

−1 0 1 2 3 4

0

1

2

3

4

5

·· ···

(x5, y5)

................................................................................................................................................................................................................................................................................................................................................................................................................................................................................

....................

....................

....................

....................

.............................................

Fig. 29.3. Solution of Exercise 22.3b.

22.4 Since the least squares estimate β of β is given by (22.1), we find that

β =100 · 5189− 231.7 · 321

100 · 2400.8− 231.72= 2.385.

From (22.2) we find that

α = 3.21− 2.317cdotβ = −2.316.

22.5 With the assumption that α = 0, the method of least squares tells us now tominimize

S(β) =

nXi=1

(yi − βxi)2 .

NowdS(β)

dβ= −2

nXi=1

(yi − βxi)xi = −2

nX

i=1

xiyi − β

nXi=1

x2i

!,

Page 81: Probability and Statistics Book Solutions

29.1 Full solutions 537

sodS(β)

dβ= 0 ⇔ β =

Pni=1 xiyiPni=1 x2

i

.

Because S(β) has a minimum for this last value of β, we see that the least squaresestimator β of β is given by

β =

Pni=1 xiYiPni=1 x2

i

.

22.6 In Exercise 22.5 we have seen that

β =

nXi=1

xiYi

nXi=1

x2i

,

so for the timber dataset we find that β = 34.13. We use the estimated regressionline y = 34.13x to predict the Janka hardness. For density x = 65 we find as aprediction for the Janka hardness y = 2218.45.

22.7 The sum of squares function S(α, β) is now given by

S(α, β) =

nXi=1

yi − eα+βxi

2

.

In order to find the values of α and β for which S(α, β) attains a maximum, one coulddifferentiate S(α, β) to α and β. However, this does not yield workable expressionssuch as those in (22.1) and (22.2). In order to find α and β for a given bivariatedataset (x1, y1), (x2, y2), . . . , (xn, yn), an iterative method is best suited.

22.8 In the model with intercept, α and β are chosen in such a way, that S(α, β) =Pni=1 r2

i is minimal. In the model with no intercept we chose β in such a way, thatS(0, β) =

Pni=1 r2

i is minimal. Clearly, in this last model the residual sum of squaresPni=1 r2

i is greater than, or equal to the residual sum of squares in the model withintercept.

22.9 With the assumption that β = 0, the method of least squares tells us now tominimize

S(α) =

nXi=1

(yi − α)2 .

NowdS(α)

dα= −2

nXi=1

(yi − α) = −2(nyn − nα) = −2n(yn − α).

So dS(α)dα

= 0 if and only if α = yn. Since S(α) has a minimum for this last value ofα, we see that Yn is the least squares estimator α of α.

22.10 a One has that n = 3,P

xi = 3,P

yi = 4,P

x2i = 5, and

Pxiyi = 2,

so β = 3·2−3·43·5−32 = −1. Since x3 = 1 and y3 = 4/3, we find that α = 7/3. Since

r1 = −1/3, r2 = 2/3, and r3 = −1/3, we find that A(α, β) = |r1|+ |r2|+ |r3| = 4/3.Note that S(α, β) = r2

1 + r22 + r2

3 = 1/3.

Page 82: Probability and Statistics Book Solutions

538 Full solutions from MIPS: DO NOT DISTRIBUTE

22.10 b A(α,−1) = |2 − α + 0| + |2 − α + 1| + |0 − α + 2|. For 2 ≤ α < 3 we findthat A(α,−1) = α− 1. For α ≥ 3 we find that A(α,−1) = 3α− 7, while for α < 2we find that A(α,−1) = 7− 3α. We find that A(α,−1) < (α, β) when α is between17/9 and 7/3. Note that A(α,−1) is minimal (and equal to 1) for α− 2.

22.10 c Since A(2,−1) = 1, we must have that 1 < α < 3 (since otherwise A(α, β) ≥|2 − α + 0| > 1). Clearly we must have that β < 0 (since otherwise A(α, β) ≥|0−α− 2β| > 1). Considering the various cases for α yields that A(α, β) attains itsminimum at α = 2 and β = −1.

22.11 a In the present set-up one has to minimize

S(β, γ) =

nXi=1

yi − (βxi + γx2

i )2

.

Differentiating S(β, γ) to β and γ yields

∂S

∂β= −2

nXi=1

xi

yi − βxi − γx2

i

and

∂S

∂γ= −2

nXi=1

x2i

yi − βxi − γx2

i

.

We find that∂S

∂β= 0 ⇔ β

nXi=1

x2i + γ

nXi=1

x3i =

nXi=1

xiyi

and∂S

∂γ= 0 ⇔ β

nXi=1

x3i + γ

nXi=1

x4i =

nXi=1

x2i yi.

22.11 b Using Cramer’s rule from linear algebra now yields that

β =

Pxiyi

Px3

iPx2

i yi

Px4

i

Px2i

Px3

iPx3

i

Px4

i

=(P

xiYi)(P

x4i )− (

Px3

i )(P

x2i Yi)

(P

x2i )(P

x4i )− (

Px3

i )2

and

γ =

Px2i

PxiyiP

x3i

Px2

i yi

Px2i

Px3

iPx3

i

Px4

i

=(P

x2i )(P

x2i Yi)− (

Px3

i )(P

xiYi)

(P

x2i )(P

x4i )− (

Px3

i )2

.

Since S(β, γ) is a ‘vase’, the above stationary point (β, γ) is a global minimum forS(β, γ). This finishes the exercise.

22.12 a Since the denominator of β is a number, not a random variable, one hasthat

Ehβi

=E[n(

PxiYi)− (

Pxi)(

PYi)]

xP

x2i − (

Pxi)2

.

Furthermore, the numerator of this last fraction can be written as

Page 83: Probability and Statistics Book Solutions

29.1 Full solutions 539

EhnX

xiYi

i− E

h(X

xi)(X

Yi)i,

which is equal to

nX

(xiE[Yi])− (X

xi)X

E[Yi] .

22.12 b Substituting E[Yi] = α + βxi in the last expression, we find that

Ehβi

=nP

(xi(α + βxi))− (P

xi) [P

(α + βxi)]

xP

x2i − (

Pxi)2

.

22.12 c The numerator of the previous expression for Ehβi

can be simplified to

nαP

xi + nβP

x2i − nα

Pxi − β(

Pxi)(

Pxi)

nP

x2i − (

Pxi)

2 ,

which is equal toβ(n

Px2

i − (P

xi)2)

nP

x2i − (

Pxi)

2 .

22.12 d From c it now follows that Ehβi

= β, i.e., β is an unbiased estimator for β.

23.1 This is the case: normal data with variance known. So we should use theformula from Section 23.2 (the case variance known):

xn − 1.96σ√n

, xn + 1.96σ√n

,

where xn = 743, σ = 5 and n = 16. Because zα/2 = z0.025 = 1.96, the 95% confidenceinterval is:

743− 1.96 · 5√16

, 743 + 1.96 · 5√16

= (740.55, 745.45).

23.2 This is the case: normal data with variance unknown. So we should use theformula from Section 23.2 (the case variance unknown):

xn − tn−1,α/2sn√n

, xn + tn−1,α/2sn√n

,

where xn = 3.54, sn = 0.13 and n = 34. Because tn−1,α/2 = t33,0.01 ≈ t30,0.01 =2.457, the 98% confidence interval is:

3.54− 2.457 · 0.13√34

, 3.54 + 2.457 · 0.13√34

= (3.485, 3.595).

One can redo the same calculation using t33,0.01 = 2.445 (obtained from a softwarepackage), and find (3.4855, 3.5945).

23.3 This is the case: normal data with variance unknown. So we should use theformula from Section 23.2 (the case variance unknown):

xn − tn−1,α/2sn√n

, xn + tn−1,α/2sn√n

,

where xn = 93.5, sn = 0.75 and n = 10. Because tn−1,α/2 = t9,0.025 = 2.262, the95% confidence interval is:

93.5− 2.262 · 0.75√10

, 93.5 + 2.262 · 0.75√10

= (92.96, 94.036).

Page 84: Probability and Statistics Book Solutions

540 Full solutions from MIPS: DO NOT DISTRIBUTE

23.4 This is the case: normal data with variance unknown. So we should use theformula from Section 23.2 (the case variance unknown):

xn − tn−1,α/2sn√n

, xn + tn−1,α/2sn√n

,

where xn = 195.3, sn = 16.7 and n = 18. Because tn−1,α/2 = t17,0.025 = 2.110, the95% confidence interval is:

195.3− 2.110 · 16.7√18

, 195.3 + 2.110 · 16.7√18

= (186.99, 203.61).

23.5 a The standard confidence interval for the mean of a normal sample withunknown variance applies, with n = 23, x = 0.82 and s = 1.78, so:

x− t22,0.025 · s√23

, x + t22,0.025 · s√23

.

The critical values come from the t(22) distribution: t22,0.025 = 2.074. The actualinterval becomes:

0.82− 2.074 · 1.78√23

, 0.82 + 2.074 · 1.78√23

= (0.050, 1.590).

23.5 b Generate one thousand samples of size 23, by drawing with replacementfrom the 23 numbers

1.06, 1.04, 2.62, . . . , 2.01.

For each sample x∗1, x∗2, . . . , x

∗23 compute: t∗ = x∗23 − 0.82/(s∗23/

√23), where s∗23 =q

122

P(x∗i − x∗23)2.

23.5 c We need to estimate the critical value c∗l such that P(T ∗ ≤ c∗l ) ≈ 0.025. Wetake c∗l = −2.101, the 25th of the ordered values, an estimate for the 25/1000 = 0.025quantile. Similarly, c∗l is estimated by the 976th, which is 2.088.The bootstrap confidence interval uses the c∗ values instead of the t-distributionvalues ±tn−1,α/2, but beware: c∗l is from the left tail and appears on the right-handside of the interval and c∗u on the left-hand side:

xn − c∗usn√n

, xn − c∗lsn√n

.

Substituting c∗l = −2.101 and c∗u = 2.088, the confidence interval becomes:0.82− 2.088 · 1.78√

23, 0.82 + 2.101 · 1.78√

23

= (0.045, 1.600).

23.6 a Because events described by inequalities do not change when we multi-ply the inqualities by a positive constant or add or subtract a constant, the

following equalities hold: PLn < θ < Un

= P(3Ln + 7 < 3µ + 7 < 3Un + 7) =

P(3Ln < 3µ < 3Un) = P(Ln < µ < Un), and this equals 0.95, as is given.

23.6 b The confidence interval for θ is obtained as the realization of (Ln, Un), thatis: (ln, un) = (3ln + 7, 3un + 7). This is obtained by transforming the confidenceinterval for µ (using the transformation that is applied to µ to get θ).

Page 85: Probability and Statistics Book Solutions

29.1 Full solutions 541

23.6 c We start with P(Ln < µ < Un) = 0.95 and try to get 1 − µ in the mid-dle: P(Ln < µ < Un) = P(−Ln > −µ > −Un) = P(1− Ln > 1− µ > 1− Un) =P(1− Un < 1− µ < 1− Ln), where we see that the minus sign causes an inter-change: Ln = 1−Un and Un = 1−Ln. The confidence interval: (1− 5, 1− (−2)) =(−4, 3).

23.6 d If we knew that Ln and Un were always positive, then we could conclude:P(Ln < µ < Un) = P

L2

n < µ2 < U2n

and we could just square the numbers in the

confidence interval for µ to get the one for θ. Without the positivity assumption, thesharpest conclusion you can draw from Ln < µ < Un is that µ2 is smaller than themaximum of L2

n and U2n. So, 0.95 = P(Ln < µ < Un) ≤ P

0 ≤ µ2 < maxL2

n, U2n

and the confidence interval [0, maxl2n, u2n) = [0, 25) has a confidence of at least

95%. This kind of problem may occur when the transformation is not one-to-one(both −1 and 1 are mapped to 1 by squaring).

23.7 We know that (ln, un) = (2, 3), where ln and un are the realizations of Ln

and Un, that have the property P(Ln < µ < Un) = 0.95. This is equivalent with

Pe−Un < e−µ < e−Ln

= 0.95,

so thate−3, e−2

= (0.050, 0.135) is a 95% confidence interval for P(X = 0) = e−µ.

23.8 Define random variables

Xi = weight of ith bottle together with filling amount

Wi = weight of ith bottle alone

Yi = weight of ith filling amount

so that Xi = Wi+Yi. It is given that Wi has a N(250, 152) distribution and accordingto Exercise 23.1, Yi has a N(µy, 52). Since they are independent Xi = Wi + Yi hasa normal distribution with expectation µ = 250 + µy and variance 152 + 52 = 250:Xi ∼ N(250 + µy, 250). Our data consist of the weights of 16 filled bottles of wine,x1, . . . , x16. On the basis of these we can construct a confidence interval for µ. Sincewe are in the case: normal data with known variance, this is

xn − zα/2σ√n

, xn + zα/2σ√n

,

where xn = 998, σ =√

250 and n = 16. Because zα/2 = z0.025 = 1.96, the 95%confidence interval is:

998− 1.96 ·√

250√16

, 998 + 1.96 ·√

250√16

= (990.25, 1005.75).

Since, this is a 95% confidence interval for µ = 250+µy, the 95% confidence intervalfor µy, is given by (990.25− 250, 1005.75− 50) = (740.25, 755.75).

23.9 a Since we do not assume a particular parametric model, we are dealing withan empirical bootstrap simulation. Generate bootstrap samples x∗1, x

∗2, . . . , x

∗n size

n = 2608, from the empirical distribution function of the dataset, which is equivalentto generate from the discrete distribution with probability mass function:

Page 86: Probability and Statistics Book Solutions

542 Full solutions from MIPS: DO NOT DISTRIBUTE

a 0 1 2 3 4p(a) 57/2608 203/2608 383/2608 525/2608 532/2608

a 5 6 7 8 9p(a) 408/2608 273/2608 139/2608 45/2608 27/2608

a 10 11 12 13 14p(a) 10/2608 4/2608 0 1/2608 1/2608

Then determine:

t∗ =x∗n − 3.8715

s∗n/√

2608.

Repeat this one thousand times. Next estimate the values c∗l and c∗u such that

P(T ∗ ≤ c∗l ) = 0.025 and P(T ∗ ≥ c∗u) = 0.025.

The bootstrap confidence interval is then given by:xn − c∗u

sn√n

, xn − c∗lsn√n

.

23.9 b For a 95% confidence interval we need the empirical 0.025-quantile and the0.975-th quantile, which we estimate by the 25th and 976th order statistic: cl ≈ 1.862and cu ≈ −1.888. This results in

3.8715− 1.8621.9225√

2608, 3.8715− (−1.888)

1.9225√2608

= (3.801, 3.943).

23.9 c The (estimated) critical values that we would obtain from the table are−2.228 and 2.234, instead of −1.888 and 1.862 we used in part b. Hence, the resultinginterval would be larger.

23.10 a This interval has been obtained fromxn − tn−1,α/2

sn√n

, xn + tn−1,α/2sn√n

,

so that the sample mean is in the middle and must be equal to (1.6 + 7.8)/2 = 4.7.

23.10 b From the formula we see that half the width of the 95% confidence intervalis

7.8− 1.6

2= 3.1 = t15,0.025

sn√n

= 2.131sn√n

.

Similarly, half the width of the 99% confidence interval is

t15,0.005sn√n

= 2.947sn√n

=2.947

2.131· 2.131

sn√n

=2.947

2.131· 3.1 = 4.287.

Hence the 99% confidence interval is

(4.7− 4.287, 4.7 + 4.287) = (0.413, 8.987).

23.11 a For the 98% confidence interval the same formula is used as for the 95%interval, replacing the critical values by larger ones. This is the case, no matterwhether the critical values are from the normal or t-distribution, or from a bootstrapexperiment. Therefore, the 98% interval contains the 95%, and so must also containthe number 0.

Page 87: Probability and Statistics Book Solutions

29.1 Full solutions 543

23.11 b From a new bootstrap experiment we would obtain new and, most prob-ably, different values c∗u and c∗l . It therefore could be, if the number 0 is close tothe edge of the first bootstrap confidence interval, that it is just outside the newinterval.

23.11 c The new dataset will resemble the old one in many ways, but things like thesample mean would most likely differ from the old one, and so there is no guaranteethat the number 0 will again be in the confidence interval.

23.12 a This follows immediately from the change of units rule for normal randomvariables on page 112 and the fact that if the Zi’s are independent, so are theµ + σZi’s.

23.12 b We have

X =1

n

nXi=1

Xi =1

n

nXi=1

(µ + σZi) = µ + σ · 1

n

nXi=1

Zi = µ + σZ.

From this we also find that

Xi − X = (µ + σZi)− (µ + σZ) = σ(Zi − Z)

It follows that

S2X =

1

n− 1

nXi=1

(Xi − X)2 =1

n− 1

nXi=1

σ2(Zi − Zi)2 = σ2S2

Z ,

so that SX = σSZ .

23.12 c The equality follows immediately from part b, by inserting X − µ = σZ,and SX = σSZ . The right hand side is the studentized mean for a random samplefrom a N(0, 1) distribution, and therefore its distribution does not depend on µand σ. The left hand side is the studentized mean for a random sample from aN(µ, σ2) distribution. Since the two are equal, also their distributions are, whichmeans that the distribution of the studentized mean for a random sample from aN(µ, σ2) distribution (the left hand side) also does not depend on µ and σ.

24.1 From Section 24.1, using z0.05 = 1.645 we find the equation:70

100− p

2

− (1.645)2

100p(1− p) < 0.

This reduces to1.0271 p2 − 1.4271 p + 0.49 < 0.

The zeroes of this parabola are

p1,2 =−(−1.4271)±

p(−1.4271)2 − 4 · 1.0271 · 0.49

2 · 1.0271= 0.6947± 0.0746

and the 90% confidence interval is (0.6202, 0.7693).

24.2 a Since n = 6 is too small to expect a good approximation from the centrallimit theorem, we cannot apply the Wilson method.

Page 88: Probability and Statistics Book Solutions

544 Full solutions from MIPS: DO NOT DISTRIBUTE

24.2 b Solve 140

250− p

2

− (1.96)2

250p(1− p) = 0

or1.0154 p2 − 1.1354 p + 0.3136 = 0.

That is

p1,2 =−(−1.1354)±

p(−1.1354)2 − 4 · 1.0154 · 0.3136

2 · 1.0154= 0.5591± 0.0611

and the 90% confidence interval is (0.4980, 0.6202).

24.3 The width of the confidence interval is 2 · 2.576 · σ/√

n, where σ = 5. So, werequire (see Section 24.4):

n ≥

2 · 2.576 · 51

2

= (25.76)2 = 663.6,

that is, at least a sample size of 664.

24.4 a The width of the confidence interval will be about 2tn−1,0.05 s/√

n. For s wesubstitute our current estimate of σ, 0.75, and we use tn−1,0.05 = z0.05 = 1.645, forthe moment assuming that n will be large. This results in

n ≥

2 · 1.645 · 0.75

0.1

2

= 608.9,

so we use n = 609 (which is indeed large, so it is appropriate to use the critical valuefrom the normal distribution).

24.4 b In our computation, we used s = 0.75. From the new dataset of size 609 weare going to compute s609 and use that in the computation. If s609 > 0.75 then theconfidence interval will be too wide.

24.5 a From

P

−zα/2 <

X − nppn/4

zα/2

!= 1− α

we deduce

P

X

n− zα/2

2√

n< p <

X

n+

zα/2

2√

n

= 1− α.

So, the approximate 95% confidence interval isx

n− z0.05

2√

n,x

n+

z0.05

2√

n

.

The width is 2z0.05/(2√

n) = z0.05/√

n and so n should satisfy 1.96/√

n ≤ 0.01 orn ≥ (196)2 = 38416.

24.5 b The confidence interval is

19477

38416± 1.96

2 · √38416= 0.5070± 0.005,

so of the intended width.

Page 89: Probability and Statistics Book Solutions

29.1 Full solutions 545

24.6 a The environmentalists are interested in a lower confidence bound, becausethey would like to make a statement like “We are 97.5% confidence that the con-centration exceeds 1.68 ppm [and that is much too high.]” We have normal data,with σ unknown so we use s16 =

√1.12 = 1.058 as an estimate and use the criti-

cal value corresponding to 2.5% from the t(15) distribution: t15,0.025 = 2.131. Thelower confidence bound is 2.24−2.131 ·1.058/

√16 = 2.24−0.56 = 1.68, the interval:

(1.68,∞).

24.6 b For similar reasons, the plant management constructs an upper confidencebound (“We are 97.5% confident pollution does not exceed 2.80 [and this is ac-ceptable.]”). The computation is the same except for a minus sign: 2.24 + 2.131 ·1.058/

√16 = 2.24 + 0.56 = 2.80, so the interval is [0, 2.80). Note that the computed

upper and lower bounds are in fact the endpoints of the 95% two-sided confidenceinterval.

24.7 a From the normal approximation we know

P

−z0.025 <

Xn − µ√µ/√

n< z0.025

≈ 0.95

or

P

Xn − µ√

µ/√

n

2

< z20.025

!≈ 0.95,

i.e.,

P

Xn − µ

2< (1.96)2

µ√n

≈ 0.95.

Just as with the derivation of the Wilson method (Section 24.1) we now concludethat the 95% confidence interval contains those µ for which

(xn − µ)2 ≤ (1.96)2µ

n.

24.7 b We need to solve (xn − µ)2 − (1.96)2µ/n = 0, where xn = 3.8715 andn = 2608, resulting in

µ2 −

2 · 3.8715 +(1.96)2

2608

µ + (3.8715)2 = 0,

orµ2 − 7.7446µ + 14.9889 = 0.

From the roots we find the confidence interval (3.7967, 3.9478).

24.7 c The confidence interval (3.7967, 3.9478)is almost the same as the one in Ex-ercise 23.9 b. This is not surprising: n is very large, so the normal approximationshould be very good.

24.8 a We solve 15

23− p

2

− (1.96)2

23p(1− p) = 0

or1.1670 p2 − 1.4714 p + 0.4253 = 0,

from which we find the confidence interval (0.4489, 0.8119).

Page 90: Probability and Statistics Book Solutions

546 Full solutions from MIPS: DO NOT DISTRIBUTE

24.8 b Replacing the 1.96 in the equation above by z0.05 = 1.645, we find (0.4808, 0.7915).The appropriate one-sided confidence interval will provide us with a lower boundon the outer lane winning probability p, so it is (0.4808, 1].

24.9 a From Section 8.4 we know: P(M ≤ a) = [FX(a)]12, so P(M/θ ≤ t) =P(M ≤ θt) = [FX(θt)]12. Since Xi has a U(0, θ) distribution, FX(θt) = t, for0 ≤ t ≤ 1. Substituting this shows the result.

24.9 b For cl we need to solve (cl)12 = α/2, or cl = (α/2)1/12 = (0.05)1/12 = 0.7791.

For cu we need to solve (cu)12 = 1−α/2, or cu = (1−α/2)1/12 = (0.95)1/12 = 0.9958.

24.9 c From b we know that P(cl < M/θ < cu) = P(0.7790 < M/θ < 0.9958) =0.90. Rewriting this equation, we get: P(0.7790 θ < M < 0.9958 θ) = 0.90 andP(M/0.9958 < θ < M/0.7790) = 0.90. This means that (m/0.9958, m/0.7790) =(3.013, 3.851) is a 90% confidence interval for θ.

24.9 d From b we derive the general formula:

P

(α/2)1/n <

M

θ< (1− α/2)1/n

= 1− α.

The left hand inequality can be rewritten as θ < M/(α/2)1/n and the right handone as M/(1− α/2)1/n < θ. So, the statement above can be rewritten as:

P

M

(1− α/2)1/n< θ <

M

(α/2)1/n

= 1− α,

so that the general formula for the confidence interval becomes:m

(1− α/2)1/n,

m

(α/2)1/n

.

24.10 a From Section 11.2 we know that Sn, being the sum of n independent Exp(λ)random variables, has a Gam(n, λ) distribution. From Exercise 8.4 we know: λXi hasan Exp(1) distribution. Combining these facts, it follows that λSn = λX1+· · ·+λXn

has a Gam(n, 1) distribution.

24.10 b From the quantiles we see

0.9 = P(q0.05 ≤ λS20 ≤ q0.95)

= P(13.25 ≤ λS20 ≤ 27.88)

= P(13.25/S20 ≤ λ ≤ 27.88/S20) .

Noting that the realization of S20 is x1 + · · ·+ x20 = 20 x20, we conclude that13.25

20 x20,

27.88

20 x20

=

0.6625

x20,

1.394

x20

is a 95% confidence interval for λ.

25.1 The alternative hypothesis should reflect the belief that arrival delays of trainsexhibit more variation during rush hours than during quiet hours. Therefore takeH1 : σ1 > σ2.

25.2 The alternative hypothesis should reflect the belief that the number of babiesborn in Cleveland, Ohio, in the month of September in 1977 is higher than 1472.Therefore, take H1 : µ > 1472.

Page 91: Probability and Statistics Book Solutions

29.1 Full solutions 547

25.3 When the regression line runs through the origin, then α = 0. One possibletesting problem is to test H0 : α = 0 against H1 : α 6= 0. If H0 : α = 0 is rejected infavor of H1 : α 6= 0, the parameter α should be left in the model.

25.4 a Denote the observed numbers of cycles for the smokers by X1, X2, . . . , Xn1

and similarly Y1, Y2, . . . , Yn2 for the nonsmokers. A test statistic should compareestimators for p1 and p2. Since the geometric distributions have expectations 1/p1

and 1/p2, we could compare the estimator 1/Xn1 for p1 with the estimator 1/Yn2 forp2, or simply compare Xn1 with Yn2 . For instance, take test statistic T = Xn1− Yn2 .Values of T close to zero are in favor of H0, and values far away from zero are infavor of H1. Another possibility is T = Xn1/Yn2 .

25.4 b In this case, the maximum likelihood estimators p1 and p2 give better indi-cations about p1 and p2. They can be compared in the same way as the estimatorsin a.

25.4 c The probability of getting pregnant during a cycle is p1 for the smokingwomen and p2 for the nonsmokers. The alternative hypothesis should express thebelief that smoking women are less likely to get pregnant than nonsmoking women.Therefore take H1 : p1 < p2.

25.5 a When the maximum is greater than 5, at least one Xi is greater than 5 so thatθ must be greater than 5, and we know for sure that the null hypothesis is false.Therefore the set of relevant values of T1 = maxX1, X2, . . . , Xn is the interval[0, 5]. Similar to Exercise 8.15, one can argue that E[T1] = nθ/(n+1). Hence valuesof T1 close to 5n/(n + 1) are in favor of H0. Values of T1 in the neighborhood of 0indicate that θ < 5, and values of T1 very close to 5 indicate that θ > 5. Both theseregions are in favor of H1.

0 5nn+1 5

values of T1 infavor of H1

values of T1 infavor of H0

values of T1 infavor of H1

25.5 b When the distance between 2Xn and 5 is greater than 5, then 2Xn must begreater than 10, which means that at least one Xi is greater than 5. In that case weknow for sure that the null hypothesis is false. Therefore the set of relevant valuesof T2 = |2Xn − 5| is the interval [0, 5]. Since Xn will be close to θ/2, values of T2

close to zero are in favor of H0. Values of T2 far away from zero either correspondto 2Xn far below 5, which indicates θ < 5, or correspond to 2Xn far above 5, whichindicates θ > 5. Hence values of T2 far away from zero are in favor of H1.

0 5

values of T2 infavor of H0

values of T2 infavor of H1

25.6 a The p-value P(T ≥ 2.34) = 0.23 is larger than 0.05, so do not reject.

25.6 b The p-value P(T ≥ 2.34) = 1 − P(T ≤ 2.34) = 0.77 is larger than 0.05, sodo not reject.

25.6 c The p-value P(T ≥ 0.03) = 0.968 is larger than 0.05, so do not reject.

Page 92: Probability and Statistics Book Solutions

548 Full solutions from MIPS: DO NOT DISTRIBUTE

25.6 d The p-value P(T ≥ 1.07) = 1 − P(T ≤ 1.07) = 0.019 is less than 0.05, soreject.

25.6 e The p-value P(T ≥ 1.07) ≥ P(T ≥ 2.34) = 0.99, which is larger than 0.05,so do not reject.

25.6 f The p-value P(T ≥ 2.34) ≤ P(T ≥ 1.07) = 0.0.019, which is smaller than0.05, so reject.

25.6 g The p-value P(T ≥ 2.34) ≤ P(T ≥ 1.07) = 0.200. Therefore, the p-value issmaller than 0.200, but that does not give enough information to decide about thenull hypothesis.

25.7 a Since the parameter µ is the expectation of T , values of T much larger than1472 suggest that µ > 1472. Because we test H0 : µ = 1472 against H1 : µ > 1472,the more values of T are to the right, the stronger evidence they provide in favorof H1.

25.7 b According to part a, values to the right of t = 1718 bare stronger evidencein favor of H1. Therefore, the p-value is P(T ≥ 1718), where T has a Poisson dis-tribution with µ = 1472. Because the distribution of T can be approximated by anormal distribution with mean 1472 and variance 1472, we can approximate thisprobability as follows

P(T ≥ 1718) = P

T − 1472√

1472≥ 1718− 1472√

1472

≈ P(Z ≥ 6.412)

where Z has an N(0, 1) distribution. From Table ?? we see that the latter probabilityis almost zero (to be precise, 7.28 · 10−11, which was obtained using a statisticalsoftware package).

25.8 The values of Fn and Φ lie between 0 and 1, so that the maximal distancebetween the two graphs also lies between 0 and 1. In fact, since Fn has jumps of size1/n, the minimal value of T must be half the size of a jump: 1/(2n). This wouldcorrespond to the situation, where at each observation, the graph of Φ precisely runsthrough the middle of the two heights of Fn. When the graph of Fn lies far to theright (or to the left) of that of Φ, the maximum distance between the two graphscan be arbitrary close to 1.When the dataset is a realization from a distribution different from the standardnormal, the corresponding distribution function F will differ from Φ. Since Fn ≈ F(recall Table 17.2), the graph of Fn will show large differences with that of Φ resultingin a relatively large value of T . On the other hand, when the dataset is indeed arealization from the standard normal, then Fn ≈ Φ, resulting in a relative smallvalue of T . We conclude that only large values of T close to 1 are evidence againstthe null hypothesis.

25.9 Only values of Tks close to 1 are evidence against the null hypothesis. There-fore the p-value is P(Tks ≥ 0.176). On the basis of the bootstrap results, this proba-bility is approximated by the relative frequency of Tks-values greater than or equalto 0.176, which is zero.

25.10 a The alternative hypothesis should express the belief that the gross calorificexceeds 23.75 MJ/kg. Therefore take H1 : µ > 23.75.

Page 93: Probability and Statistics Book Solutions

29.1 Full solutions 549

25.10 b The p-value is the probability PXn ≥ 23.788

under the null hypothesis.

We can compute this probability by using that under the null hypothesis Xn has aN(23.75, (0.1)2/23) distribution:

PXn ≥ 23.788

= P

Xn − 23.75

0.1/√

23≥ 23.788− 23.75

0.1/√

23

= P(Z ≥ 1.82) ,

where Z has an N(0, 1) distribution. From Table ?? we find P(Z ≥ 1.82) = 0.0344.

25.11 A type I error occurs when µ = 0 and |t| ≥ 2. When µ = 0, then T has anN(0, 1) distribution. Hence, by symmetry of the N(0, 1) distribution and Table ??,we find that the probability of committing a type I error is

P(|T | ≥ 2) = P(T ≤ −2) + P(T ≥ 2) = 2 · P(T ≥ 2) = 2 · 0.0228 = 0.0456.

26.1 A type I error is to falsely reject the null hypothesis, i.e., to falsely conclude“suspect is guilty”. This happened in 9 out of 140 cases. Hence, the probability of atype I error is 9/140 = 0.064.A type II error is to falsely accept the null hypothesis, i.e., to falsely conclude“suspect is innocent”. This happened in 15 out of 140 cases. Hence, the probabilityof a type II error is 15/140 = 0.107.

26.2 According to Exercise 25.11, we do not reject if |T | < 2. Therefore the prob-ability of a type II error is

P(|T | < 2) = P(−2 < T < 2) = P(T < 2)− P(T < −2) ,

where T has a N(1, 1) distribution. Using that T − 1 has a N(0, 1) distribution, wefind that

P(T < 2)− P(T < −2) = P(T − 1 < 1)− P(T − 1 < −3)

= 0.8413− 0.0013 = 0.84.

26.3 a A type I error is to falsely reject the null hypothesis H0 : θ = 2. We rejectwhen X ≤ 0.1 or X ≥ 1.9. Because under the null hypothesis, X has a U(0, 2)distribution, the probability of committing a type I error is

P(X ≤ 0.1 | θ = 2) + P(X ≥ 1.9 | θ = 2) = 0.05 + 0.05 = 0.1.

26.3 b In this case a type II error is to falsely accept the null hypothesis whenθ = 2.5 We accept when 0.1 < X < 1.9. Because under θ = 2.5, X has a U(0, 2.5)distribution, the probability of committing a type I error is

P(0.1 < X < 1.9 | θ = 2.5) =1.9− 0.1

2.5= 0.72.

26.4 a Since T has a Bin(144, p) distribution, values of T close to 144/8 = 18 are infavor of the null hypothesis. Values of T far above 18 indicate that p > 1/8, whereasvalues far below 18 indicate that p < 1/8. This means we reject only for values of Tfar above 18. Hence we are only dealing with a right critical value.

26.4 b Denote the right critical value by c. Then we must solve P(T ≥ c) = 0.01.Using the normal approximation for the binomial distribution,

Page 94: Probability and Statistics Book Solutions

550 Full solutions from MIPS: DO NOT DISTRIBUTE

P(T ≥ c) ≈ P

Z ≥ c− npp

np(1− p)

!= P

0@Z ≥ c− 18q18 · 7

8

1A ,

where Z has a N(0, 1) distribution. We find c by solving

c− 18q18 · 7

8

= z0.01 = 2.326,

which gives c = 27.2. Because T can only take integer values, the right critical valueis taken to be 28, and the critical region is 28, 29, · · · , 144. Since the observednumber t = 29 falls in the critical region, we reject H0 : p = 1/8 in favor ofH1 : p > 1/8.

26.5 a The p-value is P(X ≥ 15) under the null hypothesis H0 : p = 1/2. UsingTable 26.3 we find P(X ≥ 15) = 1− P(X ≤ 14) = 1− 0.8950 = 0.1050.

26.5 b Only values close to 23 are in favor of H1 : p > 1/2, so the critical region isof the form K = c, c + 1, . . . , 23. The critical value c is the smallest value, suchthat P(X ≥ c) ≤ 0.05 under H0 : p = 1/2, or equivalently, 1−P(X ≤ c− 1) ≤ 0.05,which means P(X ≤ c− 1) ≥ 0.95. From Table 26.3 we conclude that c− 1 = 15, sothat K = 16, 17, . . . , 23.26.5 c A type I error occurs if p = 1/2 and X ≥ 16. The probability that thishappens is P(X ≥ 16 | p = 1/2) = 1 − P(X ≤ 15 | p = 1/2) = 1 − 0.9534 = 0.0466,where we have used Table 26.3 once more.

26.5 d In this case, a type II error occurs if p = 0.6 and X ≤ 15. To approximateP(X ≤ 15 | p = 0.6), we use the same reasoning as in Section 14.2, but now withn = 23 and p = 0.6. Write X as the sum of independent Bernoulli random variables:X = R1 + · · · + Rn, and apply the central limit theorem with µ = p = 0.6 andσ2 = p(1− p) = 0.24. Then

P(X ≤ 15) = P(R1 + · · ·+ Rn ≤ 15)

= P

R1 + · · ·+ Rn − nµ

σ√

n≤ 15− nµ

σ√

n

= P

Z23 ≥ 15− 13.8√

0.24√

23

≈ Φ(0.51) = 0.6950.

26.6 a Because T has a Pois(µ) distribution (see Exercise 25.7), we always haveE[T ] = µ. Therefore values of T around 1472 are in favor of the null hypothesis,values of T far to the left of 1472 are in favor of µ < 1472, and values of T far to theright 1472 are in favor of µ > 1472. Therefore, only values far to the right of 1472are in favor of H1 : µ > 1472, so that we only have a right critical value.

26.6 b Since, according to part a, we only have a right critical value c, we mustsolve P(T ≥ c) = 0.05. Using the normal approximation

P(T ≥ c) = P

T − µ√

µ≥ c− µ√

µ

≈ P

Z ≥ c− 1472√

1472

,

where Z has a N(0, 1) distribution. We find c by solving

c− 1472√1472

= z0.05 = 1.645,

Page 95: Probability and Statistics Book Solutions

29.1 Full solutions 551

which gives c = 1535.1. Because T can only take integer values we take right criticalvalue 1536, and critical region 1536, 1537,∞. The observed number 1718 falls intothe critical region, so that we reject H0 : µ = 1472 in favor of H1 : µ > 1472.

26.7 We must solve P(X1 + X2 ≤ c) = 0.05 under the null hypothesis. Under thenull hypothesis, X1 and X2 are independent random variables with a U(0, 1) distri-bution. According to Exercise 11.5, the random variable T = X1 + X2 has densityf(t) = t, for 0 ≤ t ≤ 1, and f(t) = 2− t, for 1 ≤ t ≤ 2. Since this density integratesto 0.5 for 0 ≤ t ≤ 1, we can find c by solvingZ c

0

t dt = P(T ≤ c) = 0.05,

or equivalently 12c2 = 0.05, which gives left critical value c = 0.316. The correspond-

ing critical region for T = X1 + X2 is [0, 0.316].

26.8 a Test statistic T = Xn takes values in (0,∞). Recall that the Exp(λ) distri-bution has expectation 1/λ, and that according to the law of large numbers Xn willbe close to 1/λ. Hence, values of Xn close to 1 are in favor of H0 : λ = 1, and onlyvalues of Xn close to zero are in favor H1 : λ > 1. Large values of Xn also provideevidence against H0 : λ = 1, but even stronger evidence against H1 : λ > 1. Weconclude that T = Xn has critical region K = (0, cl]. This is an example in whichthe alternative hypothesis and the test statistic deviate from the null hypothesis inopposite directions.Test statistic T ′ = e−Xn takes values in (0, 1). Values of Xn close to zero correspondto values of T ′ close to 1, and large values of Xn correspond to values of T ′ closeto 0. Hence, only values of T ′ close to 1 are in favor H1 : λ > 1. We conclude that T ′

has critical region K′ = [cu, 1). Here the alternative hypothesis and the test statisticdeviate from the null hypothesis in the same direction.

26.8 b Again, values of Xn close to 1 are in favor of H0 : λ = 1. Values of Xn closeto zero suggest λ > 1, whereas large values of Xn suggest λ < 1. Hence, both smalland large values of Xn are in favor of H1 : λ 6= 1. We conclude that T = Xn hascritical region K = (0, cl] ∪ [cu,∞).Small and large values of Xn correspond to values of T ′ close to 1 and 0. Hence,values of T ′ both close to 0 and close 1 are in favor of H1 : λ 6= 1. We conclude thatT ′ has critical region K′ = (0, c′l] ∪ [c′u, 1). Both test statistics deviate from the nullhypothesis in the same directions as the alternative hypothesis.

26.9 a Test statistic T = (Xn)2 takes values in [0,∞). Since µ is the expectationof the N(µ, 1) distribution, according to the law of large numbers, Xn is close to µ.Hence, values of Xn close to zero are in favor of H0 : µ = 0. Large negative valuesof Xn suggest µ < 0, and large positive values of Xn suggest µ > 0. Therefore, bothlarge negative and large positive values of Xn are in favor of H1 : µ 6= 0. Thesevalues correspond to large positive values of T , so T has critical region K = [cu,∞).This is an example in which the test statistic deviates from the null hypothesis inone direction, whereas the alternative hypothesis deviates in two directions.Test statistic T ′ takes values in (−∞, 0) ∪ (0,∞). Large negative values and largepositive values of Xn correspond to values of T ′ close to zero. Therefore, T ′ hascritical region K′ = [c′l, 0) ∪ (0, c′u]. This is an example in which the test statisticdeviates from the null hypothesis for small values, whereas the alternative hypothesisdeviates for large values.

Page 96: Probability and Statistics Book Solutions

552 Full solutions from MIPS: DO NOT DISTRIBUTE

26.9 b Only large positive values of Xn are in favor of µ > 0, which correspond tolarge values of T . Hence, T has critical region K = [cu,∞). This is an example wherethe test statistic has the same type of critical region with a one-sided or two-sidedalternative. Of course, the critical value cu in part b is different from the one inpart a.Large positive values of Xn correspond to small positive values of T ′. Hence, T ′ hascritical region K′ = (0, c′u]. This is another example where the test statistic deviatesfrom the null hypothesis for small values, whereas the alternative hypothesis deviatesfor large values.

27.1 a The value of the t-test statistic is

t =xn − 10

sn/√

n=

11− 10

2/√

4= 2.

The right critical value is tn−1,α/2 = t15,0.025 = 2.131. The observed value t = 2 issmaller than this, so we do not reject H0 : µ = 10 in favor of H1 : µ 6= 10.

27.1 b The right critical value is now tn−1,α = t15,0.05 = 1.753. The observed valuet = 2 is larger than this, so we reject H0 : µ = 10 in favor of H1 : µ > 10.

27.2 a The belief that the pouring temperature is at the right target setting is putto the test. The alternative hypothesis should represent the belief that the pouringtemperature differs from the target setting. Hence, test H0 : µ = 2550 againstH1 : µ 6= 2550.

27.2 b The value of the t-test statistic is

t =xn − 2550

sn/√

n=

2558.7− 2550√517.34/

√10

= 1.21.

Because H1 : µ 6= 2550, both small and large values of T are in favor of H1. Therefore,the right critical value is tn−1,α/2 = t9,0.025 = 3.169. The observed value t = 1.21 issmaller than this, so we do not reject H0 : µ = 2550 in favor of H1 : µ 6= 2550.

27.3 a The alternative hypothesis should represent the belief that the load at failureexceeds 10 MPa. Therefore, take H1 : µ > 10.

27.3 b The value of the t-test statistic is

t =xn − 10

sn/√

n=

13.71− 10

3.55/√

22= 4.902.

Because H1 : µ > 10, only large values of T are in favor of H1. Therefore, the rightcritical value is tn−1,α = t21,0.05 = 1.721. The observed value t = 4.902 is largerthan this, so we reject H0 : µ = 10 in favor of H1 : µ > 10.

27.4 The value of the t-test statistic is

t =xn − 31

sn/√

n=

31.012− 31

0.1294/√

22= 0.435.

Because H1 : µ > 31, only large values of T are in favor of H1. Therefore, the rightcritical value is tn−1,α = t21,0.01 = 2.518. The observed value t = 0.435 is smallerthan this, so we do not reject H0 : µ = 31 in favor of H1 : µ > 31.

Page 97: Probability and Statistics Book Solutions

29.1 Full solutions 553

27.5 a The interest is whether the inbreeding coefficient exceeds 0. Let µ representthis coefficient for the species of wasps. The value 0 is the a priori specified valueof the parameter, so test null hypothesis H0 : µ = 0. The alternative hypothesisshould express the belief that the inbreeding coefficient exceeds 0. Hence, we takealternative hypothesis H1 : µ > 0. The value of the test statistic is

t =0.044

0.884/√

197= 0.70.

27.5 b Because n = 197 is large, we approximate the distribution of T under thenull hypothesis by an N(0, 1) distribution. The value t = 0.70 lies to the right ofzero, so the p-value is the right tail probability P(T ≥ 0.70). By means of the normalapproximation we find from Table ?? that the right tail probability

P(T ≥ 0.70) ≈ 1− Φ(0.70) = 0.2420.

This means that the value of the test statistic is not very far in the (right) tail ofthe distribution and is therefore not to be considered exceptionally large. We do notreject the null hypothesis.

27.6 The belief that the intercept is zero is put to the test. The alternative hy-pothesis should represent the belief that the intercept differs from zero. Therefore,test H0 : α = 0 against H1 : α 6= 0. The value of the t-test statistic is

ta =α

sa=

5.388

1.874= 2.875.

Because H1 : α 6= 0, both small and large values of Ta are in favor of H1. Therefore,the right critical value is tn−1,α/2 = t5,0.05 = 2.015. The observed value t = 2.875 islarger than this, so we reject H0 : α = 0 in favor of H1 : α 6= 0.

27.7 a The data are modeled by a simple linear regression model: Yi = α + βxi,where Yi is the gas consumption and xi is the average outside temperature in the ithweek. Higher gas consumption as a consequence of smaller temperatures correspondsto β < 0. It is natural to consider the value 0 as the a priori specified value of theparameter (it corresponds to no change of gas consumption). Therefore, we take nullhypothesis H0 : β = 0. The alternative hypothesis should express the belief that thegas consumption increases as a consequence of smaller temperatures. Hence, we takealternative hypothesis H1 : β < 0. The value of the test statistic is

tb =β

sb=−0.3932

0.0196= −20.06.

The test statistic Tb has a t-distribution with n − 2 = 24 degrees of freedom. Thevalue −20.06 is smaller than the left critical value t24,0.05 = −1.711, so we reject.

27.7 b For the data after insulation, the value of the test statistic is

tb =−0.2779

0.0252= −11.03,

and Tb has a t(28) distribution. The value −11.03 is smaller than the left criticalvalue t28,0.05 = −1.701, so we reject.

Page 98: Probability and Statistics Book Solutions

554 Full solutions from MIPS: DO NOT DISTRIBUTE

28.1 a H0 : µ1 = µ2 and H1 : µ1 6= µ2; The value of the test statistic is tp = −2.130.The critical values are ±t142,0.025. These are not in Table ??, but 1.96 < t142,0.025 <2.009, so that we reject the null hypothesis.

28.1 b The value of the test statistic is the same (see Exercise 28.4). The nor-mal approximation yields that the critical values are ±1.96. The observed valuetd = −2.130 is smaller than the left critical value −1.96, so that we reject the nullhypothesis.

28.1 c The observed value td = −2.130 is smaller than the left critical value −2.004,so that we reject the null hypothesis. The salaries differ significantly.

28.2 First consider testing H0 : µ1 = µ2 against H1 : µ1 6= µ2. In view of theobserved sample variances, there is no reason to assume equal variances, so compute

s2d =

s2x

n+

s2y

m=

7.77

775+

25.33

261= 0.1071,

and

td =xn − ym

sd=

39.08− 37.59√0.1071

= 4.553.

With sample sizes n1 = 775 and n2 = 261 we can use the normal approximation. Thep-value is P(Td ≥ 4.553) ≈ 0, so that we reject H0 : µ1 = µ2 in favor of H1 : µ1 6= µ2.The other testing problems are handled in the same way. For testing H0 : µ1 = µ3

against H1 : µ1 6= µ3, we find

s2d =

7.77

775+

4.95

633= 0.0178 and td =

39.08− 39.60√0.0178

= −3.898

with p-value P(Td ≤ −3.898) ≈ 0, so that we reject H0 : µ1 = µ3 in favor ofH1 : µ1 6= µ3.For testing H0 : µ2 = µ3 against H1 : µ2 6= µ3, we find

s2d =

25.33

261+

4.95

633= 0.1049 and td =

37.59− 39.60√0.1049

= −6.206

with p-value P(Td ≤ −6.206) ≈ 0, so that we reject H0 : µ2 = µ3 in favor ofH1 : µ2 6= µ3.

28.3 a The value of the test statistic is

tp =22.43− 11.01

4.58= 2.492.

Under the assumption of normal data with equal variances, we must compare thiswith the right critical value t43,0.025. This is not Table ??, but 2.009 < t43,0.025 <2.021. Hence, tp > t43,0.025, so that we reject the null hypothesis.

28.3 b The value tp = 2.492 is greater than the right critical value 1.959, so thatwe reject on the basis of the bootstrap simulation.

28.3 c The value of the test statistic is

td =22.43− 11.01

4.64= 2.463.

Without the assumption of equal variances, and using the normal approximation,we must compare this with the right critical value z0.025 = 1.96. Since tp > 1.96, wereject the null hypothesis.

Page 99: Probability and Statistics Book Solutions

29.1 Full solutions 555

28.3 d Because we test at level 0.05, we reject if the right tail probability corre-sponding to td = 2.463 is smaller than 0.025. Since this is the case, we reject on thebasis of the bootstrap simulation.

28.4 When n = m, then

S2p =

(n− 1)S2X + (n− 1)S2

Y

n + n− 2

1

n+

1

n

=

(n− 1)(S2X + S2

Y )

2(n− 1)

2

n=

S2X

n+

S2Y

n= S2

d .

28.5 a When aS2X + bS2

Y is unbiased for σ2, we should have EaS2

X + bS2Y

= σ2.

Using that S2X and S2

Y are both unbiased for σ2, i.e., ES2

X

= σ2 and E

S2

Y

= σ2,

we getEaS2

X + bS2Y

= aE

S2

X

+ bE

S2

Y

= (a + b)σ2.

Hence, EaS2

X + bS2Y

= σ2 for all σ > 0 if and only if a + b = 1.

28.5 b By independence of S2X and S2

Y write

VaraS2

X + (1− a)S2Y

= a2Var

S2

X

+ (1− a)2Var

S2

Y

=

a2

n− 1+

(1− a)2

m− 1

2σ4.

To find the value of a that minimizes this, differentiate with respect to a and putthe derivative equal to zero. This leads to

2a

n− 1− 2(1− a)

m− 1= 0.

Solving for a yields a = (n − 1)/(n + m − 2). Note that the second derivative ofVaraS2

X + (1− a)S2Y

is positive so that this is indeed a minimum.

28.6 a By independence of X1, X2, . . . , Xn and Y1, Y2, . . . , Ym we have

VarXn − Ym

= Var

Xn

+ Var

Ym

=

σ2X

n+

σ2Y

m.

28.6 b

ES2

p

=

(n− 1)ES2

X

+ (m− 1)E

S2

Y

n + m− 2

1

n+

1

m

=

(n− 1)σ2X + (m− 1)σ2

Y

n + m− 2

1

n+

1

m

.

In principle this may differ from VarXn − Ym

= σ2

X/n + σ2Y /m.

28.6 c First note that

EaS2

X + bS2Y

= aE

S2

X

+ bE

S2

Y

= aσ2

X + bσ2Y .

For unbiasedness this must equal σ2X/n + σ2

Y /m for all σX > 0 and σY > 0, witha, b not depending on σX and σY . This is only possible for a = 1/n and b = 1/m.

Page 100: Probability and Statistics Book Solutions

556 Full solutions from MIPS: DO NOT DISTRIBUTE

28.6 d From part a together with σ2X = σ2

Y = σ2, we have

ES2

p

=

(n− 1)σ2X + (m− 1)σ2

Y

n + m− 2

1

n+

1

m

= σ2 · (n− 1) + (m− 1)

n + m− 2

1

n+

1

m

= σ2

1

n+

1

m

.

28.6 e No, not in general, see part b. If n = m, then according to (the computationsin) Exercise 28.4, S2

p = S2d . Since, according to part c, S2

d is always an unbiasedestimator for Var

Xn − Ym

it follows that S2

p is also an unbiased estimator forVarXn − Ym

. One may also check this as follows:

ES2

p

=

(n− 1)σ2X + (m− 1)σ2

Y

n + m− 2

1

n+

1

m

=

(n− 1)σ2X + (n− 1)σ2

Y

n + n− 2

1

n+

1

n

= (σ2

X + σ2Y ) · n− 1

n + n− 2

1

n+

1

n

= (σ2

X + σ2Y )

1

n=

σ2X

n+

σ2Y

n.